[obm-l] Re: [obm-l] Re: [obm-l] Dúvida e ajuda.

2022-04-08 Por tôpico Pedro José
Grato a todos!
Já, já tenho de voltar ao trabalho.
Depois dou uma olhada.
Mas achei a demonstração usando casa de pombos, simples e prática.
Já que tem de haver um p/q com pp temos w=x+p/q,
onde x é a parte inteira de w/q, então pq e os restos só podem q-1, uma hora tem de
repetir e aí volta a sequência.
Mas saindo do trabalho dou uma olhada.
Mais uma vez, minha gratidão.

Cordialmente,
PJMS



Em sex., 8 de abr. de 2022 às 13:02, Claudio Buffara <
claudio.buff...@gmail.com> escreveu:

> A volta é fácil também: ao calcular a representação decimal de a/b (a e b
> naturais), nas divisões sucessivas por b só existem b-1 restos possíveis
> (resto = 0 em alguma etapa implica numa decimal finita) e, portanto, após
> não mais do que b-1 divisões, um resto vai se repetir, marcando o início de
> um novo período na representação decimal.
>
> Agora, suponha que  X =
> 0,123456789112233445566778899111222333444555666777888999... seja racional.
> Então existirão n e p naturais tais que, a partir da n-ésima casa decimal
> (1/10^n), os algarismos de X vão se repetir numa sequência com período p.
>
> Mas, pela lei de formação de X, vai existir uma sequência de n+p+1
> algarismos iguais a 1, e esta sequência vai começar após a n-ésima casa
> decimal.
> Ou seja, a sequência vai estar incluída na parte periódica da
> representação decimal de X.
> Mas como o período é p, isso implica que a parte periódica teria que
> ser 111..11 (p algarismos 1) ==> contradição à lei de formação de X.
>
> []s,
> Claudio.
>
>
> On Fri, Apr 8, 2022 at 11:17 AM Pedro José  wrote:
>
>> Bom dia!
>> Posso concluir que um número representado por uma infinidade de
>> algarismos decimais é racional se e somente se tem um período de repetições
>> desses algarismos?
>> A ida é fácil se tiver o período é racional.
>> Já a volta não sei se é verdade e se for há como provar?
>>
>> Meu objetivo primário é saber se:
>> 0,123456789112233445566778899111222333444555666777888999... é racional.
>> As reticências se referem ao aumento de mais um algarismo repetido a cada
>> sequência, ou seja a primeira aparição de 1 será 1, a 2a 11 a 3a 111 e
>> assim sucessivamente, o mesmo vale para os demais algarismos.
>>
>> Alguém poderia me ajudar?
>> Grato,
>> PJMS
>>
>> --
>> Esta mensagem foi verificada pelo sistema de antivírus e
>> acredita-se estar livre de perigo.
>
>
> --
> Esta mensagem foi verificada pelo sistema de antivírus e
> acredita-se estar livre de perigo.

-- 
Esta mensagem foi verificada pelo sistema de antiv�rus e
 acredita-se estar livre de perigo.



[obm-l] Re: [obm-l] Re: [obm-l] Dúvida

2021-11-22 Por tôpico Pedro José
Boa tarde!

Grato, pela ajuda!
Não conheço.
Vou abrir um leque de estudo para tentar entender!
Valeu a curiosidade, com o que cheguei consegui matar o problema.
Genericamente, consegui que a solução levaria a uma expressão que era um
quadrado perfeito,esse era o objetivo. Só que me deu curiosidade, quanto a
resolução. Vou me enveredar no tema.

Cordialmente,
PJMS.

Em ter., 16 de nov. de 2021 às 17:29, Prof. Douglas Oliveira <
profdouglaso.del...@gmail.com> escreveu:

> Equação de Pell
>
> Em seg., 15 de nov. de 2021 13:36, Pedro José 
> escreveu:
>
>> Boa tarde!
>>
>> Alguém saberia como resolver a seguinte equação:
>>
>> x^2-7y^2=1, x,y em Z?
>>
>> Fiz a-7b=1 e achei a= 8 +7k e b=1 +K
>> Logo fica fácil que para k=-1 funciona x^2=1 e y^2=0.
>> Também funciona para k=8 x^2=64 e y^2=9.
>> Mas não sei nem como achar mais soluções nem como provar que só são essas.
>> Alguém poderia me dar uma orientação?
>>
>> Cordialmente,
>> PJMS
>>
>> --
>> Esta mensagem foi verificada pelo sistema de antivírus e
>> acredita-se estar livre de perigo.
>
>
> --
> Esta mensagem foi verificada pelo sistema de antivírus e
> acredita-se estar livre de perigo.

-- 
Esta mensagem foi verificada pelo sistema de antiv�rus e
 acredita-se estar livre de perigo.



[obm-l] Re: [obm-l] Re: [obm-l] Re: [obm-l] Dúvida

2019-12-05 Por tôpico Bernardo Freitas Paulo da Costa
Oi Pedro e Pedro, e demais colegas da OBM-L

Eu também nunca lera a definição de elipses através da razão entre as
distâncias.  Achei interessante, porque talvez permita "interpolar"
entre elipses, parábolas e hipérboles.  Mas até hoje, todas as
definições que eu vira de elipses (inclusive a da soma das distâncias)
incluíram círculos.  Mas, como talvez tenha desejado indicar o Pedro
Fonini, o importante é *para que serve a definição*.  No caso das
elipses, é muitas vezes importante incluir os círculos com elas, por
exemplo para o teorema de álgebra linear que ele citou.  Talvez o caso
da definição por razão das distâncias indique um outro caminho, mas aí
minha impressão é que o caso seria que a reta diretriz está no
infinito para os círculos (o que, mais uma vez, reforça a unidade das
cônicas no plano projetivo).

Acho que o mais comum, *hoje em dia*, é definir elipses de forma a
incluir os círculos.  Talvez isto invalide a definição que você deu
via razões, que passa a ser um teorema apenas para as elipses com dois
focos distintos.  Mas, por outro lado, permite generalizar de forma
mais natural outros teoremas para os quais a inclusão dos círculos
como elipses simplifique o enunciado.  Talvez você prefira a definição
por razões, mas voltando às origens das cônicas, onde a "classe" é
determinada pela posição relativa do plano secante com relação ao cone
gerador, acredito que a inclusão dos círculos junto com as elipses
seja totalmente razoável.

Cônicas suaves me parecem um assunto avançado; esta terminologia mesmo
já faz pensar em funções (infinitamente) diferenciáveis, etc, típicas
o ensino superior.  Do ponto de vista da geometria algébrica, uma
cõnica é definida como zeros de um polinômio P(x,y) de grau dois,
então esta "definição" não pode servir para separar quem seja suave e
quem não seja.  Se há uma diferença entre um círculo e duas retas que
se intersectam, não é pela regularidade da função que os define
implicitamente: num caso é x^2 + y^2 = 1, no outro, xy = 0.  O que
acaba servindo é a definição de "variedade suave".  E daí eu estaria
puxando mais ainda para temas universitários...

On Wed, Dec 4, 2019 at 9:33 PM Pedro José  wrote:
>
> Boa noite!
> As retas são cônicas degeneradas. Mas são cônicas.
> Definição de cônica :   Dada duas retas g,l concorrentes (cuja interseção é 
> {V} no |R3 que não sejam perpendiculares e um plano Pi. A interseção desse 
> plano com o cone K, reto de vértice V e eixo l , obtido pela rotação da reta 
> g ao redor do ponto V é uma cônica. Podemos ter uma reta, duas retas ou um 
> ponto como cônicas degeneradas.
> Você poderia até ter mencionado o conjunto vazio que não é uma cônica. 
> x^2+y^2=-1.
> Mas na verdade, eu não me expressei com rigor, o que queria dizer é que se 
> escrevermos a função quadrática F(x,y)= 0, que represente a cônica 
> (degenerada ou não) F(x,y) é suave? Ou as cônicas suaves devem ser não 
> degeneradas apenas?
> Outrossim, discordo do seu argumento "...geralmente é mais útil que as 
> definições dos objetos importantes não excluam os casos particulares.."
> Geralmente não é o balizador e sim a definição.
> 1 não é primo. Pois define-se que um primo deve ter dois divisores positivos 
> e 1 só possui um. Poderia argumentar, na sua linha, os dois divisores 
> coincidentes (os que afirmam é divisível por si e pela unidade)
> O quadrado por definição está claro que é retângulo.
> A definição da elipse é de que a soma das distâncias a dois pontos fixos (e 
> não um) é constante. Aí tem a forçação de se considerar dois como um só. Não 
> existe dois pontos coincidentes. Se são dois são distintos. Podemos 
> representar algo de várias maneiras mas se são iguais é só um, representado 
> de várias maneiras. Qual o cardinal do conjunto de focos de uma elipse, no 
> caso de você aceitar a elipse com um único foco?
> Como é a prova que só existe um vazio. Por hipótese há mais de um vazio, 
> vazio1 e vazio2 e no fim chega-se a conclusão que vazio1 = vazio2 e portanto 
> absurdo.Ora, podemos ter vazios coincidentes.
> Amigo, você afirma: "Nunca vi ninguém definir elipse de uma forma que exclua 
> os círculos."
> Você nem se deu ao trabalho de ler a minha nota, antes de comentar, ou então 
> me corrija se o círculo atende à:
> Lugar geométrico do plano em que a razão entre a distância de um ponto ao 
> foco direito e a distância entre esse ponto e uma reta (diretriz direita) é 
> constante e menor que 1 e igual a excentricidade da cônica.
> Como a excentricidade da circunferência é zero, teríamos que ter um ponto 
> fixo em que a distância de cada ponto da circunferência até esse ponto fosse 
> zero. E se na definição tem foco direito está implícito que há um esquerdo. 
> Vale a definição para foco esquerdo. Só atenderia se considerarmos o ponto 
> como uma circunferência de raio zero. E só para esse caso e ainda aceitarmos 
> que quando há só um foco ele tanto é direito quanto esquerdo. Grato pelos 
> comentários. Mas as dúvida persistem.
>
> Saudações

[obm-l] Re: [obm-l] Re: [obm-l] Dúvida

2019-12-04 Por tôpico Pedro José
Boa noite!
As retas são cônicas degeneradas. Mas são cônicas.
Definição de cônica :   Dada duas retas g,l concorrentes (cuja interseção é
{V} no |R3 que não sejam perpendiculares e um plano Pi. A interseção desse
plano com o cone K, reto de vértice V e eixo l , obtido pela rotação da
reta g ao redor do ponto V é uma cônica. Podemos ter uma reta, duas retas
ou um ponto como cônicas degeneradas.
Você poderia até ter mencionado o conjunto vazio que não é uma cônica.
x^2+y^2=-1.
Mas na verdade, eu não me expressei com rigor, o que queria dizer é que se
escrevermos a função quadrática F(x,y)= 0, que represente a cônica
(degenerada ou não) F(x,y) é suave? Ou as cônicas suaves devem ser não
degeneradas apenas?
Outrossim, discordo do seu argumento "...geralmente é mais útil que as
definições dos objetos importantes não excluam os casos particulares.."
Geralmente não é o balizador e sim a definição.
1 não é primo. Pois define-se que um primo deve ter dois divisores
positivos e 1 só possui um. Poderia argumentar, na sua linha, os dois
divisores coincidentes (os que afirmam é divisível por si e pela unidade)
O quadrado por definição está claro que é retângulo.
A definição da elipse é de que a soma das distâncias a dois pontos fixos (e
não um) é constante. Aí tem a forçação de se considerar dois como um só.
Não existe dois pontos coincidentes. Se são dois são distintos. Podemos
representar algo de várias maneiras mas se são iguais é só um, representado
de várias maneiras. Qual o cardinal do conjunto de focos de uma elipse, no
caso de você aceitar a elipse com um único foco?
Como é a prova que só existe um vazio. Por hipótese há mais de um vazio,
vazio1 e vazio2 e no fim chega-se a conclusão que vazio1 = vazio2 e
portanto absurdo.Ora, podemos ter vazios coincidentes.
Amigo, você afirma: "*Nunca vi ninguém definir elipse de uma forma que
exclua os círculos*."
Você nem se deu ao trabalho de ler a minha nota, antes de comentar, ou
então me corrija se o círculo atende à:
Lugar geométrico do plano em que a razão entre a distância de um ponto ao
foco direito e a distância entre esse ponto e uma reta (diretriz direita) é
constante e menor que 1 e igual a excentricidade da cônica.
Como a excentricidade da circunferência é zero, teríamos que ter um ponto
fixo em que a distância de cada ponto da circunferência até esse ponto
fosse zero. E se na definição tem foco direito está implícito que há um
esquerdo. Vale a definição para foco esquerdo. Só atenderia se
considerarmos o ponto como uma circunferência de raio zero. E só para esse
caso e ainda aceitarmos que quando há só um foco ele tanto é direito quanto
esquerdo. Grato pelos comentários. Mas as dúvida persistem.

Saudações,
PJMS



Em qua., 4 de dez. de 2019 às 19:59, Pedro Angelo 
escreveu:

> Em matemática, geralmente é mais útil que as definições dos objetos
> importantes não excluam os casos particulares. Um quadrado é um
> retângulo? Se vc quiser que a definição de "retângulo" inclua somente
> quadriláteros com ângulos retos que não sejam quadrados, vc tem que
> explicitar a parte do "não sejam quadrados" na definição. A definição
> mais simples, "retângulo é um quadrilátero cujos ângulos são todos
> retos" (como o nome já diz!) inclui o quadrado como caso especial. Uma
> coisa parecida ocorre com a elipse. Se vc quiser excluir o círculo, vc
> teria que especificar na definição que vc quer focos distintos. A
> definição mais simples, que cita os focos como sendo "dois pontos", ao
> invés de "dois pontos distintos", inclui o círculo como caso especial.
> E é útill que inclua mesmo. Por exemplo, se vc pensar o círculo como
> sendo um tipo especial de elipse, vc pode enunciar o seguinte teorema:
> "A imagem de uma elipse por uma transformação afim é outra elipse."
> Mas se vc achar que um círculo não é uma elipse, então o teorema (da
> forma que foi enunciado) não vale mais. A questão é que praticamente
> qualquer propriedade interessante apresentada por "elipses
> não-circulares" também será compartilhada pelos círculos. É raro em
> matemática vc precisar de uma elipse que seja proibida de ser um
> círculo. Nunca vi ninguém definir elipse de uma forma que exclua os
> círculos.
>
> Sobre a suavidade: da forma que vc escreveu, eu diria que está um
> pouco ruim. Por exemplo, a função
> F(x,y)=x^2-y^2
> é uma função suave (vc consegue calcular dF/dx e dF/dy, por exemplo).
> Mas vc diria que a equação F(x,y)=0 é uma "cônica suave"? Repare que
> essa equação descreve duas retas que se cruzam na origem. Outras
> funções problemáticas são F(x,y)=x^2+y^2 e F(x,y)=0.
>
> Se F(x,y) é um polinômio de segundo grau em x e y, então F(x,y)=0 é
> uma cônica, e eu diria que essa cônica é "suave" se nenhum dos pontos
> dela (pontos (x,y) tais que F(x,y)=0) satisfaz ao mesmo tempo dF/dx=0
> e dF/dy=0. O fato de pelo menos uma das derivadas parciais de F ser
> não-nula garante que não encontraremos problemas como os do parágrafo
> acima.
>
> abraços!
>
>
> Le mer. 4 déc. 2019 à 19:10, Pedro

[obm-l] Re: [obm-l] Re: [obm-l] Re: [obm-l] Dúvida basica equação polar

2019-09-02 Por tôpico Ralph Teixeira
Pois bem, se voce parametrizar com relacao ao centro, teria
x(teta)=1+cos(teta) e y(teta)=sin(teta). Se fosse assim, teria que ser
0 wrote:

> Caro Ralf, obrigado pela resposta.Para mim ficou confuso pq pensei que a
> parametrização do círculo se daria colocando como referencia o novo centro
> do mesmo. Quando penso em circulos diferentes , por exemplo residindo em
> apenas um quadrante tenho dificuldade de imaginar varrendo todos os pontos
> . Vou refletir sobre esses casos pois parecem ser obtidos como vc disse de
> fato.
>
> Att.Gabriel
>
> Em Seg, 2 de set de 2019 18:04, Ralph Teixeira 
> escreveu:
>
>> Bom, vale a pena fazer uma figura primeiro... Fez? Note como este circulo
>> estah nos primeiro e quarto quadrantes apenas.
>>
>> Entao suponho que voce fez as contas e descobriu que r=2cos(teta). No
>> quarto quadrante vale -pi/2> onde pi/2> terceiro quadrante). Por isso que -pi/2>
>> Para ser mas exato, o que aconteceria na equacao r=2cos(teta) para teta
>> entre pi/2 e 3pi/2... Tipo, experimente pensar em teta=pi para fazer um
>> exemplo. Jogando na equacao, ficaria r=2cos(pi)=-2?!?
>>
>> Aqui ha duas opcoes:
>> a) Alguns livros vao insistir que r>=0 sempre. Neste caso, fica claro que
>> pi/2> cosseno fica negativo.
>> b) Alguns outros livros sao mais "liberais" e permitem r<0 -- a
>> interpretacao seria que quando r eh negativo voce anda na reta que forma
>> angulo teta com o eixo x NO SENTIDO OPOSTO. Por exemplo, r=-2 e teta=pi eh
>> de fato o ponto (2,0) (voce anda na direcao NEGATIVA do eixo x, mas voce
>> anda -2, entao acaba andando para a DIREITA duas unidades). Neste caso,
>> colocar teta=pi dah um ponto no circulo sim senhor! Mas, mesmo assim, eu
>> usaria apenas -pi/2> ponto (2,0) JAH APARECEU com teta=0, e nao vejo porque conta-lo duas vezes
>> (e, dependendo da aplicacao, voce NAO QUER contar cada ponto duas vezes).
>>
>> Abraco, Ralph.
>>
>> On Mon, Sep 2, 2019 at 4:55 PM Gabriel Lopes  wrote:
>>
>>> Boa tarde, tenho uma duvida básica da representação em equação polar do
>>> círculo  (x-1)^2 +y^2= 1.
>>>
>>> Pq os intervalo de teta é de -pi/2 a pi/2 e nao de 0 a 2pi?
>>>
>>> --
>>> Esta mensagem foi verificada pelo sistema de antivírus e
>>> acredita-se estar livre de perigo.
>>
>>
>> --
>> Esta mensagem foi verificada pelo sistema de antivírus e
>> acredita-se estar livre de perigo.
>
>
> --
> Esta mensagem foi verificada pelo sistema de antivírus e
> acredita-se estar livre de perigo.

-- 
Esta mensagem foi verificada pelo sistema de antiv�rus e
 acredita-se estar livre de perigo.



[obm-l] Re: [obm-l] Re: [obm-l] Dúvida basica equação polar

2019-09-02 Por tôpico Gabriel Lopes
Caro Ralf, obrigado pela resposta.Para mim ficou confuso pq pensei que a
parametrização do círculo se daria colocando como referencia o novo centro
do mesmo. Quando penso em circulos diferentes , por exemplo residindo em
apenas um quadrante tenho dificuldade de imaginar varrendo todos os pontos
. Vou refletir sobre esses casos pois parecem ser obtidos como vc disse de
fato.

Att.Gabriel

Em Seg, 2 de set de 2019 18:04, Ralph Teixeira  escreveu:

> Bom, vale a pena fazer uma figura primeiro... Fez? Note como este circulo
> estah nos primeiro e quarto quadrantes apenas.
>
> Entao suponho que voce fez as contas e descobriu que r=2cos(teta). No
> quarto quadrante vale -pi/2 onde pi/2 terceiro quadrante). Por isso que -pi/2
> Para ser mas exato, o que aconteceria na equacao r=2cos(teta) para teta
> entre pi/2 e 3pi/2... Tipo, experimente pensar em teta=pi para fazer um
> exemplo. Jogando na equacao, ficaria r=2cos(pi)=-2?!?
>
> Aqui ha duas opcoes:
> a) Alguns livros vao insistir que r>=0 sempre. Neste caso, fica claro que
> pi/2 cosseno fica negativo.
> b) Alguns outros livros sao mais "liberais" e permitem r<0 -- a
> interpretacao seria que quando r eh negativo voce anda na reta que forma
> angulo teta com o eixo x NO SENTIDO OPOSTO. Por exemplo, r=-2 e teta=pi eh
> de fato o ponto (2,0) (voce anda na direcao NEGATIVA do eixo x, mas voce
> anda -2, entao acaba andando para a DIREITA duas unidades). Neste caso,
> colocar teta=pi dah um ponto no circulo sim senhor! Mas, mesmo assim, eu
> usaria apenas -pi/2 ponto (2,0) JAH APARECEU com teta=0, e nao vejo porque conta-lo duas vezes
> (e, dependendo da aplicacao, voce NAO QUER contar cada ponto duas vezes).
>
> Abraco, Ralph.
>
> On Mon, Sep 2, 2019 at 4:55 PM Gabriel Lopes  wrote:
>
>> Boa tarde, tenho uma duvida básica da representação em equação polar do
>> círculo  (x-1)^2 +y^2= 1.
>>
>> Pq os intervalo de teta é de -pi/2 a pi/2 e nao de 0 a 2pi?
>>
>> --
>> Esta mensagem foi verificada pelo sistema de antivírus e
>> acredita-se estar livre de perigo.
>
>
> --
> Esta mensagem foi verificada pelo sistema de antivírus e
> acredita-se estar livre de perigo.

-- 
Esta mensagem foi verificada pelo sistema de antiv�rus e
 acredita-se estar livre de perigo.



[obm-l] Re: [obm-l] Re: [obm-l] Re: [obm-l] Dúvida

2019-05-24 Por tôpico Anderson Torres
Em dom, 19 de mai de 2019 às 13:24, Pedro José 
escreveu:

> Bom dia!
> Anderson,
> obrigado. Porém faltou-me saber se os entendimentos anteriores estão
> corretos.
>

O texto não tinha nenhum glossário para ajudar, ou uma referência do
gênero? Alguns bons livros de Teoria dos Números, em especial os grossões
como o do Hua Loo-Keng, costumam dedicar a primeira folha a convenções.

Eu até suspeito que esses teus entendimentos estejam corretos - dado que
são bem parecidos com outros já consagrados pelo uso - mas não bato o
martelo porque não tenho o contexto.



> Grato,
> PJMS
>
> Em sáb, 18 de mai de 2019 13:27, Anderson Torres <
> torres.anderson...@gmail.com escreveu:
>
>>
>>
>> Em sex, 17 de mai de 2019 às 10:49, Pedro José 
>> escreveu:
>>
>>> Bom dia!
>>>
>>> Tenho uma dúvida sobre os simbolismos, que aparecem recorrentemente, em
>>> artigos sobre teoria dos números, mas que não encontro a definição :
>>> Z[i]/(α) - Entendi como o conjunto das classes de equivalências mod α
>>> em Z{i}
>>> Z[i]/(α)* - Entendi como as classes de equivalência mod α em Z[i], que
>>> são inversíveis.
>>>
>>> Tentando compreender uma demonstração de que todos os números que não
>>> podem ser escritos da forma 4^k(8m+7) com k,m>=0, aceitam ser escritos como
>>> a soma de três parcelas, todas quadrados, me deparei com  [image:
>>> image.png] . O que significa?
>>>
>>
>> Algo como números da forma a+b*sqrt(m) onde a e b são racionais.
>>
>>
>>
>>>
>>> Os outros entendimentos estão corretos?
>>>
>>> Saudações,
>>> PJMS
>>>
>>>
>>> --
>>> Esta mensagem foi verificada pelo sistema de antivírus e
>>> acredita-se estar livre de perigo.
>>
>>
>> --
>> Esta mensagem foi verificada pelo sistema de antivírus e
>> acredita-se estar livre de perigo.
>
>
> --
> Esta mensagem foi verificada pelo sistema de antivírus e
> acredita-se estar livre de perigo.

-- 
Esta mensagem foi verificada pelo sistema de antiv�rus e
 acredita-se estar livre de perigo.



[obm-l] Re: [obm-l] Re: [obm-l] Dúvida

2019-05-19 Por tôpico Pedro José
Bom dia!
Anderson,
obrigado. Porém faltou-me saber se os entendimentos anteriores estão
corretos.

Grato,
PJMS

Em sáb, 18 de mai de 2019 13:27, Anderson Torres <
torres.anderson...@gmail.com escreveu:

>
>
> Em sex, 17 de mai de 2019 às 10:49, Pedro José 
> escreveu:
>
>> Bom dia!
>>
>> Tenho uma dúvida sobre os simbolismos, que aparecem recorrentemente, em
>> artigos sobre teoria dos números, mas que não encontro a definição :
>> Z[i]/(α) - Entendi como o conjunto das classes de equivalências mod α em
>> Z{i}
>> Z[i]/(α)* - Entendi como as classes de equivalência mod α em Z[i], que
>> são inversíveis.
>>
>> Tentando compreender uma demonstração de que todos os números que não
>> podem ser escritos da forma 4^k(8m+7) com k,m>=0, aceitam ser escritos como
>> a soma de três parcelas, todas quadrados, me deparei com  [image:
>> image.png] . O que significa?
>>
>
> Algo como números da forma a+b*sqrt(m) onde a e b são racionais.
>
>
>
>>
>> Os outros entendimentos estão corretos?
>>
>> Saudações,
>> PJMS
>>
>>
>> --
>> Esta mensagem foi verificada pelo sistema de antivírus e
>> acredita-se estar livre de perigo.
>
>
> --
> Esta mensagem foi verificada pelo sistema de antivírus e
> acredita-se estar livre de perigo.

-- 
Esta mensagem foi verificada pelo sistema de antiv�rus e
 acredita-se estar livre de perigo.



[obm-l] Re: [obm-l] Re: [obm-l] dúvida sobre a OBMU

2019-01-20 Por tôpico Pedro Soares
Combinatória aproveita bastante.
E pra exemplificar o que pode ter em comum, esse ano o problema 6 do Nível
U também estava na prova do nível 3 (não sei o número do problema)

On Sat, 19 Jan 2019 at 09:42, Anderson Torres 
wrote:

> Em sáb, 12 de jan de 2019 às 16:41, Luiz Kv
>  escreveu:
> >
> > Olá, boa tarde, tudo bom ?
> >
> > Gostaria de saber quais conteúdos caem na OBMU diferentes do nível 3 da
> OBM
>
> Acho que, bem, tudo! Dificilmente tem algo que se aproveite
> diretamente. No máximo Combinatória, já vi uma questão de Combinatória
> que poderia ser facilmente dada para o Nível 3.
>
> > Quais as melhores fontes para estudar ?
>
> As provas antigas, as provas da IMC e provas de Universitárias ao
> redor do mundo, como a Putnam dos EUA.
>
> E algumas revistas internacionais :)
>
> > Existe idade máxima para a OBMU ?
>
> Não exatamente, Acho que só cobram ser estudante universitário.
> Algumas competições internacionais cobram idade, porém.
>
>
> >
> > Obrigado, até mais :)
> >
> > --
> > Esta mensagem foi verificada pelo sistema de antivírus e
> > acredita-se estar livre de perigo.
>
> --
> Esta mensagem foi verificada pelo sistema de antivírus e
>  acredita-se estar livre de perigo.
>
>
> =
> Instru�ões para entrar na lista, sair da lista e usar a lista em
> http://www.mat.puc-rio.br/~obmlistas/obm-l.html
> =
>

-- 
Esta mensagem foi verificada pelo sistema de antiv�rus e
 acredita-se estar livre de perigo.



[obm-l] Re: [obm-l] Re: [obm-l] Dúvida conceitual (equações)

2018-10-15 Por tôpico Pedro José
Boa tarde!
Artur, não sou contrário a multiplicidade da raiz. Porém, mesmo coma a
multiplicidade, a raiz continua sendo única.
Todavia,não há como negar, facilita sobremaneira as relações de Girard,
para soma e produto é fácil de ajeitar, mas quando passamos a somatório de
produtos dois a dois, três a três... ficaria complicado.

Saudações,
PJMS

Em seg, 15 de out de 2018 às 12:45, Artur Steiner <
artur.costa.stei...@gmail.com> escreveu:

> Isso de se considerar multiplicidades no número de raízes de um polinômio
> é uma convenção conveniente. Facilita muito no caso, por exemplo, das
> famosas relações de Girard. Elas só funcionam se considerarmos as
> multiplicidades. Em análise complexa há também vários teoremas relativos a
> funções analíticas que contam os zeros da função contando multiplicidades.
>
> É claro, por exemplo, que o conjunto de zeros (ou raízes) da função f(x) =
> x^3  é {0}. É uma única raiz com multiplicidade 3. Mas em muitas aplicações
> é mais conveniente supor que são 3 raízes iguais a 0.Sem esquecer que esta
> f só se anula para x = 0.
>
> Há muitas convenções convenientes na matemática. Por exemplo, embora a
> soma seja uma operação binária, convenciona-se que uma soma de uma única
> parcela é a própria parcela. Isto facilita muito.
>
> Artur Costa Steiner
>
> Em dom, 14 de out de 2018 06:33, Vanderlei Nemitz 
> escreveu:
>
>> Bom dia!
>> Na seguinte questão, que me foi apresentada por um aluno, a resposta
>> proposta é a alternativa C (1/2). Eu sempre pensei que apenas
>> considerávamos multiplicidades em equações polinomiais. Como essa é uma
>> equação exponencial, obtive a resposta B (-1/2). O que é correto pensar?
>>
>> O produto das raízes da equação 16.4^3x - 40.4^2x + 17.4^x - 2 = 0 é
>> igual a:
>> A) 1
>> B) - 0,5
>> C) 0,5
>> D) - 1
>> E) 0
>>
>> Muito obrigado!
>>
>> --
>> Esta mensagem foi verificada pelo sistema de antivírus e
>> acredita-se estar livre de perigo.
>
>
> --
> Esta mensagem foi verificada pelo sistema de antivírus e
> acredita-se estar livre de perigo.

-- 
Esta mensagem foi verificada pelo sistema de antiv�rus e
 acredita-se estar livre de perigo.



[obm-l] Re: [obm-l] Re: [obm-l] Re: [obm-l] Re: [obm-l] Dúvida conceitual (equações)

2018-10-15 Por tôpico Bernardo Freitas Paulo da Costa
On Mon, Oct 15, 2018 at 8:07 AM Claudio Buffara
 wrote:
>
> Derivando e igualando a zero o lado esquerdo da sua equação, ficamos com:
> -2*cos(x)*sen(x) + sen(x) = 0 ==>
> sen(x) = 0  ou  cos(x) = 1/2 ==>
> x = 0 ou x = pi ou x = 2pi
> ou x = pi/3 ou x = 5pi/3.
>
> Assim, uma definição que me parece adequado para equações em geral (e não 
> necessariamente polinomiais) da forma f(x) = 0 é que uma raiz de 
> multiplicidade n é raiz de f, f’, ... , f^(n-1) mas não é raiz de f^(n).
>
> Naturalmente, se f não tiver todas as derivadas, precisaremos achar uma 
> definição diferente. Mas talvez, neste caso, nem faça sentido falar em 
> multiplicidade de uma raiz.

Essa definição funciona relativamente bem se f é analítica, porque o
comportamento local é determinado por inteiros.  Se f for apenas
diferenciável, talvez seja complicado dizer algo, como o exemplo
clássico de exp(-1/x^2).  A raiz tem multiplicidade infinita?

Enfim, existem, como você falou, boas razões para incorporar
multiplicidade (por exemplo estabilidade numérica), mas isso em geral
só faz sentido no mundo analítico, onde a noção de "grau" é dada pelas
derivadas.  Acho que mesmo no mundo C-infinito já pode haver
problemas, mas não sou especialista (nessas :D) patologias.  A questão
original, incluindo multiplicidades, pode ser resolvida simplesmente
usando as relações de Girard, que dependem de forma simples da
equação.

Vou tentar dar um exemplo que ilustra meu ponto de vista:  qual o
produto das raízes da equação x^2 - 4x + c?  "Qualquer um" dirá "c".
Mas, naturalmente, se c = 4, a única solução é x=2, e portanto (sem
usar multiplicidades) este produto seria apenas 2.  E daí a fórmula
fica muito mais complicada, com um caso especial, e descontínua.  A
grande sacada do Girard foi, justamente, propor incorporar as
multiplicidades, para simplificar as fórmulas (além, é claro, de
incluir também as soluções negativas, antes consideradas como
"absurdas" - este foi, provavelmente, o maior motivo de as pessoas
considerarem raízes negativas como algo que fazia sentido, e portanto
os números negativos também).  Mas isso não quer dizer que a equação
x^2 - 4x + 4 tenha duas soluções.  É apenas uma forma mais conveniente
de interpretar as raízes quando se pensam nas relações de Girard (e
várias outras fórmulas).  Neste sentido, acho que este tipo de questão
mais atrapalha (porque "era só para usar a fórmula") - a menos que,
justamente, se discuta *porque* falamos de multiplicidade: para que as
fórmulas fiquem mais simples (e você pode incluir "bonitas" também,
por minha conta).  Nada mais.  E esta "simplificação" do entendimento
através da simplificação das fórmulas não se justifica sempre: este
mesmo debate sobre multiplicidades leva a considerar objetos no
infinito (para que todas as retas se intersectem sempre em um ponto),
complexos (para x^2 + 1 = 0 ter raiz), etc.  Muitas vezes, é útil ter
esse entendimento unificado, onde tudo "só depende do grau".  Mas será
mesmo que se eu perguntar para você "em quantos pontos a reta x=3
corta a parábola y=x^2?" você vai dizer "2, é óbvio"?

Abraços,
-- 
Bernardo Freitas Paulo da Costa

-- 
Esta mensagem foi verificada pelo sistema de antiv�rus e
 acredita-se estar livre de perigo.


=
Instru��es para entrar na lista, sair da lista e usar a lista em
http://www.mat.puc-rio.br/~obmlistas/obm-l.html
=


Re: [obm-l] Re: [obm-l] Re: [obm-l] Dúvida conceitual (equações)

2018-10-15 Por tôpico Kevin Felipe Kühl Oliveira
Exatamente nisso que estava pensando. Se fizessemos 4^x = y teriamos uma 
equação polinomial de grau 3, ai fica mais evidente a existência de múltiplas 
raizes.

Abraços

Kevin Kühl
On 15 Oct 2018 07:25 -0300, Claudio Buffara , wrote:
> Qual a soma das raizes de (2^x - 8)^3 = 0?
> Se a equação acima fosse apresentada como:
> 2^(3x) - 24*2^(2x) + 192*2^x - 512 = 0,
> isso mudaria sua resposta?
>
> Enviado do meu iPhone
>
> Em 15 de out de 2018, à(s) 00:29, Vanderlei Nemitz  
> escreveu:
>
> > Valeu, Pedro! Tomara que mais alguém emita sua opinião.
> > Um abraço!
> >
> > > Em dom, 14 de out de 2018 18:59, Pedro José  
> > > escreveu:
> > > > Boa noite!
> > > > Bom questionamento. Vou me posicionar na arquibancada.Â
> > > > Minha posição é controversa. Se quer se levar em conta a repetição 
> > > > tem que se falar do produto das raízes, cada elevada a sua 
> > > > multiplicidade. No caso de soma, cada raiz multiplicada pela 
> > > > multiplicidade.
> > > > Para esse exemplo, o conjunto solução é {1/2,-1} então o produto é 
> > > > -1/2.
> > > > Em suma, não aceito n raízes iguais, mas sim uma raiz de 
> > > > multiplicidade n.
> > > > Se quando queremos provar que algo é unico supomos a existência de 
> > > > dois e provamos que são iguais. Creio que seja contraditório dois ou 
> > > > nais iguais.
> > > > Mas vamos observar as diversas posições, pois, creio que o assunto 
> > > > não seja pacífico.Â
> > > > Saudações,Â
> > > > PJMSÂ
> > > >
> > > > > Em Dom, 14 de out de 2018 06:33, Vanderlei Nemitz 
> > > > >  escreveu:
> > > > > > Bom dia!
> > > > > > Na seguinte questão, que me foi apresentada por um aluno, a 
> > > > > > resposta proposta é a alternativa C (1/2). Eu sempre pensei que 
> > > > > > apenas considerávamos multiplicidades em equações polinomiais. 
> > > > > > Como essa é uma equação exponencial, obtive a resposta B (-1/2). 
> > > > > > O que é correto pensar?
> > > > > >
> > > > > > O produto das raízes da equação 16.4^3x - 40.4^2x + 17.4^x - 2 = 
> > > > > > 0 é igual a:
> > > > > > A) 1
> > > > > > B) - 0,5
> > > > > > C) 0,5
> > > > > > D) - 1
> > > > > > E) 0
> > > > > >
> > > > > > Muito obrigado!
> > > > > >
> > > > > > --
> > > > > > Esta mensagem foi verificada pelo sistema de antivírus e
> > > > > > acredita-se estar livre de perigo.
> > > >
> > > > --
> > > > Esta mensagem foi verificada pelo sistema de antivírus e
> > > > acredita-se estar livre de perigo.
> >
> > --
> > Esta mensagem foi verificada pelo sistema de antivírus e
> > acredita-se estar livre de perigo.
>
> --
> Esta mensagem foi verificada pelo sistema de antiv�rus e
> acredita-se estar livre de perigo.

-- 
Esta mensagem foi verificada pelo sistema de antiv�rus e
 acredita-se estar livre de perigo.



Re: [obm-l] Re: [obm-l] Re: [obm-l] Re: [obm-l] Dúvida conceitual (equações)

2018-10-15 Por tôpico Claudio Buffara
Derivando e igualando a zero o lado esquerdo da sua equação, ficamos com:
-2*cos(x)*sen(x) + sen(x) = 0 ==>
sen(x) = 0  ou  cos(x) = 1/2 ==>
x = 0 ou x = pi ou x = 2pi
ou x = pi/3 ou x = 5pi/3.

Assim, uma definição que me parece adequado para equações em geral (e não 
necessariamente polinomiais) da forma f(x) = 0 é que uma raiz de multiplicidade 
n é raiz de f, f’, ... , f^(n-1) mas não é raiz de f^(n).

Naturalmente, se f não tiver todas as derivadas, precisaremos achar uma 
definição diferente. Mas talvez, neste caso, nem faça sentido falar em 
multiplicidade de uma raiz.

Enviado do meu iPhone

Em 15 de out de 2018, à(s) 08:13, Vanderlei Nemitz  
escreveu:

> Claudio:
> Eu ficaria com a mesma dúvida!
> Pensaria em apenas uma raiz.
> 
> Qual é a soma das raízes da equação (cos x)^2 - cos x + 1/4 = 0 no 
> intervalo [0, 2pi]?
> 
> Em seg, 15 de out de 2018 07:00, Claudio Buffara  
> escreveu:
>> Qual a soma das raizes de (2^x - 8)^3 = 0?
>> Se a equação acima fosse apresentada como:
>> 2^(3x) - 24*2^(2x) + 192*2^x - 512 = 0,
>> isso mudaria sua resposta?
>> 
>> Enviado do meu iPhone
>> 
>> Em 15 de out de 2018, Ã (s) 00:29, Vanderlei Nemitz  
>> escreveu:
>> 
>>> Valeu, Pedro! Tomara que mais alguém emita sua opinião.
>>> Um abraço!
>>> 
>>> Em dom, 14 de out de 2018 18:59, Pedro José  
>>> escreveu:
 Boa noite!
 Bom questionamento. Vou me posicionar na arquibancada. 
 Minha posição é controversa. Se quer se levar em conta a 
 repetição tem que se falar do produto das raízes, cada elevada a 
 sua multiplicidade. No caso de soma, cada raiz multiplicada pela 
 multiplicidade.
 Para esse exemplo, o conjunto solução é {1/2,-1} então o 
 produto é -1/2.
 Em suma, não aceito n raízes iguais, mas sim uma raiz de 
 multiplicidade n.
 Se quando queremos provar que algo é unico supomos a existência de 
 dois e provamos que são iguais. Creio que seja contraditório dois ou 
 nais iguais.
 Mas vamos observar as diversas posições, pois, creio que o assunto 
 não seja pacífico. 
 Saudações, 
 PJMS 
 
 Em Dom, 14 de out de 2018 06:33, Vanderlei Nemitz  
 escreveu:
> Bom dia!
> Na seguinte questão, que me foi apresentada por um aluno, a resposta 
> proposta é a alternativa C (1/2). Eu sempre pensei que apenas 
> considerávamos multiplicidades em equações polinomiais. Como 
> essa é uma equação exponencial, obtive a resposta B (-1/2). O 
> que é correto pensar?
> 
> O produto das raízes da equação 16.4^3x - 40.4^2x + 17.4^x - 2 = 
> 0 é igual a:
> A) 1
> B) - 0,5
> C) 0,5
> D) - 1
> E) 0
> 
> Muito obrigado!
> 
> -- 
> Esta mensagem foi verificada pelo sistema de antivírus e 
> acredita-se estar livre de perigo.
 
 -- 
 Esta mensagem foi verificada pelo sistema de antivírus e 
 acredita-se estar livre de perigo.
>>> 
>>> -- 
>>> Esta mensagem foi verificada pelo sistema de antivírus e 
>>> acredita-se estar livre de perigo.
>> 
>> -- 
>> Esta mensagem foi verificada pelo sistema de antivírus e 
>> acredita-se estar livre de perigo.
> 
> -- 
> Esta mensagem foi verificada pelo sistema de antivírus e 
> acredita-se estar livre de perigo.

-- 
Esta mensagem foi verificada pelo sistema de antiv�rus e
 acredita-se estar livre de perigo.



Re: [obm-l] Re: [obm-l] Re: [obm-l] Re: [obm-l] Dúvida conceitual (equações)

2018-10-15 Por tôpico Claudio Buffara
Pensando só como uma equação, talvez faça sentido não considerar a 
multiplicidade.

Mas, no seu exemplo, no intervalo [0,2pi], os gráficos de 
f(x) = cos(x) - 1/2 
e de
g(x) = (cos(x) - 1/2)^2
tem um comportamento bem distinto um do outro em vizinhanças de pi/3 e 5pi/3.
Por exemplo, o gráfico de f corta o eixo x em pi/3 enquanto que o de g apenas 
tangencia o eixo neste ponto.
Idem pros outros exemplos.
Isso sugere que, mesmo nestes casos, talvez seja conveniente considerar a 
multiplicidade de uma raiz.

Enviado do meu iPhone

Em 15 de out de 2018, à(s) 08:13, Vanderlei Nemitz  
escreveu:

> Claudio:
> Eu ficaria com a mesma dúvida!
> Pensaria em apenas uma raiz.
> 
> Qual é a soma das raízes da equação (cos x)^2 - cos x + 1/4 = 0 no 
> intervalo [0, 2pi]?
> 
> Em seg, 15 de out de 2018 07:00, Claudio Buffara  
> escreveu:
>> Qual a soma das raizes de (2^x - 8)^3 = 0?
>> Se a equação acima fosse apresentada como:
>> 2^(3x) - 24*2^(2x) + 192*2^x - 512 = 0,
>> isso mudaria sua resposta?
>> 
>> Enviado do meu iPhone
>> 
>> Em 15 de out de 2018, Ã (s) 00:29, Vanderlei Nemitz  
>> escreveu:
>> 
>>> Valeu, Pedro! Tomara que mais alguém emita sua opinião.
>>> Um abraço!
>>> 
>>> Em dom, 14 de out de 2018 18:59, Pedro José  
>>> escreveu:
 Boa noite!
 Bom questionamento. Vou me posicionar na arquibancada. 
 Minha posição é controversa. Se quer se levar em conta a 
 repetição tem que se falar do produto das raízes, cada elevada a 
 sua multiplicidade. No caso de soma, cada raiz multiplicada pela 
 multiplicidade.
 Para esse exemplo, o conjunto solução é {1/2,-1} então o 
 produto é -1/2.
 Em suma, não aceito n raízes iguais, mas sim uma raiz de 
 multiplicidade n.
 Se quando queremos provar que algo é unico supomos a existência de 
 dois e provamos que são iguais. Creio que seja contraditório dois ou 
 nais iguais.
 Mas vamos observar as diversas posições, pois, creio que o assunto 
 não seja pacífico. 
 Saudações, 
 PJMS 
 
 Em Dom, 14 de out de 2018 06:33, Vanderlei Nemitz  
 escreveu:
> Bom dia!
> Na seguinte questão, que me foi apresentada por um aluno, a resposta 
> proposta é a alternativa C (1/2). Eu sempre pensei que apenas 
> considerávamos multiplicidades em equações polinomiais. Como 
> essa é uma equação exponencial, obtive a resposta B (-1/2). O 
> que é correto pensar?
> 
> O produto das raízes da equação 16.4^3x - 40.4^2x + 17.4^x - 2 = 
> 0 é igual a:
> A) 1
> B) - 0,5
> C) 0,5
> D) - 1
> E) 0
> 
> Muito obrigado!
> 
> -- 
> Esta mensagem foi verificada pelo sistema de antivírus e 
> acredita-se estar livre de perigo.
 
 -- 
 Esta mensagem foi verificada pelo sistema de antivírus e 
 acredita-se estar livre de perigo.
>>> 
>>> -- 
>>> Esta mensagem foi verificada pelo sistema de antivírus e 
>>> acredita-se estar livre de perigo.
>> 
>> -- 
>> Esta mensagem foi verificada pelo sistema de antivírus e 
>> acredita-se estar livre de perigo.
> 
> -- 
> Esta mensagem foi verificada pelo sistema de antivírus e 
> acredita-se estar livre de perigo.

-- 
Esta mensagem foi verificada pelo sistema de antiv�rus e
 acredita-se estar livre de perigo.



[obm-l] Re: [obm-l] Re: [obm-l] Re: [obm-l] Dúvida conceitual (equações)

2018-10-15 Por tôpico Vanderlei Nemitz
Claudio:
Eu ficaria com a mesma dúvida!
Pensaria em apenas uma raiz.

Qual é a soma das raízes da equação (cos x)^2 - cos x + 1/4 = 0 no
intervalo [0, 2pi]?

Em seg, 15 de out de 2018 07:00, Claudio Buffara 
escreveu:

> Qual a soma das raizes de (2^x - 8)^3 = 0?
> Se a equação acima fosse apresentada como:
> 2^(3x) - 24*2^(2x) + 192*2^x - 512 = 0,
> isso mudaria sua resposta?
>
> Enviado do meu iPhone
>
> Em 15 de out de 2018, à(s) 00:29, Vanderlei Nemitz 
> escreveu:
>
> Valeu, Pedro! Tomara que mais alguém emita sua opinião.
> Um abraço!
>
> Em dom, 14 de out de 2018 18:59, Pedro José 
> escreveu:
>
>> Boa noite!
>> Bom questionamento. Vou me posicionar na arquibancada.Â
>> Minha posição é controversa. Se quer se levar em conta a repetição
>> tem que se falar do produto das raízes, cada elevada a sua multiplicidade.
>> No caso de soma, cada raiz multiplicada pela multiplicidade.
>> Para esse exemplo, o conjunto solução é {1/2,-1} então o produto é
>> -1/2.
>> Em suma, não aceito n raízes iguais, mas sim uma raiz de multiplicidade
>> n.
>> Se quando queremos provar que algo é unico supomos a existência de dois
>> e provamos que são iguais. Creio que seja contraditório dois ou nais
>> iguais.
>> Mas vamos observar as diversas posições, pois, creio que o assunto não
>> seja pacífico.Â
>> Saudações,Â
>> PJMSÂ
>>
>> Em Dom, 14 de out de 2018 06:33, Vanderlei Nemitz 
>> escreveu:
>>
>>> Bom dia!
>>> Na seguinte questão, que me foi apresentada por um aluno, a resposta
>>> proposta é a alternativa C (1/2). Eu sempre pensei que apenas
>>> considerávamos multiplicidades em equações polinomiais. Como essa é uma
>>> equação exponencial, obtive a resposta B (-1/2). O que é correto pensar?
>>>
>>> O produto das raízes da equação 16.4^3x - 40.4^2x + 17.4^x - 2 = 0 é
>>> igual a:
>>> A) 1
>>> B) - 0,5
>>> C) 0,5
>>> D) - 1
>>> E) 0
>>>
>>> Muito obrigado!
>>>
>>> --
>>> Esta mensagem foi verificada pelo sistema de antivírus e
>>> acredita-se estar livre de perigo.
>>
>>
>> --
>> Esta mensagem foi verificada pelo sistema de antivírus e
>> acredita-se estar livre de perigo.
>
>
> --
> Esta mensagem foi verificada pelo sistema de antivírus e
> acredita-se estar livre de perigo.
>
>
> --
> Esta mensagem foi verificada pelo sistema de antivírus e
> acredita-se estar livre de perigo.
>

-- 
Esta mensagem foi verificada pelo sistema de antiv�rus e
 acredita-se estar livre de perigo.



Re: [obm-l] Re: [obm-l] Re: [obm-l] Dúvida conceitual (equações)

2018-10-15 Por tôpico Claudio Buffara
Qual a soma das raizes de (2^x - 8)^3 = 0?
Se a equação acima fosse apresentada como:
2^(3x) - 24*2^(2x) + 192*2^x - 512 = 0,
isso mudaria sua resposta?

Enviado do meu iPhone

Em 15 de out de 2018, à(s) 00:29, Vanderlei Nemitz  
escreveu:

> Valeu, Pedro! Tomara que mais alguém emita sua opinião.
> Um abraço!
> 
> Em dom, 14 de out de 2018 18:59, Pedro José  escreveu:
>> Boa noite!
>> Bom questionamento. Vou me posicionar na arquibancada. 
>> Minha posição é controversa. Se quer se levar em conta a repetição tem 
>> que se falar do produto das raízes, cada elevada a sua multiplicidade. No 
>> caso de soma, cada raiz multiplicada pela multiplicidade.
>> Para esse exemplo, o conjunto solução é {1/2,-1} então o produto é -1/2.
>> Em suma, não aceito n raízes iguais, mas sim uma raiz de multiplicidade n.
>> Se quando queremos provar que algo é unico supomos a existência de dois e 
>> provamos que são iguais. Creio que seja contraditório dois ou nais iguais.
>> Mas vamos observar as diversas posições, pois, creio que o assunto não 
>> seja pacífico. 
>> Saudações, 
>> PJMSÂ 
>> 
>> Em Dom, 14 de out de 2018 06:33, Vanderlei Nemitz  
>> escreveu:
>>> Bom dia!
>>> Na seguinte questão, que me foi apresentada por um aluno, a resposta 
>>> proposta é a alternativa C (1/2). Eu sempre pensei que apenas 
>>> considerávamos multiplicidades em equações polinomiais. Como essa é uma 
>>> equação exponencial, obtive a resposta B (-1/2). O que é correto pensar?
>>> 
>>> O produto das raízes da equação 16.4^3x - 40.4^2x + 17.4^x - 2 = 0 é 
>>> igual a:
>>> A) 1
>>> B) - 0,5
>>> C) 0,5
>>> D) - 1
>>> E) 0
>>> 
>>> Muito obrigado!
>>> 
>>> -- 
>>> Esta mensagem foi verificada pelo sistema de antivírus e 
>>> acredita-se estar livre de perigo.
>> 
>> -- 
>> Esta mensagem foi verificada pelo sistema de antivírus e 
>> acredita-se estar livre de perigo.
> 
> -- 
> Esta mensagem foi verificada pelo sistema de antivírus e 
> acredita-se estar livre de perigo.

-- 
Esta mensagem foi verificada pelo sistema de antiv�rus e
 acredita-se estar livre de perigo.



[obm-l] Re: [obm-l] Re: [obm-l] Dúvida conceitual (equações)

2018-10-14 Por tôpico Vanderlei Nemitz
Valeu, Pedro! Tomara que mais alguém emita sua opinião.
Um abraço!

Em dom, 14 de out de 2018 18:59, Pedro José  escreveu:

> Boa noite!
> Bom questionamento. Vou me posicionar na arquibancada.
> Minha posição é controversa. Se quer se levar em conta a repetição tem que
> se falar do produto das raízes, cada elevada a sua multiplicidade. No caso
> de soma, cada raiz multiplicada pela multiplicidade.
> Para esse exemplo, o conjunto solução é {1/2,-1} então o produto é -1/2.
> Em suma, não aceito n raízes iguais, mas sim uma raiz de multiplicidade n.
> Se quando queremos provar que algo é unico supomos a existência de dois e
> provamos que são iguais. Creio que seja contraditório dois ou nais iguais.
> Mas vamos observar as diversas posições, pois, creio que o assunto não
> seja pacífico.
> Saudações,
> PJMS
>
> Em Dom, 14 de out de 2018 06:33, Vanderlei Nemitz 
> escreveu:
>
>> Bom dia!
>> Na seguinte questão, que me foi apresentada por um aluno, a resposta
>> proposta é a alternativa C (1/2). Eu sempre pensei que apenas
>> considerávamos multiplicidades em equações polinomiais. Como essa é uma
>> equação exponencial, obtive a resposta B (-1/2). O que é correto pensar?
>>
>> O produto das raízes da equação 16.4^3x - 40.4^2x + 17.4^x - 2 = 0 é
>> igual a:
>> A) 1
>> B) - 0,5
>> C) 0,5
>> D) - 1
>> E) 0
>>
>> Muito obrigado!
>>
>> --
>> Esta mensagem foi verificada pelo sistema de antivírus e
>> acredita-se estar livre de perigo.
>
>
> --
> Esta mensagem foi verificada pelo sistema de antivírus e
> acredita-se estar livre de perigo.

-- 
Esta mensagem foi verificada pelo sistema de antiv�rus e
 acredita-se estar livre de perigo.



[obm-l] Re: [obm-l] Re: [obm-l] Dúvida

2018-08-23 Por tôpico Pedro José
Boa noite!
É fato.
Grato,
PJMS.

Em Qua, 22 de ago de 2018 23:00, Ralph Teixeira 
escreveu:

> Acho que nao... Ah, se eu entendi corretamente, (3,6,9) e (3,5,12) seria
> um contra-exemplo.
>
> Abraco, Ralph.
>
>
> On Wed, Aug 22, 2018 at 8:06 PM Pedro José  wrote:
>
>> Boa noite.
>>
>> Sejam duas sequências em ordem crescente com ai,bi >0 e k elementos ambas.
>> se:
>> (a1+a2+a3+...+ak)/(b1+b2+b3+...+bk)=a1a2a3a3...ak/(b1b2b3...bk) podemos
>> dizer que
>> ai=bi para 0>
>> Grato,
>> PJMS
>>
>> --
>> Esta mensagem foi verificada pelo sistema de antivírus e
>> acredita-se estar livre de perigo.
>
>
> --
> Esta mensagem foi verificada pelo sistema de antivírus e
> acredita-se estar livre de perigo.

-- 
Esta mensagem foi verificada pelo sistema de antiv�rus e
 acredita-se estar livre de perigo.



[obm-l] Re: [obm-l] Re: [obm-l] Dúvida

2018-05-31 Por tôpico Pedro José
Bom dia!
Corrigindo uma grande bobagem, confirme me alertado.
A ordem de 10 nos 11 é 2 e não 1. Mas como 2|6, não muda nada.

Saudações,
PJMS

Em Sex, 25 de mai de 2018 14:37, Pedro José  escreveu:

> Boa tarde!
> Creio ter conseguido.
> Criei um número com fatores congruentes a 1 mod 6, exceto o 5 e o11.
> Além disso a ordem de 10 mod desses fatores é sempre 6, exceto o 5 e o 11
> que será 1, melhor. Mas o 5 não tem problema.
> Então o objetivo é firmar um número da seguinte forma:
> ...AB...B...C concatenado com o número criado, mencionado
> anteriormente.
> O número criado foi:
> 84.259.175 = 5^2*7^2*11*13^2*37
> Então a soma dos algarismos desse número é 41 e dos quadrados de seus
> algarismos é 265.
> No número que pretendo formar o número de algarismos em bloco será
> múltiplo de 6.
> Então fica o sistema para apenas dois blocos:
> ax+by= (1001-41)/6=160
> a^2*x +b^2*y=(S2 -265)/6.
> Onde x e y é a quantidade de repetições de blocos de 6 algarismos e a e b
> são os algarismos e S2 é a soma dos quadrados de todos dígitos.
> Agora preciso criar S2 que feche com o problema. Tem que ser 1 mod 6, para
> quando subtrair 265, ser divisível por 6. Deve ser um divisor do número
> criado no início. 5^2*7^2*11*13^2*37.
> Seja S2=5005=5*7*11*13
> xa+yb=160
> xa^2+yb^2= (5005-265)/6=790.
> Como 6| 790 - 160, 1 e 3 formam uma boa escolha, mas infortunadamente, o
> número de blocos de 1 dá negativo.
> Então introduzi 9 blocos de 8 para acertar, já que há liberdade.
>
> Aí dão 9 blocos de 8, 25 blocos de 1 e 21 blocos de três, concatenação ao
> final, 84.259.175.
> É o número fica.
> 10^274*8*(10^54-1)/9+10^124*(10^150-1)/9+10^8*3*(10^126)/9+
> 5^2*7^2*11*13^2*17.
> Como
> 10^6 =1 mod p, com p=7 ou p=11 ou p= 13 e 5 |10, S2=5*7*11*13, S2 divide
> cada parcela e portanto o número.
> O número são 54 algarismos 8, seguidos de 150 algarismos 1,seguidos de126
> algarismos 3 seguidos de 84259175.
> Deve ter um jeito mais elegante de resolver.
> Saudações,
> PJMS
>
> Em Qui, 24 de mai de 2018 23:51, Pedro José 
> escreveu:
>
>> Boa noite!
>> Minha primeira tentativa foi tudo 1. Mas aí a soma dos quadrados também é
>> 1001=7*11*13.
>> As ordens de 10 mod desses fatores são 6, 1 e 6. Mas têm 1001 algarismos
>> e aí 6 ł 1001não serve.
>> Tentei outros arranjos com grupos de algarismos iguais, mas sem sucesso.
>> Mas o que não compreendo é porque não há a divulgação da resposta.
>> Saudações,
>> PJMS
>>
>> Em Qui, 24 de mai de 2018 21:09, Anderson Torres <
>> torres.anderson...@gmail.com> escreveu:
>>
>>> Em 23 de maio de 2018 21:41, Pedro José  escreveu:
>>> > Boa noite!
>>> > Há algum motivo para não disponibilizarem o gabarito da olimpiada de
>>> mayo?
>>> > Gostaria de ver a solução de um problema da XXII olimpiada:
>>> > Dizemos que um número inteiro positivo é qua-divi se é divisível
>>> pela
>>> > soma dos quadrados de seus dígitos, e além disso nenhum de seus
>>> dígitos
>>> > é igual a zero.
>>> > a) Encontre um número qua-divi tal que a soma de seus dígitos seja
>>> 24.
>>> > b) Encontre um número qua-divi tal que a soma de seus dígitos seja
>>> 1001.
>>>
>>> Só jogando uma ideia solta, eu tentaria calcular para casos como
>>> 111...11. A soma dos dígitos é N e o número é (10^N-1)/9
>>>
>>> Se isso não servir, talvez 111..12222 também possa ser
>>> útil.
>>>
>>> >
>>> > Grato.
>>> > Saudações,
>>> > PJMS
>>> >
>>> > --
>>> > Esta mensagem foi verificada pelo sistema de antivírus e
>>> > acredita-se estar livre de perigo.
>>>
>>> --
>>> Esta mensagem foi verificada pelo sistema de antivírus e
>>>  acredita-se estar livre de perigo.
>>>
>>>
>>> =
>>> Instru�ões para entrar na lista, sair da lista e usar a lista em
>>> http://www.mat.puc-rio.br/~obmlistas/obm-l.html
>>> =
>>>
>>

-- 
Esta mensagem foi verificada pelo sistema de antiv�rus e
 acredita-se estar livre de perigo.



[obm-l] Re: [obm-l] Re: [obm-l] Dúvida

2018-05-25 Por tôpico Pedro José
Boa tarde!
Creio ter conseguido.
Criei um número com fatores congruentes a 1 mod 6, exceto o 5 e o11.
Além disso a ordem de 10 mod desses fatores é sempre 6, exceto o 5 e o 11
que será 1, melhor. Mas o 5 não tem problema.
Então o objetivo é firmar um número da seguinte forma:
...AB...B...C concatenado com o número criado, mencionado
anteriormente.
O número criado foi:
84.259.175 = 5^2*7^2*11*13^2*37
Então a soma dos algarismos desse número é 41 e dos quadrados de seus
algarismos é 265.
No número que pretendo formar o número de algarismos em bloco será múltiplo
de 6.
Então fica o sistema para apenas dois blocos:
ax+by= (1001-41)/6=160
a^2*x +b^2*y=(S2 -265)/6.
Onde x e y é a quantidade de repetições de blocos de 6 algarismos e a e b
são os algarismos e S2 é a soma dos quadrados de todos dígitos.
Agora preciso criar S2 que feche com o problema. Tem que ser 1 mod 6, para
quando subtrair 265, ser divisível por 6. Deve ser um divisor do número
criado no início. 5^2*7^2*11*13^2*37.
Seja S2=5005=5*7*11*13
xa+yb=160
xa^2+yb^2= (5005-265)/6=790.
Como 6| 790 - 160, 1 e 3 formam uma boa escolha, mas infortunadamente, o
número de blocos de 1 dá negativo.
Então introduzi 9 blocos de 8 para acertar, já que há liberdade.

Aí dão 9 blocos de 8, 25 blocos de 1 e 21 blocos de três, concatenação ao
final, 84.259.175.
É o número fica.
10^274*8*(10^54-1)/9+10^124*(10^150-1)/9+10^8*3*(10^126)/9+
5^2*7^2*11*13^2*17.
Como
10^6 =1 mod p, com p=7 ou p=11 ou p= 13 e 5 |10, S2=5*7*11*13, S2 divide
cada parcela e portanto o número.
O número são 54 algarismos 8, seguidos de 150 algarismos 1,seguidos de126
algarismos 3 seguidos de 84259175.
Deve ter um jeito mais elegante de resolver.
Saudações,
PJMS

Em Qui, 24 de mai de 2018 23:51, Pedro José  escreveu:

> Boa noite!
> Minha primeira tentativa foi tudo 1. Mas aí a soma dos quadrados também é
> 1001=7*11*13.
> As ordens de 10 mod desses fatores são 6, 1 e 6. Mas têm 1001 algarismos e
> aí 6 ł 1001não serve.
> Tentei outros arranjos com grupos de algarismos iguais, mas sem sucesso.
> Mas o que não compreendo é porque não há a divulgação da resposta.
> Saudações,
> PJMS
>
> Em Qui, 24 de mai de 2018 21:09, Anderson Torres <
> torres.anderson...@gmail.com> escreveu:
>
>> Em 23 de maio de 2018 21:41, Pedro José  escreveu:
>> > Boa noite!
>> > Há algum motivo para não disponibilizarem o gabarito da olimpiada de
>> mayo?
>> > Gostaria de ver a solução de um problema da XXII olimpiada:
>> > Dizemos que um número inteiro positivo é qua-divi se é divisível
>> pela
>> > soma dos quadrados de seus dígitos, e além disso nenhum de seus
>> dígitos
>> > é igual a zero.
>> > a) Encontre um número qua-divi tal que a soma de seus dígitos seja 24.
>> > b) Encontre um número qua-divi tal que a soma de seus dígitos seja
>> 1001.
>>
>> Só jogando uma ideia solta, eu tentaria calcular para casos como
>> 111...11. A soma dos dígitos é N e o número é (10^N-1)/9
>>
>> Se isso não servir, talvez 111..12222 também possa ser
>> útil.
>>
>> >
>> > Grato.
>> > Saudações,
>> > PJMS
>> >
>> > --
>> > Esta mensagem foi verificada pelo sistema de antivírus e
>> > acredita-se estar livre de perigo.
>>
>> --
>> Esta mensagem foi verificada pelo sistema de antivírus e
>>  acredita-se estar livre de perigo.
>>
>>
>> =
>> Instru�ões para entrar na lista, sair da lista e usar a lista em
>> http://www.mat.puc-rio.br/~obmlistas/obm-l.html
>> =
>>
>

-- 
Esta mensagem foi verificada pelo sistema de antiv�rus e
 acredita-se estar livre de perigo.



[obm-l] Re: [obm-l] Re: [obm-l] Dúvida

2018-05-24 Por tôpico Pedro José
Boa noite!
Minha primeira tentativa foi tudo 1. Mas aí a soma dos quadrados também é
1001=7*11*13.
As ordens de 10 mod desses fatores são 6, 1 e 6. Mas têm 1001 algarismos e
aí 6 ł 1001não serve.
Tentei outros arranjos com grupos de algarismos iguais, mas sem sucesso.
Mas o que não compreendo é porque não há a divulgação da resposta.
Saudações,
PJMS

Em Qui, 24 de mai de 2018 21:09, Anderson Torres <
torres.anderson...@gmail.com> escreveu:

> Em 23 de maio de 2018 21:41, Pedro José  escreveu:
> > Boa noite!
> > Há algum motivo para não disponibilizarem o gabarito da olimpiada de
> mayo?
> > Gostaria de ver a solução de um problema da XXII olimpiada:
> > Dizemos que um número inteiro positivo é qua-divi se é divisível pela
> > soma dos quadrados de seus dígitos, e além disso nenhum de seus
> dígitos
> > é igual a zero.
> > a) Encontre um número qua-divi tal que a soma de seus dígitos seja 24.
> > b) Encontre um número qua-divi tal que a soma de seus dígitos seja
> 1001.
>
> Só jogando uma ideia solta, eu tentaria calcular para casos como
> 111...11. A soma dos dígitos é N e o número é (10^N-1)/9
>
> Se isso não servir, talvez 111..12222 também possa ser
> útil.
>
> >
> > Grato.
> > Saudações,
> > PJMS
> >
> > --
> > Esta mensagem foi verificada pelo sistema de antivírus e
> > acredita-se estar livre de perigo.
>
> --
> Esta mensagem foi verificada pelo sistema de antivírus e
>  acredita-se estar livre de perigo.
>
>
> =
> Instru�ões para entrar na lista, sair da lista e usar a lista em
> http://www.mat.puc-rio.br/~obmlistas/obm-l.html
> =
>

-- 
Esta mensagem foi verificada pelo sistema de antiv�rus e
 acredita-se estar livre de perigo.



[obm-l] Re: [obm-l] Re: [obm-l] Re: [obm-l] Re: [obm-l] Dúvida num Enunciado

2018-04-26 Por tôpico Pedro José
Boa tarde!
Bernardo,
Realmente eu falhei. Fiquei com a expressão |x+3| < 4 na cabeça. Até uso um
delta, e comento que não pode ser maior que 4.
Saudações,
PJMS

Em 25 de abr de 2018 22:33, "Jaare Oregim" 
escreveu:

>
>
> 2018-04-25 21:30 GMT-03:00 Bernardo Freitas Paulo da Costa <
> bernardo...@gmail.com>:
>
>> 2018-04-25 20:41 GMT-03:00 Claudio Buffara :
>> > O [...]
>> "Determine r > 0 tal que [ |x+3| < r => (A^2 - 10A + 9 > 0 para todo A
>> real) ]."
>>
>> Que continua com o "problema" de ter um "x" livre.  Daí, a proposição
>> entre colchetes tem um valor (verdadeiro/falso) que depende de x.
>>
>
> se o x tá livre *não* tem valor-verdade. Sentença aberta não tem
> valor-verdade.
>
> tb acho que a intenção é "Determine r > 0 tal que (para todo x real, |x+3|
> < r => x^2 - 10x + 9 > 0)."
>
> --
> Esta mensagem foi verificada pelo sistema de antivírus e
> acredita-se estar livre de perigo.

-- 
Esta mensagem foi verificada pelo sistema de antiv�rus e
 acredita-se estar livre de perigo.



[obm-l] Re: [obm-l] Re: [obm-l] Re: [obm-l] Dúvida num Enunciado

2018-04-25 Por tôpico Jaare Oregim
2018-04-25 21:30 GMT-03:00 Bernardo Freitas Paulo da Costa <
bernardo...@gmail.com>:

> 2018-04-25 20:41 GMT-03:00 Claudio Buffara :
> > O [...]
> "Determine r > 0 tal que [ |x+3| < r => (A^2 - 10A + 9 > 0 para todo A
> real) ]."
>
> Que continua com o "problema" de ter um "x" livre.  Daí, a proposição
> entre colchetes tem um valor (verdadeiro/falso) que depende de x.
>

se o x tá livre *não* tem valor-verdade. Sentença aberta não tem
valor-verdade.

tb acho que a intenção é "Determine r > 0 tal que (para todo x real, |x+3|
< r => x^2 - 10x + 9 > 0)."

-- 
Esta mensagem foi verificada pelo sistema de antiv�rus e
 acredita-se estar livre de perigo.



[obm-l] Re: [obm-l] Re: [obm-l] Re: [obm-l] Re: [obm-l] Dúvida num Enunciado

2018-04-25 Por tôpico Claudio Buffara
Verdade! Reparei agora que deve ser r > 0.
Então provavelmente o "para todo x real" não deveria estar lá.
Neste caso, vira um problema com mais cara de EM:

Achar todos os r > 0 tais que
SE  x pertence ao intervalo (-3-r , -3+r )
ENTÃO  x^2 - 10x + 9 > 0

x^2 - 10x + 9 > 0  sss  x pertence a (-inf,1) união (9,+inf).

Assim, observamos que se 0 < r <= 4, então (-3-r,-3+r) está contido em
(-inf,1).
Logo, se 0 < r <= 4 então a implicação acima é verdadeira.

[]s,
Claudio.




2018-04-25 21:47 GMT-03:00 Pedro José :

> Boa noite!
> Cláudio,
> o problema tem restrição r>0. Não dá para seguir nessa linha de r< 0.
> Saudações,
> PJMS
>
> Em 25 de abr de 2018 21:42, "Bernardo Freitas Paulo da Costa" <
> bernardo...@gmail.com> escreveu:
>
>> 2018-04-25 20:20 GMT-03:00 Pedro José :
>> > Boa tarde!
>> > Realmente o enunciado está mal feito.
>> >
>> > Se |x+3| < r, não pode ser para todo o Real. Na verdade é x pertence a
>> |R.
>> >
>> > x^2 -10x + 9 >0  ==> x pertence a A = (-oo, 1) U (9,oo)
>> >
>> > então temos que escolher r de modo que quando resolvamos |x + 3| < r,
>> tenha
>> > x num subconjunto de A
>> >
>> > x < -3 ==> x+3 < 0 ==> -x -3 < r ==> r > x+3 Se r > 4 vai ter 1=< x =<9
>> > atendendo |x +3| <4 + delta. Portanto x <4
>> > então |x+3| < 4, conferindo
>> > x > -3 ==> x+3 <4  ==> x<1, atende.
>> > se x<-3 atende por hipótese. Mas se quiser conferir. -x - 3 < 4 : -x <
>> 7: x
>> >>7, mas x <-3, não tem solução.
>> >
>> > x>=- 3 ==> x+3>=0 ==> x+3 < r. Se r >=4, existirá solução em [1,9].
>> >
>> > Portanto r pertence a (0,4)
>>
>> Só um detalhe: r = 4 também serve: se |x+3| < 4, temos -7 < x < 1, que
>> está contido em A.
>>
>> A minha forma preferida de resolver este exercício é gráfica:
>> desenhamos o conjunto A, depois tomamos P = -3, e traçamos um
>> intervalo simétrico em P de maior raio possível contido em A.  Dá r <=
>> 4 ou r < 4 (no desenho, é difícil decidir entre o estrito ou não) e
>> daí tem que pensar um pouco para detectar se r = 4 serve.
>>
>> Abraços,
>> --
>> Bernardo Freitas Paulo da Costa
>>
>> --
>> Esta mensagem foi verificada pelo sistema de antivírus e
>>  acredita-se estar livre de perigo.
>>
>>
>> =
>> Instru�ões para entrar na lista, sair da lista e usar a lista em
>> http://www.mat.puc-rio.br/~obmlistas/obm-l.html
>> =
>>
>
> --
> Esta mensagem foi verificada pelo sistema de antivírus e
> acredita-se estar livre de perigo.
>

-- 
Esta mensagem foi verificada pelo sistema de antiv�rus e
 acredita-se estar livre de perigo.



[obm-l] Re: [obm-l] Re: [obm-l] Re: [obm-l] Re: [obm-l] Dúvida num Enunciado

2018-04-25 Por tôpico Luiz Antonio Rodrigues
Olá, Bernardo!
Boa noite!
Vou tentar fazer a resolução graficamente...
Muito obrigado!
Um abraço!
Luiz

On Wed, Apr 25, 2018, 9:55 PM Pedro José  wrote:

> Boa noite!
> Cláudio,
> o problema tem restrição r>0. Não dá para seguir nessa linha de r< 0.
> Saudações,
> PJMS
>
> Em 25 de abr de 2018 21:42, "Bernardo Freitas Paulo da Costa" <
> bernardo...@gmail.com> escreveu:
>
>> 2018-04-25 20:20 GMT-03:00 Pedro José :
>> > Boa tarde!
>> > Realmente o enunciado está mal feito.
>> >
>> > Se |x+3| < r, não pode ser para todo o Real. Na verdade é x pertence a
>> |R.
>> >
>> > x^2 -10x + 9 >0  ==> x pertence a A = (-oo, 1) U (9,oo)
>> >
>> > então temos que escolher r de modo que quando resolvamos |x + 3| < r,
>> tenha
>> > x num subconjunto de A
>> >
>> > x < -3 ==> x+3 < 0 ==> -x -3 < r ==> r > x+3 Se r > 4 vai ter 1=< x =<9
>> > atendendo |x +3| <4 + delta. Portanto x <4
>> > então |x+3| < 4, conferindo
>> > x > -3 ==> x+3 <4  ==> x<1, atende.
>> > se x<-3 atende por hipótese. Mas se quiser conferir. -x - 3 < 4 : -x <
>> 7: x
>> >>7, mas x <-3, não tem solução.
>> >
>> > x>=- 3 ==> x+3>=0 ==> x+3 < r. Se r >=4, existirá solução em [1,9].
>> >
>> > Portanto r pertence a (0,4)
>>
>> Só um detalhe: r = 4 também serve: se |x+3| < 4, temos -7 < x < 1, que
>> está contido em A.
>>
>> A minha forma preferida de resolver este exercício é gráfica:
>> desenhamos o conjunto A, depois tomamos P = -3, e traçamos um
>> intervalo simétrico em P de maior raio possível contido em A.  Dá r <=
>> 4 ou r < 4 (no desenho, é difícil decidir entre o estrito ou não) e
>> daí tem que pensar um pouco para detectar se r = 4 serve.
>>
>> Abraços,
>> --
>> Bernardo Freitas Paulo da Costa
>>
>> --
>> Esta mensagem foi verificada pelo sistema de antivírus e
>>  acredita-se estar livre de perigo.
>>
>>
>> =
>> Instru�ões para entrar na lista, sair da lista e usar a lista em
>> http://www.mat.puc-rio.br/~obmlistas/obm-l.html
>> =
>>
>
> --
> Esta mensagem foi verificada pelo sistema de antivírus e
> acredita-se estar livre de perigo.

-- 
Esta mensagem foi verificada pelo sistema de antiv�rus e
 acredita-se estar livre de perigo.



[obm-l] Re: [obm-l] Re: [obm-l] Re: [obm-l] Dúvida num Enunciado

2018-04-25 Por tôpico Pedro José
Boa noite!
Cláudio,
o problema tem restrição r>0. Não dá para seguir nessa linha de r< 0.
Saudações,
PJMS

Em 25 de abr de 2018 21:42, "Bernardo Freitas Paulo da Costa" <
bernardo...@gmail.com> escreveu:

> 2018-04-25 20:20 GMT-03:00 Pedro José :
> > Boa tarde!
> > Realmente o enunciado está mal feito.
> >
> > Se |x+3| < r, não pode ser para todo o Real. Na verdade é x pertence a
> |R.
> >
> > x^2 -10x + 9 >0  ==> x pertence a A = (-oo, 1) U (9,oo)
> >
> > então temos que escolher r de modo que quando resolvamos |x + 3| < r,
> tenha
> > x num subconjunto de A
> >
> > x < -3 ==> x+3 < 0 ==> -x -3 < r ==> r > x+3 Se r > 4 vai ter 1=< x =<9
> > atendendo |x +3| <4 + delta. Portanto x <4
> > então |x+3| < 4, conferindo
> > x > -3 ==> x+3 <4  ==> x<1, atende.
> > se x<-3 atende por hipótese. Mas se quiser conferir. -x - 3 < 4 : -x <
> 7: x
> >>7, mas x <-3, não tem solução.
> >
> > x>=- 3 ==> x+3>=0 ==> x+3 < r. Se r >=4, existirá solução em [1,9].
> >
> > Portanto r pertence a (0,4)
>
> Só um detalhe: r = 4 também serve: se |x+3| < 4, temos -7 < x < 1, que
> está contido em A.
>
> A minha forma preferida de resolver este exercício é gráfica:
> desenhamos o conjunto A, depois tomamos P = -3, e traçamos um
> intervalo simétrico em P de maior raio possível contido em A.  Dá r <=
> 4 ou r < 4 (no desenho, é difícil decidir entre o estrito ou não) e
> daí tem que pensar um pouco para detectar se r = 4 serve.
>
> Abraços,
> --
> Bernardo Freitas Paulo da Costa
>
> --
> Esta mensagem foi verificada pelo sistema de antivírus e
>  acredita-se estar livre de perigo.
>
>
> =
> Instru�ões para entrar na lista, sair da lista e usar a lista em
> http://www.mat.puc-rio.br/~obmlistas/obm-l.html
> =
>

-- 
Esta mensagem foi verificada pelo sistema de antiv�rus e
 acredita-se estar livre de perigo.



[obm-l] Re: [obm-l] Re: [obm-l] Dúvida num Enunciado

2018-04-25 Por tôpico Bernardo Freitas Paulo da Costa
2018-04-25 20:20 GMT-03:00 Pedro José :
> Boa tarde!
> Realmente o enunciado está mal feito.
>
> Se |x+3| < r, não pode ser para todo o Real. Na verdade é x pertence a |R.
>
> x^2 -10x + 9 >0  ==> x pertence a A = (-oo, 1) U (9,oo)
>
> então temos que escolher r de modo que quando resolvamos |x + 3| < r, tenha
> x num subconjunto de A
>
> x < -3 ==> x+3 < 0 ==> -x -3 < r ==> r > x+3 Se r > 4 vai ter 1=< x =<9
> atendendo |x +3| <4 + delta. Portanto x <4
> então |x+3| < 4, conferindo
> x > -3 ==> x+3 <4  ==> x<1, atende.
> se x<-3 atende por hipótese. Mas se quiser conferir. -x - 3 < 4 : -x < 7: x
>>7, mas x <-3, não tem solução.
>
> x>=- 3 ==> x+3>=0 ==> x+3 < r. Se r >=4, existirá solução em [1,9].
>
> Portanto r pertence a (0,4)

Só um detalhe: r = 4 também serve: se |x+3| < 4, temos -7 < x < 1, que
está contido em A.

A minha forma preferida de resolver este exercício é gráfica:
desenhamos o conjunto A, depois tomamos P = -3, e traçamos um
intervalo simétrico em P de maior raio possível contido em A.  Dá r <=
4 ou r < 4 (no desenho, é difícil decidir entre o estrito ou não) e
daí tem que pensar um pouco para detectar se r = 4 serve.

Abraços,
-- 
Bernardo Freitas Paulo da Costa

-- 
Esta mensagem foi verificada pelo sistema de antiv�rus e
 acredita-se estar livre de perigo.


=
Instru��es para entrar na lista, sair da lista e usar a lista em
http://www.mat.puc-rio.br/~obmlistas/obm-l.html
=


[obm-l] Re: [obm-l] Re: [obm-l] Dúvida num Enunciado

2018-04-25 Por tôpico Bernardo Freitas Paulo da Costa
2018-04-25 20:41 GMT-03:00 Claudio Buffara :
> O consequente (x^2 - 10x + 9 > 0 para todo x real) é falso  (tome qualquer x
> no intervalo [1,9]).
>
> Logo, para a implicação ser verdadeira, o antecedente ( |x+3| < r ) deve ser
> falso, o que ocorre se e somente se r < 0.
>
> É mais ou menos a mesma coisa que (se 1 < 0, então 3+5 = 7), que é uma
> sentença verdadeira (Falso -> Falso é Verdadeiro).

Acho que há duas coisas.  Uma é a interpretação do enunciado.  Alguns
(eu me incluo, e o Pedro José também) vão ler como:

"Determine r > 0 tal que (para todo x real, |x+3| < r => x^2 - 10x + 9 > 0)."

A minha razão principal é porque o x aparece do lado esquerdo da
implicação, e portanto eu fico com a sensação que ele deveria também
estar quantificado.  Mas não é obrigatório.

Ao ler como você fez, a frase fica

"Determine r > 0 tal que [ |x+3| < r => (x^2 - 10x + 9 > 0 para todo x real) ]."

Nesta interpretação (que está mais próxima do texto original...), a
frase entre colchetes tem uma variável livre: o "x", que não está
quantificado.  Para enfatizar, como você mesmo separou o consequente,
ela não mudaria de valor se fosse

"Determine r > 0 tal que [ |x+3| < r => (A^2 - 10A + 9 > 0 para todo A real) ]."

Que continua com o "problema" de ter um "x" livre.  Daí, a proposição
entre colchetes tem um valor (verdadeiro/falso) que depende de x.
Assim, a frase completa "Determine r ..." também depende do valor de
x.  O problema fica bem diferente.  Primeiro, podemos simplificar o
enunciado para

"Determine r > 0 tal que [ |x+3| < r => FALSO ]."

Depois, pegando carona na sua solução, temos que ter o antecedente
falso, para que a afirmação entre colchetes seja verdadeira.  Ou seja:

"Determine r > 0 tal que [ |x+3| < r é FALSO ]."

Re-escrevendo, fica "Determine r > 0 tal que [ |x+3| >= r ].", o que
dá a solução: r <= |x+3|.  Repare que a solução está em função de x,
como esperado, já que o enunciado original também tinha um x livre.
Acho esta interpretação pouco plausível para um exercício, mas acho o
exercício de resolvê-la interessante ;-)

Abraços,
-- 
Bernardo Freitas Paulo da Costa

-- 
Esta mensagem foi verificada pelo sistema de antiv�rus e
 acredita-se estar livre de perigo.


=
Instru��es para entrar na lista, sair da lista e usar a lista em
http://www.mat.puc-rio.br/~obmlistas/obm-l.html
=


[obm-l] Re: [obm-l] Re: [obm-l] Dúvida num Enunciado

2018-04-25 Por tôpico Luiz Antonio Rodrigues
Olá, Claudio!
Boa noite!
Eu não havia percebido que o consequente é falso...
Preciso ficar mais atento!
Muito obrigado pela ajuda!
Um abraço!
Luiz

On Wed, Apr 25, 2018, 8:49 PM Claudio Buffara 
wrote:

> O consequente (x^2 - 10x + 9 > 0 para todo x real) é falso  (tome qualquer
> x no intervalo [1,9]).
>
> Logo, para a implicação ser verdadeira, o antecedente ( |x+3| < r ) deve
> ser falso, o que ocorre se e somente se r < 0.
>
> É mais ou menos a mesma coisa que (se 1 < 0, então 3+5 = 7), que é uma
> sentença verdadeira (Falso -> Falso é Verdadeiro).
>
> []s,
> Claudio.
>
> 2018-04-25 16:41 GMT-03:00 Luiz Antonio Rodrigues :
>
>> Olá, pessoal!
>> Boa tarde!
>> Estou tentando fazer o exercício abaixo, mas o problema é que eu não
>> entendi o enunciado...
>>
>> Determine para quais valores de r (r>0) a implicação é verdadeira:
>> |x+3| x^2 - 10x + 9 > 0 para todo x real.
>>
>> Agradeço qualquer ajuda!
>> Um abraço!
>> Luiz
>>
>> --
>> Esta mensagem foi verificada pelo sistema de antivírus e
>> acredita-se estar livre de perigo.
>
>
>
> --
> Esta mensagem foi verificada pelo sistema de antivírus e
> acredita-se estar livre de perigo.

-- 
Esta mensagem foi verificada pelo sistema de antiv�rus e
 acredita-se estar livre de perigo.



[obm-l] Re: [obm-l] Re: [obm-l] Dúvida num Enunciado

2018-04-25 Por tôpico Luiz Antonio Rodrigues
Olá, Pedro!
Boa noite!
O resultado é esse mesmo.
Agora eu entendi o que o problema pede.
Muito obrigado!
Um abraço!
Luiz

On Wed, Apr 25, 2018, 8:29 PM Pedro José  wrote:

> Boa tarde!
> Realmente o enunciado está mal feito.
>
> Se |x+3| < r, não pode ser para todo o Real. Na verdade é x pertence a |R.
>
> x^2 -10x + 9 >0  ==> x pertence a A = (-oo, 1) U (9,oo)
>
> então temos que escolher r de modo que quando resolvamos |x + 3| < r,
> tenha x num subconjunto de A
>
> x < -3 ==> x+3 < 0 ==> -x -3 < r ==> r > x+3 Se r > 4 vai ter 1=< x =<9
> atendendo |x +3| <4 + delta. Portanto x <4
> então |x+3| < 4, conferindo
> x > -3 ==> x+3 <4  ==> x<1, atende.
> se x<-3 atende por hipótese. Mas se quiser conferir. -x - 3 < 4 : -x < 7:
> x >7, mas x <-3, não tem solução.
>
> x>=- 3 ==> x+3>=0 ==> x+3 < r. Se r >=4, existirá solução em [1,9].
>
> Portanto r pertence a (0,4)
>
> creio que seja isso.
>
> Saudações,
> PJMS.
>
>
>
> Em 25 de abril de 2018 16:41, Luiz Antonio Rodrigues <
> rodrigue...@gmail.com> escreveu:
>
>> Olá, pessoal!
>> Boa tarde!
>> Estou tentando fazer o exercício abaixo, mas o problema é que eu não
>> entendi o enunciado...
>>
>> Determine para quais valores de r (r>0) a implicação é verdadeira:
>> |x+3| x^2 - 10x + 9 > 0 para todo x real.
>>
>> Agradeço qualquer ajuda!
>> Um abraço!
>> Luiz
>>
>> --
>> Esta mensagem foi verificada pelo sistema de antivírus e
>> acredita-se estar livre de perigo.
>
>
>
> --
> Esta mensagem foi verificada pelo sistema de antivírus e
> acredita-se estar livre de perigo.

-- 
Esta mensagem foi verificada pelo sistema de antiv�rus e
 acredita-se estar livre de perigo.



[obm-l] Re: [obm-l] Re: [obm-l] Re: [obm-l] Re: [obm-l] Dúvida em uma solução (conjunto denso)

2017-07-10 Por tôpico Pedro Soares
Sim, é uma prova por absurdo.

''...o autor parte de uma hipótese contrária ao resultado pra chegar num
absurdo...''

2017-07-11 1:03 GMT-03:00 Bernardo Freitas Paulo da Costa <
bernardo...@gmail.com>:

> 2017-07-10 18:56 GMT+03:00 Antonio Carlos :
> > Entendi. Muito obrigado, Pedro!
>
> Tem um problema muito sério, que os logs são diferentes...
>
> log_2 3 = log(3)/log(2) = 1.5849625007211563
> log_3 6 = log(6)/log(3) = 1.6309297535714573
>
> Mas o problema está, provavelmente, na primeira hipótese (que ela
> também é falsa).  A demonstração por densidade está certa, e talvez
> seja no meio de um raciocínio por absurdo, mas sei lá...
> --
> Bernardo Freitas Paulo da Costa
>
> --
> Esta mensagem foi verificada pelo sistema de antivírus e
>  acredita-se estar livre de perigo.
>
>
> =
> Instru�ões para entrar na lista, sair da lista e usar a lista em
> http://www.mat.puc-rio.br/~obmlistas/obm-l.html
> =
>

-- 
Esta mensagem foi verificada pelo sistema de antiv�rus e
 acredita-se estar livre de perigo.



[obm-l] Re: [obm-l] Re: [obm-l] Re: [obm-l] Dúvida em uma solução (conjunto denso)

2017-07-10 Por tôpico Bernardo Freitas Paulo da Costa
2017-07-10 18:56 GMT+03:00 Antonio Carlos :
> Entendi. Muito obrigado, Pedro!

Tem um problema muito sério, que os logs são diferentes...

log_2 3 = log(3)/log(2) = 1.5849625007211563
log_3 6 = log(6)/log(3) = 1.6309297535714573

Mas o problema está, provavelmente, na primeira hipótese (que ela
também é falsa).  A demonstração por densidade está certa, e talvez
seja no meio de um raciocínio por absurdo, mas sei lá...
-- 
Bernardo Freitas Paulo da Costa

-- 
Esta mensagem foi verificada pelo sistema de antiv�rus e
 acredita-se estar livre de perigo.


=
Instru��es para entrar na lista, sair da lista e usar a lista em
http://www.mat.puc-rio.br/~obmlistas/obm-l.html
=


[obm-l] Re: [obm-l] Re: [obm-l] Dúvida em uma solução (conjunto denso)

2017-07-10 Por tôpico Antonio Carlos
Entendi. Muito obrigado, Pedro!

On Jul 10, 2017 06:26, "Pedro Soares"  wrote:

> u/v < log_2 3 => u/v < log_3 6 , logo ou log_2 3 é menor ou igual a log_3
> 6 ou o intervalo [log_3 6, log_2 3] não possui nenhum número racional.
>
> u/v < log_3 6 => u/v < log_2 3 , logo ou log_3 6 é menor ou igual a log_2
> 3 ou o intervalo [log_2 3, log_3 6] não possui nenhum número racional.
>
> Como os racionais são densos na reta temos que log_2 3 >= log_3 6 e log_3
> 6 >= log_2 3 ==> log_2 3 = log_3 6, o que é falso. Ou isso ou os intervalos
> seriam degenerados o que também implicaria em log_2 3 = log_3 6.
> Assim, vc chega em um absurdo.
>
> Sacou?
>
>
>
> 2017-07-09 17:03 GMT-03:00 Antonio Carlos :
>
>> Oi pessoal,
>>
>> Estava lendo uma resolução de uma questão, e em uma passagem se chega à
>> seguinte implicação (u e v são naturais, log_a x é o logaritmo de x na base
>> a):
>>
>>  u/v < log_2 3 se e somente se u/v < log_3 6, e como os racionais são
>> densos, temos que a equivalência acima implica que log_2 3 = log_3 6.
>>
>> Tudo bem com a equivalência, o autor parte de uma hipótese contrária ao
>> resultado pra chegar num absurdo, o que não entendi foi a implicação usando
>> que Q é denso. Eu já fiz um curso de análise e tenho alguma noção do que é
>> um conjunto ser denso. Se alguém puder me ajudar a entender a passagem eu
>> agradeço.
>>
>> Att,
>> Antonio
>>
>>
>>
>> --
>> Esta mensagem foi verificada pelo sistema de antivírus e
>> acredita-se estar livre de perigo.
>
>
>
> --
> Esta mensagem foi verificada pelo sistema de antivírus e
> acredita-se estar livre de perigo.

-- 
Esta mensagem foi verificada pelo sistema de antiv�rus e
 acredita-se estar livre de perigo.



[obm-l] Re: [obm-l] Re: [obm-l] Re: [obm-l] Re: [obm-l] Dúvida sobre a Obm U

2016-07-26 Por tôpico Carlos Gomes
Um bom livro é Razvan Gelca, Titu Andreescu-Putnam and Beyond (2007)

Cgomes.

Em 26 de julho de 2016 08:57, Otávio Araújo 
escreveu:

> Não, onde posso conseguir? e do que ela trata?
>
> Em 25 de julho de 2016 11:32, Carlos Victor 
> escreveu:
>
>>
>>
>>
>> Oi Otávio,
>>
>> Você já viu a Revista Matemática Universitária da SBM ?
>>
>> Em 25/07/2016 10:09, Otávio Araújo escreveu:
>>
>>
>>
>> Pois é, se algum professor com experiência em olimpíadas, como o Nicolau
>> por exemplo, respondesse minha pergunta seria de grande ajuda
>>
>> Em 24 de jul de 2016, às 23:25, Israel Meireles Chrisostomo <
>> israelmchrisost...@gmail.com> escreveu:
>>
>> Boa pergunta, eu também tenho interesse em participar da OBM U e
>> gostaria de umas dicas
>>
>> Em 16 de julho de 2016 13:29, Otávio Araújo 
>> escreveu:
>>
>>> Galera, gostaria que vocês me dessem dicas de o que estudar, como
>>> estudar e por quais livros e materiais estudar para a prova da Obm nível
>>> universitário...
>>> Estou muito interessado em participar, mas fico meio confuso por onde
>>> estudar...
>>> Por favor me ajudem
>>> --
>>> Esta mensagem foi verificada pelo sistema de antivírus e
>>> Â acredita-se estar livre de perigo.
>>>
>>>
>>> =
>>> Instruções para entrar na lista, sair da lista e usar a lista em
>>> http://www.mat.puc-rio.br/~obmlistas/obm-l.html
>>> =
>>>
>>
>> --
>> Esta mensagem foi verificada pelo sistema de antivírus e
>> acredita-se estar livre de perigo.
>>
>>
>> --
>> Esta mensagem foi verificada pelo sistema de antivrus e
>> acredita-se estar livre de perigo.
>>
>>
>> --
>> Esta mensagem foi verificada pelo sistema de antivírus e
>> acredita-se estar livre de perigo.
>>
>
>
> --
> Esta mensagem foi verificada pelo sistema de antivírus e
> acredita-se estar livre de perigo.
>

-- 
Esta mensagem foi verificada pelo sistema de antiv�rus e
 acredita-se estar livre de perigo.



[obm-l] Re: [obm-l] Re: [obm-l] Re: [obm-l] Dúvida sobre a Obm U

2016-07-26 Por tôpico Otávio Araújo
Não, onde posso conseguir? e do que ela trata?

Em 25 de julho de 2016 11:32, Carlos Victor 
escreveu:

>
>
>
> Oi Otávio,
>
> Você já viu a Revista Matemática Universitária da SBM ?
>
> Em 25/07/2016 10:09, Otávio Araújo escreveu:
>
>
>
> Pois é, se algum professor com experiência em olimpíadas, como o Nicolau
> por exemplo, respondesse minha pergunta seria de grande ajuda
>
> Em 24 de jul de 2016, às 23:25, Israel Meireles Chrisostomo <
> israelmchrisost...@gmail.com> escreveu:
>
> Boa pergunta, eu também tenho interesse em participar da OBM U e gostaria
> de umas dicas
>
> Em 16 de julho de 2016 13:29, Otávio Araújo 
> escreveu:
>
>> Galera, gostaria que vocês me dessem dicas de o que estudar, como
>> estudar e por quais livros e materiais estudar para a prova da Obm nível
>> universitário...
>> Estou muito interessado em participar, mas fico meio confuso por onde
>> estudar...
>> Por favor me ajudem
>> --
>> Esta mensagem foi verificada pelo sistema de antivírus e
>> Â acredita-se estar livre de perigo.
>>
>>
>> =
>> Instruções para entrar na lista, sair da lista e usar a lista em
>> http://www.mat.puc-rio.br/~obmlistas/obm-l.html
>> =
>>
>
> --
> Esta mensagem foi verificada pelo sistema de antivírus e
> acredita-se estar livre de perigo.
>
>
> --
> Esta mensagem foi verificada pelo sistema de antivrus e
> acredita-se estar livre de perigo.
>
>
> --
> Esta mensagem foi verificada pelo sistema de antivírus e
> acredita-se estar livre de perigo.
>

-- 
Esta mensagem foi verificada pelo sistema de antiv�rus e
 acredita-se estar livre de perigo.



[obm-l] Re: [obm-l] Re: [obm-l] Dúvida sobre a Obm U

2016-07-25 Por tôpico Carlos Victor
 

Oi Otávio, 

Você já viu a Revista Matemática Universitária da SBM ? 

Em 25/07/2016 10:09, Otávio Araújo escreveu: 

> Pois é, se algum professor com experiência em olimpíadas, como o Nicolau por 
> exemplo, respondesse minha pergunta seria de grande ajuda 
> 
> Em 24 de jul de 2016, às 23:25, Israel Meireles Chrisostomo 
>  escreveu:
> 
> Boa pergunta, eu tambÃ(c)m tenho interesse em participar da OBM U e gostaria 
> de umas dicas 
> 
> Em 16 de julho de 2016 13:29, Otávio Araújo  
> escreveu:
> Galera, gostaria que vocês me dessem dicas de o que estudar, como estudar 
> e por quais livros e materiais estudar para a prova da Obm nível 
> universitário...
> Estou muito interessado em participar, mas fico meio confuso por onde 
> estudar...
> Por favor me ajudem
> --
> Esta mensagem foi verificada pelo sistema de antivírus e
> Â acredita-se estar livre de perigo.
> 
> =
> Instruções para entrar na lista, sair da lista e usar a lista em
> http://www.mat.puc-rio.br/~obmlistas/obm-l.html [1]
> =
> 
> -- 
> Esta mensagem foi verificada pelo sistema de antivírus e 
> acredita-se estar livre de perigo.

-- 
Esta mensagem foi verificada pelo sistema de antivrus e 
acredita-se estar livre de perigo. 

Links:
--
[1] http://www.mat.puc-rio.br/~obmlistas/obm-l.html

-- 
Esta mensagem foi verificada pelo sistema de antiv�rus e
 acredita-se estar livre de perigo.



[obm-l] Re: [obm-l] Re: [obm-l] Re: [obm-l] Re: [obm-l] Dúvida sobre a Obm U

2016-07-25 Por tôpico Tiago Sandino
Égua ma, sou mais ou menos da UFC, de qualquer forma, começar matemática
UFC prox ano. Fiz olimpíada um tempo, imergi totalmente nisso. Fiz e
trabalhei com engenharia elétrica uns anos, larguei o curso no final pq o
negócio na engenharia era próprio e precisava de tempo. Atualmente tô dando
aula de turma ITA IME e olimpíada de mat no Colégio Militar. Não sei se eu
vou poder fazer a prova da OBMU, mas estudarei. E a olimpíada universitária
é uma continuação da não universitária, daí meu interesse, já que sou
professor disso.
Precisando de dica, ta aí meu contato. 85 9 99134896. Se tem uma coisa que
eu sei sobre olimpíada é que o cara tem que tá no meio, tem que falar sobre
isso, tem que conhecer pessoas do meio etc. Só assim vc evolui. Só assim vc
passa de ser um cara que consegue aplicar fórmula a ser um cara que "cria"
matemática todo dia, que na minha concepção é a maior realização pessoal na
olimpíada.


Em 25 de julho de 2016 17:19, Otávio Araújo 
escreveu:

>
>
> Égua Tiago, eu também sou do Ceará mas meu celular atualmente não tem
> chip Mas tu é da UFC Tiago? E ainda estou esperando algum professor com
> experiência em olimpíadas de matemática responder a minha pergunta
>
> Em 25 de jul de 2016, às 13:38, Tiago Sandino 
> escreveu:
>
> Oi pessoal.
> Tem diversos livros de olimpíadas para graduandos (undergrads) ou com
> capítulos de temas exclusivamente (até onde eu saiba) universitários.
> Grátis na net, que eu saiba, tem muita coisa no AOPS. Dois links aqui:
> 1) *Fórum*: https://www.artofproblemsolving.com/community/c7_college_math
> 2) *Fórum por Competições*:
> https://www.artofproblemsolving.com/community/c15_undergraduate_contests
>
> Sou do Ceará, tava meio afastado da Matemática, mas fiz as pazes com ela
> recentemente. Se alguém quiser formar um grupo de estudo pelo Whatsapp...
> segue meu número: 85 9 9913 4896.
>
> Att.
> Tiago Sandino
>
> Em 25 de julho de 2016 10:20, Raul Alves  escreveu:
>
>> Também tenho interesse na OBMU, e a 1ª fase tá chegando.
>> Se algum professor puder organizar algum material de apoio, seria de
>> grande ajuda
>>
>> Em 25 de julho de 2016 10:09, Otávio Araújo 
>> escreveu:
>>
>>>
>>>
>>> Pois é, se algum professor com experiência em olimpíadas, como o
>>> Nicolau por exemplo, respondesse minha pergunta seria de grande ajuda
>>>
>>> Em 24 de jul de 2016, Ã s 23:25, Israel Meireles Chrisostomo <
>>> israelmchrisost...@gmail.com> escreveu:
>>>
>>> Boa pergunta, eu também tenho interesse em participar da OBM U e
>>> gostaria de umas dicas
>>>
>>> Em 16 de julho de 2016 13:29, Otávio Araújo <
>>> otavio17.ara...@gmail.com> escreveu:
>>>
 Galera, gostaria que vocês me dessem dicas de o que estudar,
 como estudar e por quais livros e materiais estudar para a prova da Obm
 nível universitário...
 Estou muito interessado em participar, mas fico meio confuso por onde
 estudar...
 Por favor me ajudem
 --
 Esta mensagem foi verificada pelo sistema de antivírus e
  acredita-se estar livre de perigo.



 =
 Instruções para entrar na lista, sair da lista e usar a lista em
 http://www.mat.puc-rio.br/~obmlistas/obm-l.html

 =

>>>
>>>
>>> --
>>> Esta mensagem foi verificada pelo sistema de antivírus e
>>> acredita-se estar livre de perigo.
>>>
>>>
>>> --
>>> Esta mensagem foi verificada pelo sistema de antivírus e
>>> acredita-se estar livre de perigo.
>>>
>>
>>
>>
>> --
>> *Raul Lima Alves*
>>
>> *Estagiário na Aton Engenharia*
>> *Estudante de Engenharia de Computação - UFBA*
>> *Telefone: (71) 9103-0878*
>> *Facebook:Â *https://www.facebook.com/raul.alves.161
>> *LinkedIn*:Â https://br.linkedin.com/in/raul-alves-8b090228
>> 
>>
>>
>>
>> --
>> Esta mensagem foi verificada pelo sistema de antivírus e
>> acredita-se estar livre de perigo.
>>
>
>
> --
> Esta mensagem foi verificada pelo sistema de antivírus e
> acredita-se estar livre de perigo.
>
>
> --
> Esta mensagem foi verificada pelo sistema de antivírus e
> acredita-se estar livre de perigo.
>

-- 
Esta mensagem foi verificada pelo sistema de antiv�rus e
 acredita-se estar livre de perigo.



Re: [obm-l] Re: [obm-l] Re: [obm-l] Re: [obm-l] Dúvida sobre a Obm U

2016-07-25 Por tôpico Otávio Araújo


Égua Tiago, eu também sou do Ceará mas meu celular atualmente não tem chip 
Mas tu é da UFC Tiago? E ainda estou esperando algum professor com experiência 
em olimpíadas de matemática responder a minha pergunta 

> Em 25 de jul de 2016, às 13:38, Tiago Sandino  
> escreveu:
> 
> Oi pessoal.
> Tem diversos livros de olimpíadas para graduandos (undergrads) ou com 
> capítulos de temas exclusivamente (até onde eu saiba) universitários. 
> Grátis na net, que eu saiba, tem muita coisa no AOPS. Dois links aqui:
> 1) Fórum: https://www.artofproblemsolving.com/community/c7_college_math
> 2) Fórum por Competições: 
> https://www.artofproblemsolving.com/community/c15_undergraduate_contests
> 
> Sou do Ceará, tava meio afastado da Matemática, mas fiz as pazes com ela 
> recentemente. Se alguém quiser formar um grupo de estudo pelo Whatsapp... 
> segue meu número: 85 9 9913 4896.
> 
> Att.
> Tiago Sandino
> 
> Em 25 de julho de 2016 10:20, Raul Alves  escreveu:
>> Também tenho interesse na OBMU, e a 1ª fase tá chegando.
>> Se algum professor puder organizar algum material de apoio, seria de grande 
>> ajuda
>> 
>> Em 25 de julho de 2016 10:09, Otávio Araújo  
>> escreveu:
>>> 
>>> 
>>> Pois é, se algum professor com experiência em olimpíadas, como o Nicolau 
>>> por exemplo, respondesse minha pergunta seria de grande ajuda
>>> 
>>> Em 24 de jul de 2016, Ã s 23:25, Israel Meireles Chrisostomo 
>>>  escreveu:
>>> 
 Boa pergunta, eu também tenho interesse em participar da OBM U e 
 gostaria de umas dicas
 
 Em 16 de julho de 2016 13:29, Otávio Araújo 
  escreveu:
> Galera, gostaria que vocês me dessem dicas de o que estudar, como 
> estudar e por quais livros e materiais estudar para a prova da Obm 
> nível universitário...
> Estou muito interessado em participar, mas fico meio confuso por onde 
> estudar...
> Por favor me ajudem
> --
> Esta mensagem foi verificada pelo sistema de antivírus e
>  acredita-se estar livre de perigo.
> 
> 
> =
> Instruções para entrar na lista, sair da lista e usar a lista em
> http://www.mat.puc-rio.br/~obmlistas/obm-l.html
> =
 
 
 -- 
 Esta mensagem foi verificada pelo sistema de antivírus e 
 acredita-se estar livre de perigo.
>>> 
>>> -- 
>>> Esta mensagem foi verificada pelo sistema de antivírus e 
>>> acredita-se estar livre de perigo.
>> 
>> 
>> 
>> -- 
>> Raul Lima Alves
>> 
>> Estagiário na Aton Engenharia
>> Estudante de Engenharia de Computação - UFBA
>> Telefone: (71) 9103-0878
>> Facebook:Â https://www.facebook.com/raul.alves.161
>> LinkedIn:Â https://br.linkedin.com/in/raul-alves-8b090228
>> 
>> 
>> 
>> -- 
>> Esta mensagem foi verificada pelo sistema de antivírus e 
>> acredita-se estar livre de perigo.
> 
> 
> -- 
> Esta mensagem foi verificada pelo sistema de antivírus e 
> acredita-se estar livre de perigo.

-- 
Esta mensagem foi verificada pelo sistema de antiv�rus e
 acredita-se estar livre de perigo.



[obm-l] Re: [obm-l] Re: [obm-l] Re: [obm-l] Dúvida sobre a Obm U

2016-07-25 Por tôpico Tiago Sandino
Oi pessoal.
Tem diversos livros de olimpíadas para graduandos (undergrads) ou com
capítulos de temas exclusivamente (até onde eu saiba) universitários.
Grátis na net, que eu saiba, tem muita coisa no AOPS. Dois links aqui:
1) *Fórum*: https://www.artofproblemsolving.com/community/c7_college_math
2) *Fórum por Competições*:
https://www.artofproblemsolving.com/community/c15_undergraduate_contests

Sou do Ceará, tava meio afastado da Matemática, mas fiz as pazes com ela
recentemente. Se alguém quiser formar um grupo de estudo pelo Whatsapp...
segue meu número: 85 9 9913 4896.

Att.
Tiago Sandino

Em 25 de julho de 2016 10:20, Raul Alves  escreveu:

> Também tenho interesse na OBMU, e a 1ª fase tá chegando.
> Se algum professor puder organizar algum material de apoio, seria de
> grande ajuda
>
> Em 25 de julho de 2016 10:09, Otávio Araújo 
> escreveu:
>
>>
>>
>> Pois é, se algum professor com experiência em olimpíadas, como o Nicolau
>> por exemplo, respondesse minha pergunta seria de grande ajuda
>>
>> Em 24 de jul de 2016, às 23:25, Israel Meireles Chrisostomo <
>> israelmchrisost...@gmail.com> escreveu:
>>
>> Boa pergunta, eu também tenho interesse em participar da OBM U e
>> gostaria de umas dicas
>>
>> Em 16 de julho de 2016 13:29, Otávio Araújo 
>> escreveu:
>>
>>> Galera, gostaria que vocês me dessem dicas de o que estudar, como
>>> estudar e por quais livros e materiais estudar para a prova da Obm nível
>>> universitário...
>>> Estou muito interessado em participar, mas fico meio confuso por onde
>>> estudar...
>>> Por favor me ajudem
>>> --
>>> Esta mensagem foi verificada pelo sistema de antivírus e
>>> Â acredita-se estar livre de perigo.
>>>
>>>
>>> =
>>> Instruções para entrar na lista, sair da lista e usar a lista em
>>> http://www.mat.puc-rio.br/~obmlistas/obm-l.html
>>> =
>>>
>>
>>
>> --
>> Esta mensagem foi verificada pelo sistema de antivírus e
>> acredita-se estar livre de perigo.
>>
>>
>> --
>> Esta mensagem foi verificada pelo sistema de antivírus e
>> acredita-se estar livre de perigo.
>>
>
>
>
> --
> *Raul Lima Alves*
>
> *Estagiário na Aton Engenharia*
> *Estudante de Engenharia de Computação - UFBA*
> *Telefone: (71) 9103-0878*
> *Facebook: *https://www.facebook.com/raul.alves.161
> *LinkedIn*: https://br.linkedin.com/in/raul-alves-8b090228
> 
>
>
>
> --
> Esta mensagem foi verificada pelo sistema de antivírus e
> acredita-se estar livre de perigo.
>

-- 
Esta mensagem foi verificada pelo sistema de antiv�rus e
 acredita-se estar livre de perigo.



[obm-l] Re: [obm-l] Re: [obm-l] Dúvida sobre a Obm U

2016-07-25 Por tôpico Raul Alves
Também tenho interesse na OBMU, e a 1ª fase tá chegando.
Se algum professor puder organizar algum material de apoio, seria de grande
ajuda

Em 25 de julho de 2016 10:09, Otávio Araújo 
escreveu:

>
>
> Pois é, se algum professor com experiência em olimpíadas, como o Nicolau
> por exemplo, respondesse minha pergunta seria de grande ajuda
>
> Em 24 de jul de 2016, às 23:25, Israel Meireles Chrisostomo <
> israelmchrisost...@gmail.com> escreveu:
>
> Boa pergunta, eu também tenho interesse em participar da OBM U e gostaria
> de umas dicas
>
> Em 16 de julho de 2016 13:29, Otávio Araújo 
> escreveu:
>
>> Galera, gostaria que vocês me dessem dicas de o que estudar, como
>> estudar e por quais livros e materiais estudar para a prova da Obm nível
>> universitário...
>> Estou muito interessado em participar, mas fico meio confuso por onde
>> estudar...
>> Por favor me ajudem
>> --
>> Esta mensagem foi verificada pelo sistema de antivírus e
>> Â acredita-se estar livre de perigo.
>>
>>
>> =
>> Instruções para entrar na lista, sair da lista e usar a lista em
>> http://www.mat.puc-rio.br/~obmlistas/obm-l.html
>> =
>>
>
>
> --
> Esta mensagem foi verificada pelo sistema de antivírus e
> acredita-se estar livre de perigo.
>
>
> --
> Esta mensagem foi verificada pelo sistema de antivírus e
> acredita-se estar livre de perigo.
>



-- 
*Raul Lima Alves*

*Estagiário na Aton Engenharia*
*Estudante de Engenharia de Computação - UFBA*
*Telefone: (71) 9103-0878*
*Facebook: *https://www.facebook.com/raul.alves.161
*LinkedIn*: https://br.linkedin.com/in/raul-alves-8b090228


-- 
Esta mensagem foi verificada pelo sistema de antiv�rus e
 acredita-se estar livre de perigo.



[obm-l] Re: [obm-l] Re: [obm-l] Dúvida em Geometria Espacial

2016-06-08 Por tôpico Daniel Rocha
Muito Obrigado, Grande Carlos !!!

Em 8 de junho de 2016 20:13, Carlos Gomes  escreveu:

> Vc pode fazer assim:
>
> área total = 60 ==> 2.pi.r^2+2.pi.r.h=60  ==>h=(60-2.pi.r^2)/(2.pi.r)   (*)
> Por outro lado o volume é
>
> V=pi.r^2.h
>
> substituindo a expressão (*) do h , segue que
>
> V=60r-2.pi.r^3
>
> Fazendo dV/dr=0 (derivada igual a zero para achar os pontos críticos),
> segue que
>
> 0=60-6.pir^2  ==> r=sqtr(10/pi) (é ponto de máximo, pois
> d^2V/dr^2(sqtr(10/pi))<0
>
> substituindo com esse valor de r na expressão (*), segue que
> h=2.sqrt(10/pi).
>
> O que revela que r/h=sqtr(10/pi) / 2.sqtr(10/pi)=1/2.
>
> Cgomes.
>
> Em 8 de junho de 2016 19:17, Daniel Rocha 
> escreveu:
>
>> Alguém poderia, por favor, solucionar a questão abaixo:
>>
>> Uma lata de forma cilíndrica, com tampa, deve ser construída com 60 cm^2
>> de folha de alumínio. Se r é o raio da base e h é a altura da lata que
>> proporcionam o volume máximo, então o valor de r/h é:
>>
>> GABARITO: 1/2
>>
>> --
>> Esta mensagem foi verificada pelo sistema de antivírus e
>> acredita-se estar livre de perigo.
>
>
>
> --
> Esta mensagem foi verificada pelo sistema de antivírus e
> acredita-se estar livre de perigo.

-- 
Esta mensagem foi verificada pelo sistema de antiv�rus e
 acredita-se estar livre de perigo.



[obm-l] Re: [obm-l] Re: [obm-l] Dúvida em Logaritmos

2016-06-07 Por tôpico Daniel Rocha
Muito Obrigado (mais uma vez), Carlos !!!

Em 6 de junho de 2016 22:02, Carlos Gomes  escreveu:

> log[(sqrt 2)^(x-2)] = x ==>
> (x-2)log(sqrt 2) = x ==>
> x=2log(sqrt2)/(log(sqrt2)-1).
>
> Cgomes.
>
> Em 6 de junho de 2016 19:23, Daniel Rocha 
> escreveu:
>
>> Alguém poderia, por favor, solucionar o problema abaixo:
>>
>> Ache a solução real da equação:
>> log[(sqrt 2)^(x-2)] = x
>>
>> --
>> Esta mensagem foi verificada pelo sistema de antivírus e
>> acredita-se estar livre de perigo.
>
>
>
> --
> Esta mensagem foi verificada pelo sistema de antivírus e
> acredita-se estar livre de perigo.

-- 
Esta mensagem foi verificada pelo sistema de antiv�rus e
 acredita-se estar livre de perigo.



[obm-l] Re: [obm-l] Re: [obm-l] Re: [obm-l] Dúvida em Geometria Plana

2016-06-05 Por tôpico Carlos Gomes
De nada amigo! Sempre um prazer qdo posso ajudar!

Abraço, Cgomes.

Em 2 de junho de 2016 19:03, Daniel Rocha 
escreveu:

> Muito Obrigado, Carlos !!!
>
> Em 2 de junho de 2016 18:54, Carlos Gomes  escreveu:
>
>> Seja x a medida do ângulo BAC. Como o triângulo APQ é isosceles de base
>> AP, segue q a medida do ângulo APQ também é x. Note que o ângulo BQP é
>> externo ao triângulo APQ, portanto, mede x+x=2x. Agora como o triângulo BQP
>> é isosceles de base BQ, segue que o ângulo PBQ também mede 2x. Por fim note
>> que o ângulo BPC é externo ao triângulo  ABP, portanto mede x+2x=3x...como
>> o triângulo BCP também é isosceles de base PC, segue que o ângulo PCB
>> também mede 3x...como o triângulo ABC é isosceles, segue que o ângulo ABC
>> também mede 3x, o que revela q o ângulo PCB mede x. Assim, no triângulo BCP
>> temos que
>> x+3x+3x=π   ==>x=π/7.
>> Em 2 de jun de 2016 18:32, "Daniel Rocha" 
>> escreveu:
>>
>>> Olá a todos,
>>>
>>> Alguém poderia, por favor, apresentar os cálculos corretos da seguinte
>>> questão:
>>>
>>> Considere um triângulo ABC isósceles de base BC, e os pontos P e Q tais
>>> que P pertence a AC e Q pertence a AB. Se BC=BP=PQ=QA, a medida do ângulo
>>> do vértice A, em radianos, é:
>>>
>>> GABARITO: Pi/7.
>>>
>>> Eu agradeço a quem apresentar os cálculos corretos.
>>>
>>> --
>>> Esta mensagem foi verificada pelo sistema de antivírus e
>>> acredita-se estar livre de perigo.
>>
>>
>> --
>> Esta mensagem foi verificada pelo sistema de antivírus e
>> acredita-se estar livre de perigo.
>
>
>
> --
> Esta mensagem foi verificada pelo sistema de antivírus e
> acredita-se estar livre de perigo.
>

-- 
Esta mensagem foi verificada pelo sistema de antiv�rus e
 acredita-se estar livre de perigo.



[obm-l] Re: [obm-l] Re: [obm-l] Dúvida em Geometria Plana

2016-06-02 Por tôpico Daniel Rocha
Muito Obrigado, Carlos !!!

Em 2 de junho de 2016 18:54, Carlos Gomes  escreveu:

> Seja x a medida do ângulo BAC. Como o triângulo APQ é isosceles de base
> AP, segue q a medida do ângulo APQ também é x. Note que o ângulo BQP é
> externo ao triângulo APQ, portanto, mede x+x=2x. Agora como o triângulo BQP
> é isosceles de base BQ, segue que o ângulo PBQ também mede 2x. Por fim note
> que o ângulo BPC é externo ao triângulo  ABP, portanto mede x+2x=3x...como
> o triângulo BCP também é isosceles de base PC, segue que o ângulo PCB
> também mede 3x...como o triângulo ABC é isosceles, segue que o ângulo ABC
> também mede 3x, o que revela q o ângulo PCB mede x. Assim, no triângulo BCP
> temos que
> x+3x+3x=π   ==>x=π/7.
> Em 2 de jun de 2016 18:32, "Daniel Rocha" 
> escreveu:
>
>> Olá a todos,
>>
>> Alguém poderia, por favor, apresentar os cálculos corretos da seguinte
>> questão:
>>
>> Considere um triângulo ABC isósceles de base BC, e os pontos P e Q tais
>> que P pertence a AC e Q pertence a AB. Se BC=BP=PQ=QA, a medida do ângulo
>> do vértice A, em radianos, é:
>>
>> GABARITO: Pi/7.
>>
>> Eu agradeço a quem apresentar os cálculos corretos.
>>
>> --
>> Esta mensagem foi verificada pelo sistema de antivírus e
>> acredita-se estar livre de perigo.
>
>
> --
> Esta mensagem foi verificada pelo sistema de antivírus e
> acredita-se estar livre de perigo.

-- 
Esta mensagem foi verificada pelo sistema de antiv�rus e
 acredita-se estar livre de perigo.



[obm-l] Re: [obm-l] Re: [obm-l] Dúvida "aparentemente" simples

2015-08-11 Por tôpico Pedro José
Boa tarde!

Pela definição, simétrico ou oposto de um elemento a de um anel é o
elemento do anel que operado com a por + resulte 0.
Portanto o simétrico ou oposto de zero é zero.

Saudações,
PJMS.

Em 11 de agosto de 2015 12:02, Ralph Teixeira  escreveu:

> Acho que a convencao de quase todos eh que -0=0. Nao vejo problema de que
> o oposto de algo seja ele mesmo.
>
> 2015-08-11 11:27 GMT-03:00 Vanderlei Nemitz :
>
>> Trabalho com edição de material didático e outro dia, em conversa com um
>> autor de material do ensino médio, surgiu uma questão sobre a qual gostaria
>> de saber a opinião de vocês.
>>
>> O oposto de zero é zero ou não faz sentido falar dele?
>>
>> Por um lado, a solução da equação x = -x é x = 0, ou seja, poderíamos
>> dizer que -0 = 0 (o oposto de zero é zero).
>>
>> Por outro, trata-se de um único número (ele e seu oposto) e assim não faz
>> sentido falar dele.
>>
>> A pergunta foi motivada por outra:
>>
>> Para quais valores de x temos que sqrt(x^2) = -x?
>>
>> A resposta é x < 0 ou x<= 0?
>>
>>
>> Obrigado!
>>
>> Vanderlei
>>
>> --
>> Esta mensagem foi verificada pelo sistema de antivírus e
>> acredita-se estar livre de perigo.
>
>
>
> --
> Esta mensagem foi verificada pelo sistema de antivírus e
> acredita-se estar livre de perigo.
>

-- 
Esta mensagem foi verificada pelo sistema de antiv�rus e
 acredita-se estar livre de perigo.



[obm-l] Re: [obm-l] Re: [obm-l] Re: [obm-l] dúvida

2015-07-09 Por tôpico Israel Meireles Chrisostomo
Obrigado Ralph

Em 9 de julho de 2015 12:37, Ralph Teixeira  escreveu:

> Vamos generalizar para R^n: com a noção usual (Euclideana) de comprimento,
> o comprimento do segmento que liga (x1,x2,...,xn) a (y1,y2,...,yn) é:
>
> d=raiz((y1-x1)^2+(y2-x2)^2+...+(yn-xn)^2)
>
> Esta é a noção usual de distância entre dois pontos -- confira que é o que
> você conhece na reta (n=1) e no plano (n=2).
>
> Abraço, Ralph.
>
> 2015-07-09 10:27 GMT-03:00 Pedro José :
>
>> Bom dia!
>>
>> E o segmento???
>>
>> Em 8 de julho de 2015 21:48, Israel Meireles Chrisostomo <
>> israelmchrisost...@gmail.com> escreveu:
>>
>>> Como posso encontrar o comprimento de um segmento de reta no espaço
>>> tridimensional?Considere a origem da reta no ponto (x_0,y_0,z_0) e o final
>>> da reta no ponto (x_1,y_1,z_1)
>>>
>>> --
>>> Esta mensagem foi verificada pelo sistema de antivírus e
>>> acredita-se estar livre de perigo.
>>
>>
>>
>> --
>> Esta mensagem foi verificada pelo sistema de antivírus e
>> acredita-se estar livre de perigo.
>
>
>
> --
> Esta mensagem foi verificada pelo sistema de antivírus e
> acredita-se estar livre de perigo.
>

-- 
Esta mensagem foi verificada pelo sistema de antiv�rus e
 acredita-se estar livre de perigo.



[obm-l] Re: [obm-l] Re: [obm-l] dúvida

2015-07-09 Por tôpico Ralph Teixeira
Vamos generalizar para R^n: com a noção usual (Euclideana) de comprimento,
o comprimento do segmento que liga (x1,x2,...,xn) a (y1,y2,...,yn) é:

d=raiz((y1-x1)^2+(y2-x2)^2+...+(yn-xn)^2)

Esta é a noção usual de distância entre dois pontos -- confira que é o que
você conhece na reta (n=1) e no plano (n=2).

Abraço, Ralph.

2015-07-09 10:27 GMT-03:00 Pedro José :

> Bom dia!
>
> E o segmento???
>
> Em 8 de julho de 2015 21:48, Israel Meireles Chrisostomo <
> israelmchrisost...@gmail.com> escreveu:
>
>> Como posso encontrar o comprimento de um segmento de reta no espaço
>> tridimensional?Considere a origem da reta no ponto (x_0,y_0,z_0) e o final
>> da reta no ponto (x_1,y_1,z_1)
>>
>> --
>> Esta mensagem foi verificada pelo sistema de antivírus e
>> acredita-se estar livre de perigo.
>
>
>
> --
> Esta mensagem foi verificada pelo sistema de antivírus e
> acredita-se estar livre de perigo.

-- 
Esta mensagem foi verificada pelo sistema de antiv�rus e
 acredita-se estar livre de perigo.



[obm-l] Re: [obm-l] Re: [obm-l] Dúvida Análise(tannery's theorem)

2015-05-05 Por tôpico Israel Meireles Chrisostomo
Obrigado a ambos, as suas respostas são ambas interessantes.Em particular
quero agradecer ao Ralph, que mesmo depois de eu o contrapor em
argumentos(que por sinal eram infundados) em uma outa pergunta, mesmo assim
respondeu com paciência minha dúvida

Em 5 de maio de 2015 10:40, Ralph Teixeira  escreveu:

> Não sei se entendi a pergunta também... Mas *talvez* esse seja um exemplo
> bom...
>
> Considere a sequencia dupla a(k,n) (onde k,n=1,2,3,...) dada por:
>
> a(k,n) = 1/k se n<=k
> a(k,n) = 0 se n>k
>
> Ou seja, mais explicitamente, colocando k fixo e variando n em cada linha:
>
> a(1,n): 1,0,0,0,0,0,0,..
> a(2,n): 1/2, 1/2, 0, 0, 0, 0, ...
> a(3,n): 1/3, 1/3, 1/3, 0, 0, 0, 0,...
> a(4,n): 1/4, 1/4, 1/4, 1/4, 0, 0, 0, 0,...
>
> Então S(k)=sum(n=1 a +Inf) a(k,n) = 1 (soma da linha), e portanto lim
> (k->Inf) S(k) = 1 também.
>
> Porém se você fixar n e tomar primeiro o limite k->Inf, verá que lim
> (k->Inf) a(k,n) = 0. Então a soma dos limites dos a(k,n) é 0 -- não deu 1!
>
> Em suma: a soma (infinita) dos limites não é o limite da soma (infinita).
>
> Era algo assim que você queria?
>
> Abraço, Ralph.
>
> 2015-05-05 1:40 GMT-03:00 Israel Meireles Chrisostomo <
> israelmchrisost...@gmail.com>:
>
>> Olá tenho um dúvida de análise seja a_k(n) um termo dependente de n e a_k
>> o resultado do limite lim n->inf a_k(n)=a_k, se |Sa_k(n)-Sa_k|<épsilon, com
>> épsilon maior que zero então, isto significa dizer que lim
>> n->inf  Sa_k(n)=S a_k(em que S está no lugar de sigma e representa a soma
>> da série)?Se a resposta é sim, alguém poderia me explicar o pq?No caso, eu
>> não deveria ter épsilon próximo de zero?Tipo assim, para concluir que ambas
>> se são "iguais", se alguém puder me ajudar, é que estou lendo sobre o
>> Tannery's Theorem para séries
>>
>>
>> --
>> Esta mensagem foi verificada pelo sistema de antivírus e
>> acredita-se estar livre de perigo.
>
>
>
> --
> Esta mensagem foi verificada pelo sistema de antivírus e
> acredita-se estar livre de perigo.
>

-- 
Esta mensagem foi verificada pelo sistema de antiv�rus e
 acredita-se estar livre de perigo.



[obm-l] RE: [obm-l] Re: [obm-l] Dúvida(questão simples)

2014-02-22 Por tôpico marcone augusto araújo borges
Obrigado!

Date: Sat, 22 Feb 2014 00:31:24 -0300
Subject: [obm-l] Re: [obm-l] Dúvida(questão simples)
From: tarsise...@gmail.com
To: obm-l@mat.puc-rio.br

Erramos juntos. Pq tb achei 58.

--

Esta mensagem foi verificada pelo sistema de antivírus e 

 acredita-se estar livre de perigo.   
-- 
Esta mensagem foi verificada pelo sistema de antivírus e
 acredita-se estar livre de perigo.



[obm-l] RE: [obm-l] Re: [obm-l] Dúvida(questão simples)

2014-02-22 Por tôpico marcone augusto araújo borges
Eu pensei assim também.Obrigado!

From: ilhadepaqu...@bol.com.br
To: obm-l@mat.puc-rio.br
Subject: [obm-l] Re: [obm-l] Dúvida(questão simples)
Date: Sat, 22 Feb 2014 00:28:27 -0300








quando ele anda no sentido horário ele anda 380 
graus em 40 minutos porque o ponteiro das horas em 40 minutos andou 20 
graus
 
Assim sua velocidade é de 9,5 graus por 
minuto
 
o ponteiro das horas  anda a 0,5 grau por 
minuto
 
logo falta calcular  quando vão se encontrando 
e a velociadade conjunta é de 10 graus por minuto
 
para percorrer 180 graus eles vão demorar 18 
minutos 
 
40 +18 = 58 minutos
 
abraços 
Hermann
 
 
 
 

  - Original Message - 
  From: 
  marcone augusto araújo borges 
  
  To: obm-l@mat.puc-rio.br 
  Sent: Friday, February 21, 2014 11:33 
  PM
  Subject: [obm-l] Dúvida(questão 
  simples)
  

  Exatamente no  momento em que o ponteiro das horas passa 
  pelo 12, uma formiga começa a andar ao longo
da borda de um relógio no 
  sentido anti-horário,partindo do 6,com velocidade constante.Quando a formiga 
  en-
contra o ponteiro das horas,ela muda de direção e continua a andar na 
  mesma velocidade no sentido horário
Quarenta minutos após o primeiro 
  encontro,a formiga se encontra pela segunda vez com o ponteiro das horas 
e 
  morre.Quanto tempo a formiga andou?

O gabarito dá 54 minutos mas eu só 
  tô achando 58.
Agradeço por esclarecimento.


-- 
Esta 
  mensagem foi verificada pelo sistema de antivírus e 
acredita-se estar 
  livre de perigo. 
--

Esta mensagem foi verificada pelo sistema de antivírus e 

 acredita-se estar livre de perigo.   
-- 
Esta mensagem foi verificada pelo sistema de antivírus e
 acredita-se estar livre de perigo.



[obm-l] Re: [obm-l] RE: [obm-l] RE: [obm-l] Dúvida Indução

2012-05-17 Por tôpico Ralph Teixeira
Cuidado: ao passar de n=k para n=k+1 no Passo de Inducao... o ultimo
termo "era" 3n-1, agora eh 3(n+1)-1=3n+2 -- nao eh questao de "somar
um no termo", eh "trocar n por n+1".

Abraco,
 Ralph

2012/5/17 Thiago Bersch :
> Então eu estava tentando fazer mas parava no mesmo ponto, fazia
> 2+5+8+...+(3n-1)+[(3n-1)+1], chegando aí eu me perco
>
> 
> From: joao_maldona...@hotmail.com
> To: obm-l@mat.puc-rio.br
> Subject: [obm-l] RE: [obm-l] Dúvida Indução
> Date: Mon, 14 May 2012 15:24:47 -0300
>
> Vamos dizer que para n respeite a formula
> Logo 2+4+6+...+2n=n.(n+1)
> Somando 2n+2
> 2+4+6+...+(2n+2=n(n+1)+2n+2=(n+1)(n+2) que respeita a formula
> Logo se vale para n, vale para n+1
> Como vale para 1, vale para 2, e  entao para 3, 4, 5...
> Vale para qualquer natural
>
> Tente fazer o segundo agora
> []s Joao
>
> 
> From: thiago_t...@hotmail.com
> To: obm-l@mat.puc-rio.br
> Subject: [obm-l] Dúvida Indução
> Date: Mon, 14 May 2012 01:09:39 -0300
>
>  2 + 4 + . . . + 2n.
>  2 + 5 + 8 + . . . + (3n-1).
> Bem eu sei que o primeiro irá dar n(n+1) e o segundo n(3n+1)/2
> O que em si eu não entendi o resultado
> O primeiro eu tentei fazer assim:
> 2+4...+2n
> +
> n+2n+(2n+1),  e fiquei parado nisso e o segunda também, gostaria de uma
> explicação passo-a-passo pois não entendo.

=
Instruções para entrar na lista, sair da lista e usar a lista em
http://www.mat.puc-rio.br/~obmlistas/obm-l.html
=


[obm-l] RE: [obm-l] RE: [obm-l] Dúvida Indução

2012-05-16 Por tôpico Thiago Bersch

Então eu estava tentando fazer mas parava no mesmo ponto, fazia 
2+5+8+...+(3n-1)+[(3n-1)+1], chegando aí eu me perco

From: joao_maldona...@hotmail.com
To: obm-l@mat.puc-rio.br
Subject: [obm-l] RE: [obm-l] Dúvida Indução
Date: Mon, 14 May 2012 15:24:47 -0300




Vamos dizer que para n respeite a formula
Logo 2+4+6+...+2n=n.(n+1)
Somando 2n+2
2+4+6+...+(2n+2=n(n+1)+2n+2=(n+1)(n+2) que respeita a formula
Logo se vale para n, vale para n+1
Como vale para 1, vale para 2, e  entao para 3, 4, 5...
Vale para qualquer natural

Tente fazer o segundo agora
[]s Joao

From: thiago_t...@hotmail.com
To: obm-l@mat.puc-rio.br
Subject: [obm-l] Dúvida Indução
Date: Mon, 14 May 2012 01:09:39 -0300





 2 + 4 + . . . + 2n. 2 + 5 + 8 + . . . + (3n-1).Bem eu sei que o primeiro irá 
dar n(n+1) e o segundo n(3n+1)/2O que em si eu não entendi o resultado O 
primeiro eu tentei fazer assim:2+4...+2n + n+2n+(2n+1),  e fiquei parado nisso 
e o segunda também, gostaria de uma explicação passo-a-passo pois não entendo.  


  

[obm-l] Re: [obm-l] Re: [obm-l] Dúvida de geometria analítica: É perpendicular, portanto... (?)

2012-03-22 Por tôpico Felipe Blassioli
Muito obrigado.

Fiquei perdido porque o livre fez essa passagem do "portanto" sem fazer
referência explícita à operação vetorial e eu não tinha conhecimento algum
de álgebra linear, por isso fiquei perdido, mas agora deu pra situar ^^

2012/3/22 Eduardo Wilner 

> Antes de mais nada seria interessante verificar se os símbolos a e b que
> aparecem em lugares diferentes têm o mesmo significado...?  Parece que
> houve alguma confusão nesse ponto.
>
> Mas sendo A(a,b) um ponto da perpendicular à reta r passando pela origem,
> os vetores
>
> (a,b) e [(x_1 - x_2),(y_1 -y_2)] são perpendiculares, daí segue a condição
> de que o produto escalar a(x_1 - x_2) + b (y_1 - y_2) seja nulo.
>
> [ ]'s
>
> --- Em *qua, 21/3/12, Felipe Blassioli *escreveu:
>
>
> De: Felipe Blassioli 
> Assunto: [obm-l] Dúvida de geometria analítica: É perpendicular,
> portanto... (?)
> Para: obm-l@mat.puc-rio.br
> Data: Quarta-feira, 21 de Março de 2012, 21:41
>
>
> Pessoal,
> alguém poderia me explicar porque
>
> Seja a reta r representada pela equação ax + by = c.
> Seja A = (a,b) um ponto diferente de O sobre a reta OA, perpendicular a r
> que é baixada da origem O.
>
> Se P = (x_1, y_1) e Q = (x_2, y_2) são dois pontos quaisquer sobre a reta
> r, o segmento PQ é perpendicular a OA, portanto
>
> a(x_1 - x_2) + b (y_1 - y_2) = 0
>
> ?
>
>


[obm-l] RE: [obm-l] Re: [obm-l] Dúvida

2012-03-12 Por tôpico Vanessa Nunes de Souza

Obrigada!Vanessa Nunes

Date: Mon, 12 Mar 2012 17:04:46 -0300
Subject: [obm-l] Re: [obm-l] Dúvida
From: tarsise...@gmail.com
To: obm-l@mat.puc-rio.br

a) Fazendo-se a diferença entre 

(100c+10b+a) - (100a+10b+c) = 396
99c-99a=396
c-a=4

Sendo 2c=a => a=4 e c=8, como a,b,c formando uma PA, temos que b=6

b) A área do triângulo QMN pode ser dada por S1 = [(QP/2)x(QR/2)xsenQ]/2 => 
S1=(QPxQRxsenQ)/8



A área do triângulo QPR pode ser dada por S2 = (QPxQRxsenQ)/2

Donde tiramos que S2/4 = S1

Sendo a área do quadrilátero PMNR dada por S3 = 51, e sendo S3 = S2 - S1

S2 - S2/4 = 51 => S2 = 68m²






On Mon, Mar 12, 2012 at 2:42 PM, Vanessa Nunes de Souza 
 wrote:






 Gostaria da ajuda dos colegas para resolver duas questões:
 
 



  


  


  


  


 
 



  


  


  


  


1- Um número de três algarismos, a, b e c do sistema de numeração decimal, é 
representando por abc, isto é, abc=100a+ 10b+ c. Escrito na ordem inversa(cba), 
esse número aumenta de 396 unidades. Sabendo-se qie (a,b,c) forma, nessa ordem, 
uma progressão aritmética e que c=2a, o algarismo b é igual a:

a)6   b) 5   c) 4   d) 3
2-Considere um triângulo escaleno PQR onde M e N representam, respectivamente, 
os pontos médios dos lados PQ e QR. Se a área do quadrilátero PMNR é igual a 
51m^2, a área do triângulo PQR, em m^2, é igual a: a) 68 b) 54 c)72 
  d) 84


Obrigada
Vanessa Nunes 

  

[obm-l] Re: [obm-l] Re: [obm-l] Dúvida de Lógica

2012-02-08 Por tôpico Ralph Teixeira
H nao sei nao. Vou usar "C" para "estah contido" e "E" para
"pertence a".

Concordo que toda inclusao de conjuntos pode ser pensada como uma
implicacao (bom, com um quantificador "para todo"). Afinal:

A C B
eh o mesmo que dizer
para todo x, xEA => xEB

Por isso, concordo que a Teoria dos Conjuntos e a Logica Matematica estao
profundamente emaranhadas Mas nem toda sentenca logica tem um conjunto
-- tem que ter uma variavel na sentenca logica!

Assim, se voce considera a proposicao "x^3+x+1>0", a ela estah associada o
conjunto A dos valores de x que satisfazem aquela proposicao. Mas aquela
proposicao eh ABERTA, isto eh, tem uma variavel x SOLTA nela, entao faz
sentido falar do conjunto dos x que a satisfaz. (Variavel SOLTA eh uma que
pode ser substituida por valores; uma proposicao ABERTA nunca eh nem V nem
F -- mas se voce substituir valores em todas as variaveis soltas de uma
proposicoa aberta, ai ela passa a ser V ou F, dependendo dos valores)

Agora, na proposicao "2>3", nao tem x. Entao nao faz sentido pensar no
conjunto A que voce cita, pelo menos nao diretamente. Voce poderia forcar a
barra e dizer que a proposicao eh r(x): 2>3, e entao eu concordo contigo,
mas tem que dizer algo explicito sobre a variavel que NAO aparece na
proposicao.

(Isto eh mais ou menos equivalente aa diferenca entre escrever f=5 ou
f(x)=5; se voce escreve do primeiro jeito, eu vou pensar que f eh um
NUMERO, igual a 5; do segundo jeito, eu penso que f eh a FUNCAO CONSTANTE e
igual a 5, cuja variavel x calhou de nao aparecer na expressao de f. Sao
objetos distintos.)

Idem para a segunda proposicao: "Todo brasileiro eh desonesto", nao tem
variavel solta, eh V ou F e acabou. Sim, ela eh equivalente a "Para todo y,
se y eh brasileiro entao y eh desonesto", mas note que nesta sentenca y nao
eh uma variavel SOLTA -- o "para todo y" da frente te proibe de substituir
valores em y. A proposicao "se y eh brasileiro entao y eh desonesto" eh
ABERTA e permite susbtituir valores para y, mas ela nao tem o "para todo".
Uma grande parte da confusao eh que a gente costuma pensar numa implicacao
e incluir o "para todo" nela sem escrever...

To sendo confuso e chato, eu sei mas eh que eu acho que o tempo que se
gasta em teoria dos conjuntos no ensino fundamental e medio podia ser MUITO
melhor aproveitado indo direto na logica -- e eu nao sei explixar tudo isso
em poucas linhas.

Abraco,
  Ralph

2012/2/7 Francisco Barreto 

> Saudações a todos!
> Seja A o conjunto dos objetos que satisfazem a propriedade r  de que 2 > 3.
> Seja B o conjunto dos objetos que satisfazem a propriedade s de que "Todo
> brasileiro é desonesto"
>
> Sabemos que A é o conjunto vazio. O conjunto vazio está contido em
> qualquer conjunto, incluindo B. Portanto os elementos de A (se você
> encontrar) devem satisfazer a  propriedade s de que todo brasileiro é
> desonesto.
>
>
>
> 2012/2/7 Pedro Chaves 
>
>>
>> Caros Colegas,
>>
>> Pode-se dizer que "2 > 3 => Todo brasileiro é desonesto"?   (O símbolo =>
>> indica implicação lógica.)
>>
>>
>> Sei que é verdadeira a proposição condicional "Se 2 > 3, então todo
>> brasileiro é desonesto", mas me parece que não existe implicação lógica.
>>
>> Desde já, muito obrigado.
>> Um abraço do Pedro Chaves!
>> =
>> Instruções para entrar na lista, sair da lista e usar a lista em
>> http://www.mat.puc-rio.br/~obmlistas/obm-l.html
>> =
>>
>
>
>
> --
> Sinceramente,
> Francisco Costa D. Barreto
>
>


[obm-l] Re: [obm-l] RE: [obm-l] Dúvida em gabarito

2011-11-27 Por tôpico JOSE AIRTON CARNEIRO
Sua resposta está correta. A área da figura em questão é formada por um
triângulo equilátero + 2 segmentos circulares de arcos 60°. = a^2(4pi -
3raiz(3))/12.
Airton
Em 26 de novembro de 2011 15:19, João Maldonado  escreveu:

>
> Deixa  eu ver se entendi
>
>
> Dado um  quadrado,  construa dois circulos  de centro em  vértices
> adjacentes e raios iguais a a (que é o lado do quadrado).  Calcule a região
> em comum entre os círculos e o quadrado.
>
> Faça assim divida o quadrado no meio,  vai originar duas figuras
> semelhantes.  A área de cada uma pode ser calculada:
>  (Área do setor-Área do triângulo)/2  *  2
> (pois  são duas figuras)
>
> Pi.a²/3 -  a²sqrt(3)/4 = a²(4Pi-3sqrt(3))/12
>
> João
>
>  --
> From: marconeborge...@hotmail.com
> To: obm-l@mat.puc-rio.br
> Subject: [obm-l] Dúvida em gabarito
> Date: Sat, 26 Nov 2011 11:28:50 +
>
>
> seja um quadrado ABCD de lado a.Os arcos AC e BD formam com 3 dos lados do
> quadrado dois setores ciculares(um quarto do círculo de raio a
> cada).Deteminar a área comum aos setores circulares.
> A resposta que o livro dá é a^2(9raiz(3) - 4pi)/12 e a que eu achei foi
> a^2(4pi - 3raiz(3))/12
> Dsculpem a falta da figura.Agradeço a quem puder ajudar.
>


[obm-l] Re: [obm-l] Re: [obm-l] Re: [obm-l] Re: [obm-l] Dúvida

2011-11-02 Por tôpico Kleber Bastos
Achei que faltava a regra para f(x),mas a menina disse que não. Vou
verificar com a pessoa e retorno.
Obrigado!

Em 02/11/2011 23:08, "Joao Maldonado" escreveu:



Não tem como ser isso não cara
Traduz isso aí que   não dá pra entender

O que poderia ser é
Mostre que  qualquer que seja o número  racional e positivo  a/b com a e  b
inteiros  primos entre si,  é válido   que f(a/b) =  f(1)^(a/b)



Tudo bem,  vamos dizer  que é isto, mas qual a regra para  a função f(x)??

Não são  todas as funções f(x)  que  satisfazem tal  afirmação  (aliás,  a
única   que consegui pensar até agora é  f(x) = a^x

f(x) = x²,  f(4/7) = 16/49 !=   1^(4/7) = 1

[]'s

João



--
Date: Wed, 2 Nov 2011 17:17:59 -0200
Subject: [obm-l] Dúvida
From: klebe...@gmail.com
To: obm-l@mat.puc-rio.br



Olá grupo,
Estou me enrolando nesta prova.

Mostre q ∀ nº a/b>0, MDC(a,b) = 1,
é válido: f(a/b) =...

--
*De:* Kleber Bastos 
*Para:* obm-l@mat.puc-rio.br
*Enviadas:* Quarta-feira, 2 de Novembro de 2011 22:21
*Assunto:* [obm-l] Re: [obm-l] Re: [obm-l] Dúvida


É isso mesmo:
Mostrar que ∀ nº racional a/b>0, M.D.C.(a,b)=1 é válida a sentença:
f(a/b)=f(1)^a/b (...


[obm-l] Re: [obm-l] Re: [obm-l] Re: [obm-l] Dúvida

2011-11-02 Por tôpico Joao Maldonado



Não tem como ser isso não cara
Traduz isso aí que   não dá pra entender



O que poderia ser é
Mostre que  qualquer que seja o número  racional e positivo  a/b com a e  b 
inteiros  primos entre si,  é válido   que f(a/b) =  f(1)^(a/b)    



Tudo bem,  vamos dizer  que é isto, mas qual a regra para  a função f(x)??

Não são  todas as funções f(x)  que  satisfazem tal  afirmação  (aliás,  a 
única   que consegui pensar até agora é  f(x) = a^x

f(x) = x²,  f(4/7) = 16/49 !=   1^(4/7) = 1

[]'s

João






Date: Wed, 2 Nov 2011 17:17:59 -0200
Subject: [obm-l] Dúvida
From: klebe...@gmail.com
To: obm-l@mat.puc-rio.br

Olá grupo,
Estou me enrolando nesta prova.

Mostre q ∀ nº a/b>0, MDC(a,b) = 1, 
é válido: f(a/b) = f(1)^a/b .

-- 
Kleber.



De: Kleber Bastos 
Para: obm-l@mat.puc-rio.br
Enviadas: Quarta-feira, 2 de Novembro de 2011 22:21
Assunto: [obm-l] Re: [obm-l] Re: [obm-l] Dúvida


É isso mesmo:
Mostrar que ∀ nº racional a/b>0, M.D.C.(a,b)=1 é válida a sentença: 
f(a/b)=f(1)^a/b ( f(1) elevado a a/b)


Em 2 de novembro de 2011 20:57, Victor Hugo Rodrigues 
 escreveu:

Como assim? Acho que falta algo aí.
>
>
>Em 2 de novembro de 2011 17:17, Kleber Bastos  escreveu:
>
>
>Olá grupo,
>>Estou me enrolando nesta prova.
>>
>>Mostre q ∀ nº a/b>0, MDC(a,b) = 1, 
>>é válido: f(a/b) = f(1)^a/b
.
>>
>>-- 
>>Kleber.
>>
>


-- 
Kleber B. Bastos

[obm-l] Re: [obm-l] Re: [obm-l] Dúvida

2011-11-02 Por tôpico Kleber Bastos
É isso mesmo:
Mostrar que ∀ nº racional a/b>0, M.D.C.(a,b)=1 é válida a sentença:
f(a/b)=f(1)^a/b ( f(1) elevado a a/b)

Em 2 de novembro de 2011 20:57, Victor Hugo Rodrigues <
victorhcr.victorh...@gmail.com> escreveu:

> Como assim? Acho que falta algo aí.
>
> Em 2 de novembro de 2011 17:17, Kleber Bastos escreveu:
>
> Olá grupo,
>> Estou me enrolando nesta prova.
>>
>> Mostre q ∀ nº a/b>0, MDC(a,b) = 1,
>> é válido: f(a/b) = f(1)^a/b .
>>
>> --
>> Kleber.
>>
>
>


-- 
Kleber B. Bastos


[obm-l] Re: [obm-l] Re: [obm-l] Dúvida

2011-10-24 Por tôpico Julio César Saldaña



O mesmo visto de outro modo:

Lucas e Pedro tem a mesma velocidade. Então quando lucas sai da ponte, pedro
percorriou 2/5 da ponta, ou seja falta ainda 1/5 da ponte. Então o trem percorre
a ponte inteira no mesmo tempo que pedro percorre 1/5 da ponte, ouseja o trem é
5 vezes mais veloz que pedro, portanto su velocidade é 5x15=75.

Julio Saldaña


-- Mensaje original ---
De : obm-l@mat.puc-rio.br
Para : obm-l@mat.puc-rio.br
Fecha : Mon, 24 Oct 2011 17:08:45 -0200
Asunto : [obm-l] Re: [obm-l] Dúvida

Lucas corre 2/5 da ponte com velocidade de 15 km/h. Sendo p o comprimento da
ponte, leva (2/5)p/15 = 2p/75 h para sair da ponte.
Pedro corre 3/5 da ponte com velocidade de 15 km/h. Sendo p o comprimento da
ponte, leva (3/5)p/15 = 3p/75 h para sair da ponte.

A diferença entre o momento em que Lucas sai da ponte (momento em que o trem
entra na ponte) e o momento em que Pedro sai dela (momento em que o trem sai
da ponte) é 3p/75 - 2p/75 = p/75. Este é o tempo que o trem leva para
percorrer a ponte, ou seja, para percorrer p km.

Portanto, a velocidade do trem será dada por p/(p/75) = 75 km/h, alternativa
C

Abraços.

Hugo.

Em 24 de outubro de 2011 15:29, Jorge Paulino da Silva Filho <
jorge...@yahoo.com.br> escreveu:


**
Dois amigos, Lucas e Pedro, seguiam o leito de uma ferrovia e começaram a
atravessar uma ponte estreita na qual havia espaço apenas para o trem. No
momento em que completavam 2/5 do percurso da ponte, ouviram o trem que
se aproxima por trás deles. Lucas começou a correr de encontro ao trem,
saindo da ponte praticamente no instante em que o trem entrava. Pedro,
que correu no sentindo oposto ao sentido de Lucas, conseguir sair da ponte
praticamenteno instante em que o trem saía. Sendo 15km/h a velocidade que
Lucas
e Pedro correram, assinale a alternativa que contém a velocidade do trem:

a)60km/h
b)37km/h
c)75km/h
d)30km/h
e)67,5km/h




__
Si desea recibir, semanalmente, el Boletín Electrónico de la PUCP, ingrese a:
http://www.pucp.edu.pe/puntoedu/suscribete/

=
Instruções para entrar na lista, sair da lista e usar a lista em
http://www.mat.puc-rio.br/~obmlistas/obm-l.html
=


[obm-l] RE: [obm-l] Re: [obm-l] dúvida - teoria dos números

2011-08-05 Por tôpico Rhilbert Rivera

Pequeno Teorema de Fermat: a^(p-1) ==1(mod p), se mdc(a,p)=1.

Como 47 é primo e mdc(2,47) =1, então  2^46 ==1 (mod 47). É claro que podemos 
dizer ( de acordo com as propriedades das potências nas  congruências) que 
2^23==1 (mod 47), o que nos leva 2^23 -1 ==0(mod 47).

Date: Sun, 31 Jul 2011 16:00:56 -0700
From: jeffma...@yahoo.com.br
Subject: [obm-l] Re: [obm-l] dúvida - teoria dos números
To: obm-l@mat.puc-rio.br

Tente pensar no pequeno Teorema de Newton, ou se preferir use congruencias.abs
De: Marco Antonio Leal 
Para: obm-l@mat.puc-rio.br
Enviadas: Quinta-feira, 28 de Julho de 2011 21:39
Assunto: [obm-l] dúvida - teoria dos números






Não estou conseguindo uma prova satisfatória para o seguinte exercício:

 

prove que 2 ^23  -  1 é divisivel por 47.

 
   


  

[obm-l] Re: [obm-l] Re: [obm-l] RE: [obm-l] Re: [obm-l] Re: [obm-l] dúvida sobre séries

2011-06-08 Por tôpico Artur Costa Steiner
O critério mais simples para mostrar que a série harmônica diverge talvez
seja o baseado no seguinte teorema:

Se x_n é uma sequência decrescente de reais tal que Soma x_n converge, então
lim n x_ n = 0. (Prove isto)

Se x_n = 1/n, x_n decresce para 0 mas lim n x_n = 1, o que mostra que Soma
x_n diverge. Para infinito, pois os termos são positivos.

Mas talvez não seja uma prova tão elucidativa quanto as outras dadas.

Artur

Artur Costa Steiner
Em 07/06/2011 11:29, "Rodrigo Renji"  escreveu:
>
> Olá!
>
> Então acho bem bacana esse também ( e nem é tão complicado de
> demonstrar, eu acho )
>
>
> Esse critério pode ser usado para estudar a convergência de [ SOMA de
> 1/ k^p  ]  também
>
> pois  [ SOMA de 2^k / 2^(kp)  ]  =  [ SOMA de 2^(k (1-p))  ]
>
> se 1 - p< 0, isto é 1< p a série converge por série geometrica
>
> se 1-p > 0 , 1 > p a série diverge de novo por série geometrica .
>
> =
> Instruções para entrar na lista, sair da lista e usar a lista em
> http://www.mat.puc-rio.br/~obmlistas/obm-l.html
> =


[obm-l] Re: [obm-l] RE: [obm-l] Re: [obm-l] Re: [obm-l] dúvida sobre séries

2011-06-07 Por tôpico Rodrigo Renji
Olá!

Então acho bem bacana esse também ( e nem é tão complicado de
demonstrar, eu acho )


Esse critério pode ser usado para estudar a convergência de [ SOMA de
1/ k^p  ]  também

pois  [ SOMA de 2^k / 2^(kp)  ]  =  [ SOMA de 2^(k (1-p))  ]

se 1 - p< 0, isto é 1< p a série converge por série geometrica

se 1-p > 0 , 1 > p a série diverge de novo por série geometrica .

=
Instruções para entrar na lista, sair da lista e usar a lista em
http://www.mat.puc-rio.br/~obmlistas/obm-l.html
=


[obm-l] RE: [obm-l] Re: [obm-l] Re: [obm-l] dúvida sobre séries

2011-06-07 Por tôpico Luís Lopes

Sauda,c~oes, 

Legal este critério, parece ter sido criado para a série harm. 

E a esse respeito, o autor da pergunta poderia ler também sobre 
a constante de Euler. 

[]'s 
Luís 

> Date: Mon, 6 Jun 2011 23:50:37 -0300
> Subject: [obm-l] Re: [obm-l] Re: [obm-l] dúvida sobre séries
> From: rodrigo.uff.m...@gmail.com
> To: obm-l@mat.puc-rio.br
> 
> Olá!
> 
> Uma outra maneira  ( além da que os colegas enviaram antes), para
> mostrar que a série  não converge, tem um critério de convergência que
> acho legal, Critério de condensação de Cauchy:
> 
> 
>  Se x_k é uma sequência decrescente de termos positivos ( como é o caso de 
> 1/k )
> 
> 
> então a série [ SOMA de x_k]  converge , se e somente se , a série [
> SOMA de 2^k  x_(2^k) ]  converge.
> 
> Aplicando isso para a série do email
> 
> temos com a_k= 1/k
> 
> 
> [ SOMA de 2^k  x_(2^k) ]  =  [ SOMA de 2^k  ,  1/ (2^k)   ] =  [ SOMA 1 ]
> 
> que diverge, pois somando de 1 até n resulta em "n", com n indo pro
> infinito , diverge  : ) Pode não ajudar muito, mas acho esse critério
> legal
> 
> abraço
> 
> =
> Instruções para entrar na lista, sair da lista e usar a lista em
> http://www.mat.puc-rio.br/~obmlistas/obm-l.html
> =
  

[obm-l] Re: [obm-l] Re: [obm-l] dúvida sobre séries

2011-06-06 Por tôpico Rodrigo Renji
Olá!

Uma outra maneira  ( além da que os colegas enviaram antes), para
mostrar que a série  não converge, tem um critério de convergência que
acho legal, Critério de condensação de Cauchy:


 Se x_k é uma sequência decrescente de termos positivos ( como é o caso de 1/k )


então a série [ SOMA de x_k]  converge , se e somente se , a série [
SOMA de 2^k  x_(2^k) ]  converge.

Aplicando isso para a série do email

temos com a_k= 1/k


[ SOMA de 2^k  x_(2^k) ]  =  [ SOMA de 2^k  ,  1/ (2^k)   ] =  [ SOMA 1 ]

que diverge, pois somando de 1 até n resulta em "n", com n indo pro
infinito , diverge  : ) Pode não ajudar muito, mas acho esse critério
legal

abraço

=
Instruções para entrar na lista, sair da lista e usar a lista em
http://www.mat.puc-rio.br/~obmlistas/obm-l.html
=


[obm-l] Re: [obm-l] Re: [obm-l] Re: [obm-l] Dúvida - OBM Nível Universitário

2011-01-24 Por tôpico Tiago
Seria uma olimpiada mais dificil ainda de ganhar, haha.

2011/1/24 Hugo Fernando Marques Fernandes 

> Oi, Bruna.
>
> Pois é, eu já tinha ouvido dizer isso e queria confirmar.
> É uma pena, mas fazer o que? Regras são regras...
>
> Talvez fosse o caso de criar uma categoria nova pra quem já tem diploma,
> né?
>
> Obrigado pela resposta.
>
> Hugo.
>
> Em 24 de janeiro de 2011 02:20, Bruna Campos escreveu:
>
> PS.: E só até o quarto ano de graduação.
>>
>> Abraços!
>>
>> Em 23/01/11, Bruna Campos escreveu:
>> > Hugo, que eu saiba não pode. Só pode participar quem não tem diploma
>> > de curso superior :(
>> >
>> >
>> >
>> >
>> > Em 20/01/11, Hugo Fernando Marques Fernandes
>> > escreveu:
>> >> Boa noite.
>> >>
>> >> Acabo de ser aprovado para o curso de matemática da UERJ e tenho uma
>> >> dúvida
>> >> em relação à OBM de nível universitário. Sendo esta minha segunda
>> >> graduação,
>> >> ou seja, possuindo um outro diploma de nível superior, ainda assim
>> >> poderei
>> >> participar?
>> >>
>> >> Grato por sua atenção.
>> >>
>> >> Hugo F. M. Fernandes.
>> >>
>> >
>>
>> =
>> Instruções para entrar na lista, sair da lista e usar a lista em
>> http://www.mat.puc-rio.br/~obmlistas/obm-l.html
>> =
>>
>
>


-- 
Tiago J. Fonseca
http://legauss.blogspot.com


[obm-l] Re: [obm-l] Re: [obm-l] Dúvida - OBM Nível Universitário

2011-01-24 Por tôpico Hugo Fernando Marques Fernandes
Oi, Bruna.

Pois é, eu já tinha ouvido dizer isso e queria confirmar.
É uma pena, mas fazer o que? Regras são regras...

Talvez fosse o caso de criar uma categoria nova pra quem já tem diploma, né?

Obrigado pela resposta.

Hugo.

Em 24 de janeiro de 2011 02:20, Bruna Campos escreveu:

> PS.: E só até o quarto ano de graduação.
>
> Abraços!
>
> Em 23/01/11, Bruna Campos escreveu:
> > Hugo, que eu saiba não pode. Só pode participar quem não tem diploma
> > de curso superior :(
> >
> >
> >
> >
> > Em 20/01/11, Hugo Fernando Marques Fernandes
> > escreveu:
> >> Boa noite.
> >>
> >> Acabo de ser aprovado para o curso de matemática da UERJ e tenho uma
> >> dúvida
> >> em relação à OBM de nível universitário. Sendo esta minha segunda
> >> graduação,
> >> ou seja, possuindo um outro diploma de nível superior, ainda assim
> >> poderei
> >> participar?
> >>
> >> Grato por sua atenção.
> >>
> >> Hugo F. M. Fernandes.
> >>
> >
>
> =
> Instruções para entrar na lista, sair da lista e usar a lista em
> http://www.mat.puc-rio.br/~obmlistas/obm-l.html
> =
>


[obm-l] Re: [obm-l] Re:[obm-l] Dúvida-Geometria ana lítica

2010-10-11 Por tôpico Ralph Teixeira
Eu pensei dum jeito mais "desenho geométrico". No fundo, é a mesma solução
do Marcone acima, só pensada de outro jeito:

Faça uma figura com A, B e a reta y=6 onde mora C. Novamente, o ponto-chave
da questão é D, o médio de BC.

Como C está na reta y=6 e B=(4,0), o médio D está na reta y=3 (homotetia de
centro B e razão 1/2 aplicada à reta y=3).

Por outro lado, D está no círculo de centro A e raio 3. Intersectando este
círculo com a reta anterior, achamos as possíveis posições para D.

Neste problema em particular, a interseção destes 2 L.G.s é única, é o ponto
(0,3) (o círculo é tangente à reta y=3). Então D=(0,3), portanto C=(-4,6).
Como a primeira coordenada de C é negativa, o ângulo em A é obtuso, o
triângulo é obtusângulo.

Abraço,
 Ralph


[obm-l] RE: [obm-l] Re: [obm -l] Dúvida-Geometria ana lítica

2010-10-11 Por tôpico marcone augusto araújo borges

Eu tambem estranhei essas medidas mas penso q o triângulo existe.A mediana de 
BC divide o triângulo ABC em dois de mesma àrea((12/2)=6 unidades).Seja D o pé 
da referida mediana.A área do triângulo ABD=(1/2)*4*3*senÂ=6.Dai,senÂ=1 e  é 
um ângulo reto,então BD=CD=5 implica BC=10.Nesse caso,a projeção de AC sobre a 
reta suporte de AB mede 4 e AC = raiz(52).Portanto,ABC seria obtusângulo,pois 
BC^2>AB^2+AC^2(100>16+52).Para testar,calculei a área de ABC usando as medidas 
dos seus lados e o resultado bateu:12 unidades.Quando perguntei não estava tão 
convicto e queria ver uma solução diferente tambem.

Obrigado pela sua atenção.
 


Date: Sun, 10 Oct 2010 16:57:11 -0700
From: eduardowil...@yahoo.com.br
Subject: [obm-l] Re: [obm-l] Dúvida-Geometria ana lítica
To: obm-l@mat.puc-rio.br





Pelo menos na geometria euclidiana esse triângulo inexiste: a altura relativa à 
AB vale 6, logo o pé da referida mediana está à uma altura de 3, logo sua 
medida tem que ser maior... 

  

[obm-l] Re: [obm-l] RE: [obm-l] Re: [obm-l] dúvida sobre le ma de teoria dos números.

2010-06-05 Por tôpico Bernardo Freitas Paulo da Costa
2010/6/5 Lucas Hagemaister 
>
>  Hum... Entendi. Obrigado!
> O que mais ou menos o lema quer dizer é o seguinte:
> Sempre que termos m|a e n|a, onde mn|a, m e n serão primos entre si.
Tivermos, para não assassinar o português. E não, cuidado com a ordem
das implicações. A e B => C  não quer dizer que A e C => B Veja
bem, podemos ter
20 | 100
5 | 100
20*5 | 100
mas 20 e 5 não são primos entre si.

Muito cuidado com a ordem das coisas... Que nem aquela do "todo corvo
é preto, ora, minha ovelha é preta, logo minha ovelha é um corvo".

> O que eu fiz foi o contrário(ali no caso do 4 e 10):
O que você fez foi usar a conclusão de um teorema sem verificar as
hipóteses... Por exemplo, "todo ser vivo é mortal, logo os
computadores são mortais". Repare que falta o conectivo "ora, os
computadores são seres vivos". (que, por sinal, me parece falsa...)

> Sempre que termos m e n primos entre si, onde m|a e n|a, mn|a.
> Como vimos, no caso do 4 e  10 isso não ocorre.
Justamente, o problema do caso do 4 e do 10 que "faz falhar o
teorema", é que 4 e 10 não são primos entre si, e daí "sobram fatores
primos", que são "contados duas vezes".

Uma sugestão para entender o teorema de Gauss: tente demonstrar sem
essa hipótese. Veja o que falha. Tente corrigir essa primeira
demonstração, sem usar Bézout. A idéia é simplesmente partir da
observação "bom, 3 | x e 2 | x => 6 | x", e tentar generalizar essa
demonstração (que é meio "fácil") para o caso em que 2 e 3 são números
quaisquer. Depois de um certo tempo, acho que vem "naturalmente" a
hipótese certa que falta adicionar ao teorema (que os números são
primos entre si). Mas o grande problema é que agora é praticamente
impossível de generalizar a demonstração que você deu no caso 2,3 =>
6. E essa é uma das partes interessantes (e difíceis) da matemática:
quando você começa a tentar entender mais profundamente alguma coisa,
e depois de um monte de esforço consegue chegar num enunciado que você
acredita suficientemente (ou seja, você esgotou a sua fonte de
contra-exemplos para a situação em questão, e os evitou nas hipóteses,
que capturam a "essência" do que deve fazer funcionar), você descobre
que falta alguma coisa mais forte para demonstrar. Você podia ter
começado com números primos somente (e nesse caso eu acho que tem uma
prova bem simples), mas agora, você precisa usar algo muito mais
sofisticado. E é exatamente entendendo a sofisticação que você
introduz (Bézout é crucial, mesmo, para demonstrar, e você vê
exatamente como ele faz "tudo funcionar"), que você terá entendido o
porquê de duas coisas. A primeira, porque o teorema é verdade, dito
assim. E a segunda, mais importante ainda, é "porque as hipóteses do
teorema são essas aí, e não outras".

E uma outra sugestão. Se você estiver com uma dúvida, escreva sempre
as hipóteses *antes* da conclusão. Escreva [ Se A, B, C e D, Então X
]. Ajuda a evitar erros, e simplifica também bastante na hora de
escrever a contrapositiva, a negação, ... Escrever enunciados de forma
"contorcida" pode ser mais elegante poeticamente, mas é mais ou menos
como usar numerais romanos para fazer multiplicações. Você conseguirá,
mas o algoritmo é muito menos eficiente do que o dos algarismos
arábicos.

Abraços,
--
Bernardo Freitas Paulo da Costa

=
Instruções para entrar na lista, sair da lista e usar a lista em
http://www.mat.puc-rio.br/~obmlistas/obm-l.html
=


[obm-l] RE: [obm-l] Re: [obm -l] dúvida sobre lem a de teoria dos núme ros.

2010-06-05 Por tôpico Lucas Hagemaister


 Hum... Entendi. Obrigado!

O que mais ou menos o lema quer dizer é o seguinte:

Sempre que termos m|a e n|a, onde mn|a, m e n serão primos entre si.

O que eu fiz foi o contrário(ali no caso do 4 e 10):

Sempre que termos m e n primos entre si, onde m|a e n|a, mn|a.

Como vimos, no caso do 4 e  10 isso não ocorre.  

 


From: bfr...@gmail.com
Date: Sat, 5 Jun 2010 13:46:07 -0300
Subject: [obm-l] Re: [obm-l] dúvida sobre lema de teoria dos números.
To: obm-l@mat.puc-rio.br

As hipóteses do lema são:

(1) m|a
(2) n|a
(3) (m, n) = 1 -- isto é, m e n são primos entre si

A tese é: (m*n) | a

Vc aplicou o lema inicialmente para o caso m = 4, n = 5, a = 20. Neste caso as 
3 hipóteses estão satisfeitas, então vale a tese: 4*5 | 20.

Depois, vc tentou aplicar o lema para o caso m = 4, n = 10, a = 20. Neste caso, 
a 3a. hipótese não é válida, então vc não pode aplicar o lema.

Se as hipóteses de uma proposição não estão satisfeitas, sua tese não tem 
significado nenhum.


A falha não está no lema.


Bruno
--
Bruno FRANÇA DOS REIS

msn: brunoreis...@hotmail.com
skype: brunoreis666
tel: +55 11 9961-7732

http://brunoreis.com
http://brunoreis.com/tech (en)
http://brunoreis.com/blog (pt)

GPG Key: http://brunoreis.com/bruno-public.key

e^(pi*i)+1=0



2010/6/5 Lucas Hagemaister 


Tem-se o lema:
 


Se  e  entao  quando ,  sao primos entre si.

 

Por exemplo,

m=4

n=5

a=20

 

4|20= está correto

5|20=também está correto

4.5|20= está correto, pois 20|20

 

Agora, se eu tenho:

m=4

n=10

a=20

 

4|20= já vimos

10|20= está correto, pois 20/10=2

Agora, temos o seguinte:

4.10=40

Então 4.10|20? 40|20? 40 divide 20?

 

Há alguma falha no lema ou há alguma falha minha?

 



GUARDE GRATUITAMENTE SEUS ARQUIVOS NA WEB. CLIQUE AQUI E VEJA UM PASSO A PASSO.
  
_
ACESSE O MESSENGER DO SEU CELULAR AGORA MESMO. CLIQUE E VEJA AQUI UM PASSO A 
PASSO.
http://celular.windowslive.com.br/messenger.asp?produto=Messenger&utm_source=Live_Hotmail&utm_medium=Tagline&utm_content=ACESSEOMES83&utm_campaign=MobileServices

[obm-l] Re: [obm-l] Re: [obm-l] Dúvida sobre estimadores te ndenciosos e não-tendenciosos

2009-11-16 Por tôpico Marcelo Salhab Brogliato
Olá, Ralph,
chegamos aos mesmos valores e conclusões.
Entendi perfeitamente o problema dos "100% de amostras 0", hehehe.

Muito obrigado pela resposta,
grande abraço,
Salhab



2009/11/16 Ralph Teixeira 

> Oi, Marcelo.
>
> Não sou expert nisso não, mas eu faria do mesmo jeito que você fez...
> Em primeiro, E(D)=0,65...
>
> Achei E(u1)=0,3106<0,65, então o estimador u1 é tendencioso.
> Achei E(u2)=0,7750>0,65, então o estimador u2 é tendencioso.
>
> Agora, "se D1>D2" introduz uma condição que faz com que E(D1-D2 |
> D1>D2) não seja 0... Você teria que calcular este E(D1-D2) usando
> APENAS os casos em que D1>D2, usando probabilidades condicionais (DADO
> D1>D2), e aí o bihco complica porque você está querendo fazer uma
> conta que não precise usar a distribuição... Seria algo assim:
>
> E(u2)=Pr(D1>D2)*E(D1-D2 | D1>D2) + Pr(D1=D2)*E(D1-D2 | D1=D2) +
> Pr(D1
> De qualquer forma, não faça isto: imagine que você tem uma população
> onde 100% das amostras dão 0. Para esta população, estes estimadores
> **não** serão tendenciosos... :) :) :)
>
> Abraço,
>  Ralph
>
> 2009/11/15 Marcelo Salhab Brogliato :
> > Fala pessoal,
> > to precisando de ajuda para provar se os seguintes estimadores são
> > tendenciosos ou não:
> > Tenho uma população com uma determinada propriedade que segue a seguinte
> > distribuição de probabilidade (p, v):
> > p=probabilidade
> > v=valor
> > (0 ; 0.5) , (1 ; 0.4) , (2 ; 0.05) , (3 ; 0.05)
> >
> > Seja (D1, D2) uma amostra da minha população.
> > u1 e u2 são estimadores da média da amostra.
> > i) u1 = sqrt(D1*D2)
> > ii) u2 = |D1-D2|
> >
> > O que eu fiz foi determinar a distribuição destes dois estimadores (basta
> > analisar todos os casos e determinar a probabilidade de ocorrência de
> cada
> > um dos valores possíveis).
> >
> > Feito isso, estou querendo analisar se estes estimadores são tendenciosos
> ou
> > não.
> > Para isso, determinei E(u1) e E(u2)... obtendo seus respectivos números.
> > Então, determinei a esperança da minha população e comparei os valores
> dos
> > estimadores.
> > Como são diferentes, conclui que estes estimadores são tendenciosos.
> > É isso mesmo?
> >
> > Minha outra dúvida é:
> > Como faço para provar que estes estimadores são tendenciosos para a média
> de
> > qualquer população?
> > Alias, isso é verdade?
> > Tentei calcular E(u1) = E(sqrt(D1*D2)) e travei!
> > Pensei em escrever u1 = (sqrt(D1)+sqrt(D2))^2 - D1 - D2 - sqrt(D1D2)
> > Então, 2u1 = [sqrt(D1)+sqrt(D2)]^2 - D1 - D2
> > 2E(u1) = E([sqrt(D1)+sqrt(D2)]^2) - 2(media da população)
> > Como trabalhar com este termo que "sobrou"?
> >
> > O mesmo para E(u2) = E(|D1-D2|)
> > Se D1 > D2, temos E(u2) = E(D1 - D2) = 0
> > Se D2 > D1, temos E(u2) = E(D2 - D1) = 0
> > Posso dizer que E(u2) = p(D1>D2) * E(D1-D2) + p(D2>D1) * E(D2-D1) ??
> > Meu sentimento é que não. hehehe
> >
> > obrigado por qualquer ajuda,
> > abraços,
> > Salhab
> >
> >
> >
> >
> >
> >
> >
>
> =
> Instruções para entrar na lista, sair da lista e usar a lista em
> http://www.mat.puc-rio.br/~obmlistas/obm-l.html
> =
>


[obm-l] Re: [obm-l] Re: [obm-l] Dúvida sobre limites

2009-10-28 Por tôpico Paulo Barclay Ribeiro
 Luiz.
Creio que o erro que cometi foi ter dividido o calculo desse limite em duas 
partes( produto dos limites 1+n/n+1 com sen(npi/2), nesta parte fiz: 
-1 escreveu:


De: Luiz Paulo 
Assunto: [obm-l] Re: [obm-l] Dúvida sobre limites
Para: obm-l@mat.puc-rio.br
Data: Quarta-feira, 28 de Outubro de 2009, 14:14







Podemos ver Tn da seguinte forma:
T(n)=1+1/[1+1/n]*sen(npi/2).
Tomando n=2k (k inteiro) vemos que daí teremos sen(kpi) que fica sendo zero.
Tomando n=2k+1(k inteiro) teremos sen[(2k+1)pi/2] que oscila entre -1 ou 1 
dependendo do k.
Daí tomando k tendendo ao infinito vemos que o termo em sen oscila entre esses 
valores.
Portanto T(n) diverge.

--- Em ter, 27/10/09, Paulo Barclay Ribeiro  
escreveu:


De: Paulo Barclay Ribeiro 
Assunto: [obm-l] Dúvida sobre limites
Para: obm-l@mat.puc-rio.br
Data: Terça-feira, 27 de Outubro de 2009, 18:35







pessoal , peço uma ajuda para esclarecer a seguinte questão;
 
A sucessão: T_n = 1+(n/n+1)*sin(npi/2) com n sendo um número natural é 
convergente ?
 
A achei que a sucessão acima é convergente , mas conversando,por alto, com um 
colega ele levantou a hipótese dessa sucessão ser divergente. Aí fiquei em 
dúvida.
Gostaria de uma orientação de vocês.
 
Obrigado, mais uma vez
 
Paulo


Veja quais são os assuntos do momento no Yahoo! + Buscados: Top 10 - 
Celebridades - Música - Esportes


Veja quais são os assuntos do momento no Yahoo! + Buscados: Top 10 - 
Celebridades - Música - Esportes


  

Veja quais são os assuntos do momento no Yahoo! +Buscados
http://br.maisbuscados.yahoo.com

[obm-l] Re: [obm-l] RE: [obm-l] Re: [obm-l] dúvida de inter pretação

2009-05-14 Por tôpico Marcelo Costa
*HUMILDEMENTE PEÇO DESCULPAS AOS MEMBROS DA LISTA POR FUGIR AOS PROPÓSITOS
DA MESMA, E AGRADEÇO A BOA VONTADE DO PROF. PALMERIM. DORAVANTE TOMAREI MAIS
CUIDADO AO APRESENTAR PROBLEMAS PARA QUE SEJAM PERTINENTES AOS PROPÓSITOS DA
LISTA. *

2009/5/12 Albert Bouskela 

>  Olá Palmerim,
>
>
>
> Obrigado pela citação!
>
>
>
> Sua resposta está correta e didática. Não obstante, vou pedir-lhe um favor:
> acho que deveríamos parar de elucidar dúvidas tais como a que foi
> apresentada pelo Marcelo. Acredito que seja prudente preservar o propósito
> desta Lista: a discussão de problemas, que sejam, em tese, pelo menos em
> potencial, pertinentes às Olimpíadas de Matemática. Neste sentido, é de todo
> conveniente que a Lista continue voltada para o objetivo fixado pelo Prof.
> Nicolau Saldanha: apoiar a preparação de estudantes para essas Olimpíadas.
> Caso esta Lista passe a ficar poluída por questões do tipo
> “tire-suas-dúvidas-elementares-em-matemática-básica”, certamente vai
> afugentar aqueles participantes que aqui estão para contribuir com o
> propósito original deste fórum. Além disto, estudantes a exemplo do Marcelo,
> podem valer-se de outros fóruns na Internet, bem mais apropriados, para
> sanar as suas dúvidas em Matemática Básica. Podem, outrossim, contar com os
> seus professores.
>
>
>
> *Albert Bouskela*
>
> bousk...@msn.com
>
>
>
> *From:* owner-ob...@mat.puc-rio.br [mailto:owner-ob...@mat.puc-rio.br] *On
> Behalf Of *Palmerim Soares
> *Sent:* Tuesday, May 12, 2009 11:32 AM
> *To:* obm-l@mat.puc-rio.br
> *Subject:* [obm-l] Re: [obm-l] dúvida de interpretação
>
>
>
> Olá Marcelo
>
>
>
> Numa fração os termos são necessariamente números inteiros. Mas uma fração
> pode representar inúmeras coisas: um número, uma divisão, uma RAZÃO etc. A
> Razão na verdade é uma comparação entre duas quantidades, feita por meio da
> divisão entre essas quantidades, as quais podem ser ou não números inteiros.
> Por exemplo, 2/5 é uma fração, e pode estar representando um número, uma
> divisão, e também uma RAZÃO, se estivermos, por exemplo, comparando a
> quantidade de mulheres numa festa em relação à quantidade de convidados.
> Neste caso, a RAZÃO 2/5 quer dizer que de cada 5 convidados, 2 são do sexo
> feminino. Ou em outras palavras, já que 2/5 = 40/100, 40% dos convidados são
> do sexo feminino. Veja que 40/100 é uma fração, mas neste exemplo é também
> uma razão. Então, 40% é uma FRAÇÂO centesimal (denominador igual a 100) e
> também é uma taxa de comparação e, neste sentido, é uma razão. Mas, para os
> puristas, estaria errado dizer que RAIZ(5)/3 é uma fração porque o numerador
> é irracional e não inteiro; pode estar representando uma RAZÃO, mas não é
> uma fração.
>
>
>
> Como diria o mestre Bouskela: Fui claro? :-)
>
>
>
> Abraços
>
> Palmerim
>
>
>
>
>
>
>
> 2009/5/12 Marcelo Costa 
>
> Seguinte:
>
>
>
> Pode-se afirmar que uma porcentagem é uma razão especial, uma razão em que
> o consequente é sempre igual a 100 ?!
>
> Se sim, por ex., 25 % = 25/100 = ¼, não é ?!
>
> Posso ler então, como sendo razão de um para quatro. Está correto ?!
>
> Nesse caso, são 5 partes no total (1 + 4). Onde está a confusão ou o erro
> de interpretação ?!
>
>
>
> Acho que posso afirmar que uma porcentagem é uma fração (e não uma razão)
> em que o denominador é 100
>
>
>
> Por ex,:
>
>
>
> Fração ¼ significa uma parte em quatro.
>
> Razão ¼ significa uma parte para quatro, perfazendo um total de cinco
> partes...
>
>
>
> Favor comentar
>
>
>
>
>
>
>
>
>
>
> --
> "Matemática é o alfabeto com o qual Deus escreveu o Universo"
> Galileu Galilei
>
>
>
>
> --
> Palmerim
>



-- 
"Matemática é o alfabeto com o qual Deus escreveu o Universo"
Galileu Galilei


[obm-l] RE: [obm-l] Re: [obm-l] dúvida d e interpretação

2009-05-12 Por tôpico Albert Bouskela
Olá Palmerim,

 

Obrigado pela citação!

 

Sua resposta está correta e didática. Não obstante, vou pedir-lhe um favor: 
acho que deveríamos parar de elucidar dúvidas tais como a que foi apresentada 
pelo Marcelo. Acredito que seja prudente preservar o propósito desta Lista: a 
discussão de problemas, que sejam, em tese, pelo menos em potencial, 
pertinentes às Olimpíadas de Matemática. Neste sentido, é de todo conveniente 
que a Lista continue voltada para o objetivo fixado pelo Prof. Nicolau 
Saldanha: apoiar a preparação de estudantes para essas Olimpíadas. Caso esta 
Lista passe a ficar poluída por questões do tipo 
“tire-suas-dúvidas-elementares-em-matemática-básica”, certamente vai afugentar 
aqueles participantes que aqui estão para contribuir com o propósito original 
deste fórum. Além disto, estudantes a exemplo do Marcelo, podem valer-se de 
outros fóruns na Internet, bem mais apropriados, para sanar as suas dúvidas em 
Matemática Básica. Podem, outrossim, contar com os seus professores. 

 

Albert Bouskela

bousk...@msn.com

 

From: owner-ob...@mat.puc-rio.br [mailto:owner-ob...@mat.puc-rio.br] On Behalf 
Of Palmerim Soares
Sent: Tuesday, May 12, 2009 11:32 AM
To: obm-l@mat.puc-rio.br
Subject: [obm-l] Re: [obm-l] dúvida de interpretação

 

Olá Marcelo

 

Numa fração os termos são necessariamente números inteiros. Mas uma fração pode 
representar inúmeras coisas: um número, uma divisão, uma RAZÃO etc. A Razão na 
verdade é uma comparação entre duas quantidades, feita por meio da divisão 
entre essas quantidades, as quais podem ser ou não números inteiros. Por 
exemplo, 2/5 é uma fração, e pode estar representando um número, uma divisão, e 
também uma RAZÃO, se estivermos, por exemplo, comparando a quantidade de 
mulheres numa festa em relação à quantidade de convidados. Neste caso, a RAZÃO 
2/5 quer dizer que de cada 5 convidados, 2 são do sexo feminino. Ou em outras 
palavras, já que 2/5 = 40/100, 40% dos convidados são do sexo feminino. Veja 
que 40/100 é uma fração, mas neste exemplo é também uma razão. Então, 40% é uma 
FRAÇÂO centesimal (denominador igual a 100) e também é uma taxa de comparação 
e, neste sentido, é uma razão. Mas, para os puristas, estaria errado dizer que 
RAIZ(5)/3 é uma fração porque o numerador é irracional e não inteiro; pode 
estar representando uma RAZÃO, mas não é uma fração.

 

Como diria o mestre Bouskela: Fui claro? :-)

 

Abraços 

Palmerim

 

 

 

2009/5/12 Marcelo Costa 

Seguinte: 

 

Pode-se afirmar que uma porcentagem é uma razão especial, uma razão em que o 
consequente é sempre igual a 100 ?!

Se sim, por ex., 25 % = 25/100 = ¼, não é ?! 

Posso ler então, como sendo razão de um para quatro. Está correto ?!

Nesse caso, são 5 partes no total (1 + 4). Onde está a confusão ou o erro de 
interpretação ?!

 

Acho que posso afirmar que uma porcentagem é uma fração (e não uma razão) em 
que o denominador é 100

 

Por ex,:

 

Fração ¼ significa uma parte em quatro. 

Razão ¼ significa uma parte para quatro, perfazendo um total de cinco partes...

 

Favor comentar

 

 

 






-- 
"Matemática é o alfabeto com o qual Deus escreveu o Universo"
Galileu Galilei




-- 
Palmerim



[obm-l] Re: [obm-l] Re: [obm-l] Dúvida em questão de R aciocínio CORRIGINDO

2009-02-03 Por tôpico João Luís
hahahahahhahahahahhaha

tá certo seu puxão de orelha Nehab.

Mas convenhamos, não seria bem melhor se cada questão que fosse postada aqui 
viesse com os dados de sua origem?

Abração a todos,

João Luís.
  - Original Message - 
  From: Carlos Nehab 
  Cc: obm-l@mat.puc-rio.br 
  Sent: Tuesday, February 03, 2009 11:39 AM
  Subject: Re: [obm-l] Re: [obm-l] Dúvida em questão de Raciocínio CORRIGINDO 


  Corrigindo:

  Foi num teste de um curso preparatório para a Anpad.  
  João, deixa de ser preguiçoso e acredite no Google... :-)   
  Foi o que fiz...

  Nehab

  Carlos Nehab escreveu: 
Oi, João,

Foi numa prova da ANPAD.  Em geral boas questões...


http://www.anpadcurso.com/provas_anteriores/rl/PROVA_TESTE_ANPAD_RL-fev_08.pdf

Abraços,
Nehab


João Luís escreveu: 
  Não fiz ainda, mas creio que é uma simples questão de considerar cada 
fazenda como sendo um conjunto cujos elementos são o tipo de animal que é 
criado lá. Então afirmativas como "os animais comuns às fazendas Alfa e Beta 
são somente cães e bois" ,  quer dizer que a intersecção dos conjuntos Alfa e 
Beta é o conjunto {cães, bois}.

  Monte o diagrama de Venn para 3 conjuntos, vá preenchendo as regiões com 
as informações de I a VI, que eu acho que sai fácil.

  Uma pergunta:de onde é essa questão? vestibular? concurso? que 
instituição? e que ano?

  Um abraço,

  João Luís.
- Original Message - 
From: Marcos Xavier 
To: OBM 
Sent: Monday, February 02, 2009 4:31 PM
Subject: [obm-l] Dúvida em questão de Raciocínio


Amigos, estou com dificuldade na seguinte questão:




Em um país, há três fazendas: Alfa, Beta e Gama. Sabe-se que nessas 
fazendas criam-se somente animais, e também que,


I.se reunirmos os animais das três fazendas, teremos porcos, 
galinhas, cães, gatos, bois, ovelhas, cavalos, chinchilas e coelhos;

II.   os animais comuns às fazendas Alfa e Beta são somente cães e 
bois;

III.  os animais comuns às fazendas Beta e Gama são somente cães e 
cavalos; 

IV. os animais comuns às fazendas Alfa e Gama são somente cães e 
ovelhas; 

V.  se reunirmos os animais presentes nas fazendas Alfa e Gama, 
então ficaremos com galinhas, cães, gatos, bois, ovelhas, cavalos, chinchilas e 
coelhos; e

VI. reunirmos os animais presentes nas fazendas Alfa e Beta, então 
teremos porcos, cães, gatos, bois, ovelhas, cavalos e galinhas.


Logo, pode-se afirmar que:


A)  a fazenda Alfa abriga apenas bois, cães e. ovelhas;

B)  a fazenda Beta abriga apenas bois, cães e cavalos;

C)  a fazenda Beta abriga bois, cães, porcos e cavalos;


D)  as outras fazendas juntas abrigam um menor número de espécies 
de animais que a fazenda Alfa;

E)  bois e galinhas vivem apenas na fazenda Alfa.




A resposta é Letra (C).




Agradeço a todos.




Marcos.





Receba GRÁTIS as mensagens do Messenger no seu celular quando você 
estiver offline. Conheça o MSN Mobile! Crie já o seu! 
  = 
Instruções para entrar na lista, sair da lista e usar a lista em 
http://www.mat.puc-rio.br/~obmlistas/obm-l.html 
=

[obm-l] Re: [obm-l] Re: [obm-l] Dúvida em questão de R aciocínio

2009-02-03 Por tôpico João Luís
Obrigado Nehab
  - Original Message - 
  From: Carlos Nehab 
  To: obm-l@mat.puc-rio.br 
  Sent: Tuesday, February 03, 2009 11:27 AM
  Subject: Re: [obm-l] Re: [obm-l] Dúvida em questão de Raciocínio


  Oi, João,

  Foi numa prova da ANPAD.  Em geral boas questões...

  http://www.anpadcurso.com/provas_anteriores/rl/PROVA_TESTE_ANPAD_RL-fev_08.pdf

  Abraços,
  Nehab


  João Luís escreveu: 
Não fiz ainda, mas creio que é uma simples questão de considerar cada 
fazenda como sendo um conjunto cujos elementos são o tipo de animal que é 
criado lá. Então afirmativas como "os animais comuns às fazendas Alfa e Beta 
são somente cães e bois" ,  quer dizer que a intersecção dos conjuntos Alfa e 
Beta é o conjunto {cães, bois}.

Monte o diagrama de Venn para 3 conjuntos, vá preenchendo as regiões com as 
informações de I a VI, que eu acho que sai fácil.

Uma pergunta:de onde é essa questão? vestibular? concurso? que instituição? 
e que ano?

Um abraço,

João Luís.
  - Original Message - 
  From: Marcos Xavier 
  To: OBM 
  Sent: Monday, February 02, 2009 4:31 PM
  Subject: [obm-l] Dúvida em questão de Raciocínio


  Amigos, estou com dificuldade na seguinte questão:




  Em um país, há três fazendas: Alfa, Beta e Gama. Sabe-se que nessas 
fazendas criam-se somente animais, e também que,


  I.se reunirmos os animais das três fazendas, teremos porcos, 
galinhas, cães, gatos, bois, ovelhas, cavalos, chinchilas e coelhos;

  II.   os animais comuns às fazendas Alfa e Beta são somente cães e 
bois;

  III.  os animais comuns às fazendas Beta e Gama são somente cães e 
cavalos; 

  IV. os animais comuns às fazendas Alfa e Gama são somente cães e 
ovelhas; 

  V.  se reunirmos os animais presentes nas fazendas Alfa e Gama, então 
ficaremos com galinhas, cães, gatos, bois, ovelhas, cavalos, chinchilas e 
coelhos; e

  VI. reunirmos os animais presentes nas fazendas Alfa e Beta, então 
teremos porcos, cães, gatos, bois, ovelhas, cavalos e galinhas.


  Logo, pode-se afirmar que:


  A)  a fazenda Alfa abriga apenas bois, cães e. ovelhas;

  B)  a fazenda Beta abriga apenas bois, cães e cavalos;

  C)  a fazenda Beta abriga bois, cães, porcos e cavalos;


  D)  as outras fazendas juntas abrigam um menor número de espécies de 
animais que a fazenda Alfa;

  E)  bois e galinhas vivem apenas na fazenda Alfa.




  A resposta é Letra (C).




  Agradeço a todos.




  Marcos.




--
  Receba GRÁTIS as mensagens do Messenger no seu celular quando você 
estiver offline. Conheça o MSN Mobile! Crie já o seu! 
  = 
Instruções para entrar na lista, sair da lista e usar a lista em 
http://www.mat.puc-rio.br/~obmlistas/obm-l.html 
=

[obm-l] Re: [obm-l] Re: [obm-l] Dúvida em questão de Racio cínio

2009-02-03 Por tôpico Ralph Teixeira
Fiz como o João Luís falou, deu certo: faça um diagrama de Venn,
preencha "de dentro para fora". Chamando os conjuntos de A, B e C,
(chamo "e" de interseção, "ou" de união):

A e B e C = (A e B) e (A e C) = {Cão} (usando II e IV)
A e B e (não C) = (A e B) - (A e B e C) = {Boi} (II)
A e (não B) e C = (A e C) - (A e B e C) = {Ove} (IV)
(não A) e B e C = (B e C) - (A e B e C) = {Cav} (III)
(não A) e B e (não C) = tudo - (A ou C) = {Por} (I e V)
(não A) e (não B) e C = tudo - (A ou B) = {Chi, Coe} (I e VI)
A e (não B) e (não C) = o que sobrou = {Gat, Gal}

Resposta (c). Tá certo.

Abraço,
  Ralph

2009/2/3 João Luís :
> Não fiz ainda, mas creio que é uma simples questão de considerar cada
> fazenda como sendo um conjunto cujos elementos são o tipo de animal que é
> criado lá. Então afirmativas como "os animais comuns às fazendas Alfa e Beta
> são somente cães e bois" ,  quer dizer que a intersecção dos conjuntos Alfa
> e Beta é o conjunto {cães, bois}.
>
> Monte o diagrama de Venn para 3 conjuntos, vá preenchendo as regiões com as
> informações de I a VI, que eu acho que sai fácil.
>
> Uma pergunta:de onde é essa questão? vestibular? concurso? que instituição?
> e que ano?
>
> Um abraço,
>
> João Luís.
>
> - Original Message -
> From: Marcos Xavier
> To: OBM
> Sent: Monday, February 02, 2009 4:31 PM
> Subject: [obm-l] Dúvida em questão de Raciocínio
>
> Amigos, estou com dificuldade na seguinte questão:
>
> Em um país, há três fazendas: Alfa, Beta e Gama. Sabe-se que nessas fazendas
> criam-se somente animais, e também que,
>
> I.se reunirmos os animais das três fazendas, teremos porcos,
> galinhas, cães, gatos, bois, ovelhas, cavalos, chinchilas e coelhos;
>
> II.   os animais comuns às fazendas Alfa e Beta são somente cães e bois;
>
> III.  os animais comuns às fazendas Beta e Gama são somente cães e
> cavalos;
>
> IV. os animais comuns às fazendas Alfa e Gama são somente cães e
> ovelhas;
>
> V.  se reunirmos os animais presentes nas fazendas Alfa e Gama, então
> ficaremos com galinhas, cães, gatos, bois, ovelhas, cavalos, chinchilas e
> coelhos; e
>
> VI. reunirmos os animais presentes nas fazendas Alfa e Beta, então
> teremos porcos, cães, gatos, bois, ovelhas, cavalos e galinhas.
>
> Logo, pode-se afirmar que:
>
> A)  a fazenda Alfa abriga apenas bois, cães e. ovelhas;
>
> B)  a fazenda Beta abriga apenas bois, cães e cavalos;
>
> C)  a fazenda Beta abriga bois, cães, porcos e cavalos;
>
> D)  as outras fazendas juntas abrigam um menor número de espécies de
> animais que a fazenda Alfa;
>
> E)  bois e galinhas vivem apenas na fazenda Alfa.
>
> A resposta é Letra (C).
>
> Agradeço a todos.
>
> Marcos.
>
> 
> Receba GRÁTIS as mensagens do Messenger no seu celular quando você estiver
> offline. Conheça o MSN Mobile! Crie já o seu!

=
Instruções para entrar na lista, sair da lista e usar a lista em
http://www.mat.puc-rio.br/~obmlistas/obm-l.html
=


[obm-l] RES: [obm-l] Re: [obm-l] Re: [obm-l] Dúvida com questão

2008-09-16 Por tôpico Bouskela
E provar que x^2+x+1 é maior do que 0 é muito fácil:
 
1] x^2+x+1 não tem raízes reais. Logo a parábola x^2+x+1 está totalmente
acima OU ("ou" exclusivo) totalmente abaixo do eixo X. Como 0^2+0+1>0 ,
então a parábola está totalmente acima do eixo X .
 
[EMAIL PROTECTED]
[EMAIL PROTECTED]   

 


  _  

De: [EMAIL PROTECTED] [mailto:[EMAIL PROTECTED] Em nome
de João Luís
Enviada em: terça-feira, 16 de setembro de 2008 13:01
Para: obm-l@mat.puc-rio.br
Assunto: [obm-l] Re: [obm-l] Re: [obm-l] Dúvida com questão


Sim, é verdade. Ficou incompleto mesmo.
 
O que acontece é que eu quis enfatizar que, independentemente da
incompletude do enunciado, a bicondicional dada será falsa. E, com isso,
acabei me esquecendo do sinal do termo quadrático.
 
Obrigado pela observação, Bouskela.

- Original Message - 
From: Bouskela <mailto:[EMAIL PROTECTED]>  
To: obm-l@mat.puc-rio.br 
Sent: Tuesday, September 16, 2008 11:46 AM
Subject: Re: [obm-l] Re: [obm-l] Dúvida com questão

João Luís:
 
Sua solução está correta! Entretanto repare que ela (sua solução) está
correta apenas porque "x^2+x+1" é positivo (maior do que "0") para qualquer
que seja "x" real. É, então, necessário, no âmbito da sua solução,
demonstrar isto:
 
x^2+x+1 > 0 para qualquer que seja "x" real.
 
Sds.,
AB 


2008/9/16 João Luís <[EMAIL PROTECTED]>


Bom, faltou um símbolo de desigualdade no primeiro membro (antecedente)
dessa bicondicional: [(x^2+x+1)/(x-2)] 3. Mas de qualquer forma, a
afirmativa é falsa, já que o sinal da desigualdade vai mudar de sentido
conforme o sinal do termo de primeiro grau (x-2): se (x-2) < 0, a segunda
desigualdade será o contrário da primeira; se (x-2) > 0, será igual à
primeira
 
Um abraço,
 

João Luís.

- Original Message - 
From: Robÿe9rio Alves <mailto:[EMAIL PROTECTED]>  
To: obm-l@mat.puc-rio.br 
Sent: Tuesday, September 16, 2008 10:05 AM
Subject: [obm-l] Dúvida com questão


A afirmação: para todo x real, x diferente de 2,[( x^2+x+1) / (x - 2 )]
3  <=> x^2+x+1 > 3.(x - 2 ) é verdaeira ou falsa ? Justifique.
 
Como resolver ???
 

  _  

Novos endereços, o Yahoo! que você conhece. Crie um email novo
<http://br.rd.yahoo.com/mail/taglines/mail/*http://br.new.mail.yahoo.com/add
resses>  com a sua cara @ymail.com <http://ymail.com/>  ou @rocketmail.com
<http://rocketmail.com/> .




-- 
Saudações,
AB
[EMAIL PROTECTED]
[EMAIL PROTECTED]




[obm-l] Re: [obm-l] Re: [obm-l] Dúvida com questão

2008-09-16 Por tôpico João Luís
Sim, é verdade. Ficou incompleto mesmo.

O que acontece é que eu quis enfatizar que, independentemente da incompletude 
do enunciado, a bicondicional dada será falsa. E, com isso, acabei me 
esquecendo do sinal do termo quadrático.

Obrigado pela observação, Bouskela.
  - Original Message - 
  From: Bouskela 
  To: obm-l@mat.puc-rio.br 
  Sent: Tuesday, September 16, 2008 11:46 AM
  Subject: Re: [obm-l] Re: [obm-l] Dúvida com questão


  João Luís:

  Sua solução está correta! Entretanto repare que ela (sua solução) está 
correta apenas porque "x^2+x+1" é positivo (maior do que "0") para qualquer que 
seja "x" real. É, então, necessário, no âmbito da sua solução, demonstrar isto:

  x^2+x+1 > 0 para qualquer que seja "x" real.

  Sds.,
  AB 


  2008/9/16 João Luís <[EMAIL PROTECTED]>

Bom, faltou um símbolo de desigualdade no primeiro membro (antecedente) 
dessa bicondicional: [(x^2+x+1)/(x-2)] 3. Mas de qualquer forma, a afirmativa é 
falsa, já que o sinal da desigualdade vai mudar de sentido conforme o sinal do 
termo de primeiro grau (x-2): se (x-2) < 0, a segunda desigualdade será o 
contrário da primeira; se (x-2) > 0, será igual à primeira

Um abraço,

João Luís.
  - Original Message - 
  From: Robÿe9rio Alves 
  To: obm-l@mat.puc-rio.br 
  Sent: Tuesday, September 16, 2008 10:05 AM
  Subject: [obm-l] Dúvida com questão


A afirmação: para todo x real, x diferente de 2,[( x^2+x+1) / 
(x - 2 )]  3  <=> x^2+x+1 > 3.(x - 2 ) é verdaeira ou falsa ? Justifique.

Como resolver ???
   


--
  Novos endereços, o Yahoo! que você conhece. Crie um email novo com a sua 
cara @ymail.com ou @rocketmail.com.



  -- 
  Saudações,
  AB
  [EMAIL PROTECTED]
  [EMAIL PROTECTED]


[obm-l] Re: [obm-l] RE: [obm-l] Dúvida em trigonomet ria e nos Complexos

2008-07-22 Por tôpico Paulo Mello
Victor, valew!
Vou aplicar as sua dicas e resolver os problemas. 
Muito obrigado pela sua atenção.
 
Um grande abraço.
 
Paulo Mello.
=
--- Em ter, 22/7/08, [EMAIL PROTECTED] <[EMAIL PROTECTED]> escreveu:

De: [EMAIL PROTECTED] <[EMAIL PROTECTED]>
Assunto: [obm-l] RE: [obm-l] Dúvida em trigonometria e nos Complexos
Para: obm-l@mat.puc-rio.br, obm-l@mat.puc-rio.br
Data: Terça-feira, 22 de Julho de 2008, 15:59

Olá  Paulo,

1) Para  o primeiro , você pode usar  a relação para o sen2x  e  desenvolver
; no entanto  acredito ficar mais  simples  se utilizar a relaçãosen2x
= 2t/(1+t^2)
onde  t =tanx  e  estudar a  desigualdade , ok ?

2) para o  segundo , pense  assim :no plano  de Argand-Gauss , o lugar  de
z é uma  circunferência  de  centro (2,0)  e  raio  1 .Estude o menor  e
o maior  argumento  de  z , analisando  os pontos  sobre  a  circunferência,
ok ?

3) Para  o terceiro,faça o seguinte :divida tudo  por  2. Do lado esquerdo
ficará  o  cos[(pi/3) +3x] e  do  lado  direito ficará igual  a sqrt(2)/2
 que é igual  ao cos(pi/4) .Daí é só resolver  a  equação 
trigonométrica
 simples cosa=cosb ,ok ?. 

Abraços 

Carlos Victor


 '>'-- Mensagem Original --
 '>'Date: Tue, 22 Jul 2008 07:51:42 -0700 (PDT)
 '>'From: Paulo Mello <[EMAIL PROTECTED]>
 '>'Subject: [obm-l] Dúvida em trigonometria e nos Complexos
 '>'To: obm-l@mat.puc-rio.br
 '>'Reply-To: obm-l@mat.puc-rio.br
 '>'
 '>'
 '>' 
 '>' 
 '>'pessoal, bom dia.
 '>'peço orientação para resolver os seguintes problemas.
 '>'1) Resolver a inequação tan(x)-sen(2x)>0 em [-pi;+pi].
 '>'2)Sendo Q o conjunto dos números complexos z tais que
|z-2|=1.calcule
o elemento
 '>'de Q que possua o menor argumento possível.
 '>' 
 '>'Obs: Q não representa conjunto dos racionais.
 '>' 
 '>'3) Resolva a equação: cos(3x)- (raizde 3)sen(3x) = raizde 2.
 '>' 
 '>'Desde já agradeço a atenção.
 '>' 
 '>'Paulo Mello
 '>' 
 '>'
 '>'
 '>'  Novos endereços, o Yahoo! que você conhece. Crie um
email novo
com
 '>'a sua cara @ymail.com ou @rocketmail.com.
 '>'http://br.new.mail.yahoo.com/addresses




=
Instruções para entrar na lista, sair da lista e usar a lista em
http://www.mat.puc-rio.br/~obmlistas/obm-l.html
=


  Novos endereços, o Yahoo! que você conhece. Crie um email novo com a sua 
cara @ymail.com ou @rocketmail.com.
http://br.new.mail.yahoo.com/addresses

Re: [obm-l] Re: [obm-l] Re:[obm-l] dúvida (Q uadriláteros)

2006-05-05 Por tôpico cleber vieira
Valeu claudio, a idéia de fazer DO = OX  e daí provar que X coincide com H foi um xeque-mate no problema, parabéns e muito obrigado pela sua resolução.  Abraços  Cleber 
		 
Abra sua conta no Yahoo! Mail - 1GB de espaço, alertas de e-mail no celular e anti-spam realmente eficaz. 

[obm-l] Re: [obm-l] RE: [obm-l] dúvida fatorial

2006-04-12 Por tôpico Ronaldo Luiz Alonso
Um conjunto A de funcoes analiticas, duas a duas distintas,  definidas em 
C ( C e o conjunto dos numeros complexos )  tal que para cada z 
pertencente a C fixado,  o conjunto { f(z), f variando em A}  seja 
enumeravel.


Pergunto : A e um conjunto enumeravel ?

E respondo propondo um exercicio :
Prove que se a HIPOTESE DO CONTINUO e falsa, entao qualquer A sera sempre 
um conjunto enumeravel. Se, porem, a HIPOTESE DO CONTINUO e verdadeira, 
entao existe ao menos um conjunto A não enumeravel.


 Parece uma afirmação legal.   Se a imagem da função é um conjunto 
enumerável então #f(C) = ALEPH0 e a função tem
que ter um número infinito de descontinuidades se a hipótese for verdadeira. 
Logo a quantidade o número de funções
é enumerável e por conseguinte A é enumerável. Se for falsa, é possível 
produzir um conjunto entre ALEPH0 e c e portanto o

número de tais funcões é não enumerável.

Um vazio de afinidades parece ser algo diferente de um vazio de 
cardinalidade. Existiria assim dois conjuntos vazios. Para um dado 
conjunto, existiria o seu numero cardinal e o conjunto das afinidades de 
seus elementos. Exemplo :


conjunto A ={joao,maria}
Afinidades associadas a A ={{ mesma idade, mesma cor, mesma altura }}

conjunto B={pedro,marta}
Afinidades associadas a B = {{ }}( sem afinidades )



Parece interessante.  Os conjuntos neste caso são definidos por propriedades 
intrínsecas

que definem seus elementos
que poderiam ser propriedades topológicas (por exemplo).

No caso do conjunto {joao, maria}
os elementos guardam entre si uma relação de equivalência topológica (mesma 
idade, mesma cor,
mesma altura).  No caso dos naturais e racionais a propriedade topológica 
seria que qualquer ponto
pode ser isolado de outro ponto por uma bola aberta (ou que poderíamos ser 
capazes de

identificar os elementos desse conjunto, "pescando-os com uma agulha")

   No caso dos reais essa propriedade topológica não é verificada: Não é 
sempre possível "pegar

um número real com uma agulha na reta", o que faz com que o conjunto
seja não enumerável.  Mas não é tão simples.  Existem outras propriedades 
(afinidades) que não estamos

identificando nos reais que dizem respeito à cardinalidade desses conjuntos?


Seria necessario agora construir um conjunto de axiomas que regulassem a 
relacao entre estas duas categorias de conjuntos : os conjuntos e suas 
afinidades.  Mas e certo que poderiamos passar a interpretar 
combinatoriamente :


BINOM(N,0)=2


Tah... mas as afinidades formam um conjunto.  O que dizer então da afinidade 
do conjunto de afinidades?





Esta mensagem toca em assuntos dificeis e importantes, mas e absolutamente 
despretensiosa



  É assim que as coisas começam.  Sempre tiramos coisas úteis, por mais 
besta e idiota que a discussão pareça.
Geralmente na cantina do instituto é que surgem as grandes idéias.   Confira 
o link:


http://ctjovem.mct.gov.br/index.php?action=/content/view&cod_objeto=14986

Abraço a todos.
Gandhi. 


=
Instruções para entrar na lista, sair da lista e usar a lista em
http://www.mat.puc-rio.br/~nicolau/olimp/obm-l.html
=


[obm-l] Re: [obm-l] Re: [obm-l] dúvida fatorial

2006-04-04 Por tôpico Ronaldo Luiz Alonso
Qualquer valor diferente de "um" atribuído por "convenção" estaria negando 
a definição de fatorial.


SE considerarmos  a interpretação de fatorial
como número de bijeções de um conjunto com n
elementos em um conjunto com n elementos
e SE considerarmos a definição de números
binomiais em termos de fatorial como usualmente
nos são apresentadas, aí podemos dizer que o que
vc escreveu está correto. Não há como trocar
a definição sem causar conflitos.

Mas todas essas definições são, de fato, convenções.
Então a definição 0! = 1 também (do mesmo jeito)
é uma convenção.
Não consigo ver como não seria com o que
nos foi apresentado até agora  ...




Ojesed.

- Original Message - 
From: "Nicolau C. Saldanha" <[EMAIL PROTECTED]>

To: 
Sent: Monday, April 03, 2006 4:19 PM
Subject: Re: [obm-l] dúvida fatorial


On Mon, Apr 03, 2006 at 09:49:58AM -0300, reginaldo.monteiro wrote:

Alguém saberia me informar por que 0! = 1?


Alguém já respondeu corretamente que isto é uma convenção,
mas acho que há mais para ser dito.

A interpretação combinatória para n! é que este é o número
de permutações de um conjunto A com n elementos. Recapitulando,
uma permutação de A é uma função bijetora f:A->A, ou,
equivalentemente, um subconjunto F de AxA (o gráfico de f)
tal que, para todo a em A:
* existe um único b em A tal que (a,b) pertence a F;
* existe um único c em A tal que (c,a) pertence a F.

Com esta definição, se A = 0 (vazio) então F = 0 é o gráfico
de uma bijeção f:A->A, a função vazia. As condições para verificar
que f é bijetora são satisfeitas por vacuidade. É bem claro
que esta é a única permutação de A, donde 0!=1.

[]s, N.
=
Instruções para entrar na lista, sair da lista e usar a lista em
http://www.mat.puc-rio.br/~nicolau/olimp/obm-l.html
=


--
No virus found in this incoming message.
Checked by AVG Free Edition.
Version: 7.1.385 / Virus Database: 268.3.4/299 - Release Date: 31/3/2006


=
Instruções para entrar na lista, sair da lista e usar a lista em
http://www.mat.puc-rio.br/~nicolau/olimp/obm-l.html
=



=
Instruções para entrar na lista, sair da lista e usar a lista em
http://www.mat.puc-rio.br/~nicolau/olimp/obm-l.html
=


[obm-l] Re: [obm-l] RE: [obm-l] dúvida fatorial

2006-04-03 Por tôpico Ronaldo Luiz Alonso

Eu havia imaginado vagamente (a tempos atrás)
tudo o que o professor Paulo
colocou nesta mensagem (Show de Bola).
  Só que não tinha exemplos concretos nem clareza
de idéias e também nem citações suficientes
para explicitá-las como as que foram por ele colocadas.

   A moral disso tudo é que devemos sempre
QUESTIONAR aquilo que nos é ensinado e da maneira
como é ensidado, pois podemos frequentemente nos
deparar com situações práticas onde a teoria precisa
ser ligeiramente adaptada e/ou a interpretação IPSIS
LITERIS da teoria pode tornar inviável a sua aplicação.
 O caso das geometrias não euclidianas são um exemplo
prático deste caso.


"WHAT I CAN'T CREATE I CAN'T UNDERSTAND"
-- RICHARD FEYNMAN.



- Original Message - 
From: "Paulo Santa Rita" <[EMAIL PROTECTED]>

To: 
Sent: Monday, April 03, 2006 12:29 PM
Subject: [obm-l] RE: [obm-l] dúvida fatorial



Ola Reginaldo e demais
colegas desta lista ... OBM-L,
( estou escrevendo sem acentos )

Porque e conveniente ... Este postulado ( 0!=1 ) e consistente com outras 
crencas e as implicacoes dele sao uteis na pratica. De fato :


1) ACREDITANDO que a FUNCAO GAMA e a generalizacao do conceito de 
fatorial, e possivel DEDUZIR este postulado.
2) Admitindo este postulado, calculos combinatorios com numeros binomiais 
ficam mais faceis, sinteticos e concordam com a nossa PRATICA habitual de 
contar.


Entretanto :

3) Nao existe nenhuma prova ou razao apoditica de que a funcao gama seja A 
GENERALIZACAO do conceito de fatorial. Muito provavelmente ela e UMA 
GENERALIZACAO, sem nenhuma vantagem logica sobre outras potencialmente 
possiveis ( e potencialmente interessantes ! )
4) Em Matematica, o fato da pratica confirmar a teoria nao e uma razao 
definitiva para nao modificarmos esta teoria. Basta lembrar do nascimento 
das Geometrias nao-euclidianas ...


Quando voce olha para um numero binomial BINOM(N,P) e o define assim :

BINOM(N,P) = N! / ( P! * (N-P)! )

Voce, em verdade, esta ADMITINDO PREVIAMENTE A DEFINICAO DE FATORIAL e 
definindo uma funcao a duas variaveis com certas restricoes. Mas voce 
tambem poderia fazer assim :


Defino :

BINOM(N,0)=1 onde N=0,1,2,...
BINOM(N+1,P)=BINOM(0,P-1) + ... + BINOM(N,P-1) onde P=1,2,...

Neste caso voce vai obter OS MESMOS RESULTADOS sem lancar mao de uma 
definicao previa de fatorial. Mas, o que nos impede de definir :


BINOM(N,0)=2 onde N=0,1,2, ...
BINOM(N+1,P)=BINOM(0,P-1) + ... + BINOM(N,P-1) onde P=1,2,...

No primeiro caso voce obtem o bem conhecido TRIANGULO DE PASCAL, que 
chamaremos doravante de PASCAL. No segundo caso, um "PASCAL DOBRADO" ou 
2*PASCAL :


2
2 2
2 4 2
2 6 6 2
...

E no entanto, o 2*PASCAL mantem formalmente as mesmas propriedades basicas 
( Ex : Teorema das Colunas ) do triangulo tradicional, conforme se ve 
facilmente ...


Quanto vale 0! no 2*PASCAL ?

BINOM(N,0) = 2 = N! / (0! * (N-0)! ) => 0!=1/2

Veja que agora temos maior liberdade. Nao somos mais escravos da 
postulacao 0!=1


Evidentemente, no 2*PASCAL,  nao podemos mais interpretar BINOM(N,P) como 
o numero de combinacoes com P elementos que podemos formar se dispormos de 
N elementos. Bom, isso e decisivo ? E o noumeno sagrado que nao se pode 
tocar ? Sera que o 2*PASCAL nao nos permite fazer interpretacoes 
igualmente interessantes ?


Para um K*PASCAL, defino : NIC=1/0! . Assim, o triangulo de pascal e o 
1*PASCAL=PASCAL com NIC=1. Para cada real NIC ha um triangulo bem 
determinado com potenciais interpretacoes nao menos interessantes.


Finalmente, me permito uma digressao. Pode-se definir a sequencia de 
fibonaci pelo triangulo de pascal. Basta partir da coluna zero e "subir" 
em diagonal, somando os numeros binomiais. Obteremos assim esta sequencia 
tao conhecida.


( Veja : Um ensaio sobre a beleza na Matematica - Huntley - Editora Univ. 
de Brasilia )


Se fizermos NIC=fi, fi = LIM f(n+1)/f(n) onde f(n) e o enesimo termo da 
sequencia de fibonaci, qual e o "triangulo tipo Pascal" correspondente ?


Um Abraco a Todos
Paulo Santa Rita
2,1225,030406





  From: reginaldo.monteiro
  To: obm-l
  Sent: Monday, April 03, 2006 9:49 AM
  Subject: [obm-l] dúvida fatorial


  Bom dia,

  Alguém saberia me informar por que 0! = 1?

  Obrigado

  Reginaldo


_
Seja um dos primeiros a testar o novo Windows Live Mail Beta. Acesse 
http://www.ideas.live.com/programpage.aspx?versionId=5d21c51a-b161-4314-9b0e-4911fb2b2e6d


=
Instruções para entrar na lista, sair da lista e usar a lista em
http://www.mat.puc-rio.br/~nicolau/olimp/obm-l.html
=



=
Instruções para entrar na lista, sair da lista e usar a lista em
http://www.mat.puc-rio.br/~nicolau/olimp/obm-l.html
=


[obm-l] Re: [obm-l] Re:[obm-l] Dúvida!

2006-02-04 Por tôpico Marcelo Salhab Brogliato



Olá,
primeiramente vamos analisar o seguinte 
problema:
x+y+z = k, x >= 0, y >= 0, z >= 
0
 
Imaginemos que vc tem k palitos de sorvete e 2 
pedras.. de quantos modos vc pode organiza-los?
(k+2)! / (k! 2!), certo?
que é igual a C(k+2, 2) .. combinação de k+2, 
tomados 2 a 2.
 
Agora, considere que cada palito é a unidade, e as 
pedras são os sinais de soma.. então, este tbem eh q quantidade de soluções 
inteiras e não negativas da equação.
Deu pra entender?
 
Analogamente vamos tentar resolver seu 
problema,
ax + by + cz = k, x>= 0, y >= 0, z >= 
0
 
Se pegarmos os casos em que x > 0, então ax 
>= a ... logo: ax - a >= 0
ax - a = X .. logo.. ax = X + a
Analogamente para os outros casos, entao, 
teremos:
X + a + Y + b + Z + c = k
X + Y + Z = k - a - b - c  X >= 0, Y >= 
0, Z >= 0
Bom, ja vimos que o numero de solucoes é: C(k - a - 
b - c, 2)
 
Agora, falta os casos em que eles podem ser zero.. 
entao:
x = 0... y > 0.. z > 0 => by + cz = k 
...
analogamente ao primeiro exemplo, teremos (k - b - 
c + 1)! / (k - b - c)! = C(k - b - c, 1)
 
Assim..
y = 0 => C(k - a - c, 1)
z = 0 => C(k - a - b, 1)
 
Agora, com 2 iguais a zero:
x = 0, y = 0... só terá solução se c | k 
podendo ser 1 ou 0
analogo para os outros..
 
Não tenho certeza da minha solucao.. apenas 
apliquei uma ideia que ja sabia (a que apresentei no comeco da 
mensagem)...
 
Abraços,
Salhab
 
 
 

  - Original Message - 
  From: 
  Luiz H. 
  Barbosa 
  To: obm-l 
  Sent: Saturday, February 04, 2006 9:13 
  PM
  Subject: [obm-l] Re:[obm-l] Dúvida!
  
  
  Como faço para achar o número de soluções de uma 
  equação do tipo ax+by+cz=k, de modo que a,b e c são 
  inteiros não-negativos e k um inteiro maior ou igual a 3?! 
  Para ser mais prático, como acharia o número de soluções de x+2y+3z=7, 
  sendo x,y e z inteiros não-negativos?! Será de suma importância a colaboração 
  dos senhores! 
   
  ===
  Bom , não conheço nenhum método.Mas você pode dar um jeitinho e resolver, 
  veja :
  x+2y+3z=7  (i)
  (x+y)+2(y+z) =7
  Isso quer dizer que a soma tem um multiplo de 2 e como os multiplos de 2 
  menores que 7 são 2,4 e 6 , dividimos o problema em 3 casos:
  1°) 2(y+z) = 2
  2°) 2(y+z) = 4
  3°)2(y+z) = 6
   
  Vou resolver só o primeiro e os demais são semelhantes:
  1°caso:
  y+z = 1 e consequentemente x+y = 5 .Resolvendo tudo em função de uma 
  variável, escolhi y , fica:
  x=4+y
  z=1-y
   
  Substituindo em (i) encontramos y=1 , com isso , x=5 e z=0.
  Faça o mesmo para os outros casos.
   
  []'s
  Luiz H. Barbosa 
  MSN: [EMAIL PROTECTED]


Re: [obm-l] RE: [obm-l] Re: [obm-l] Dúvida Raiz

2005-12-31 Por tôpico ricardo.bioni
O valor de x^(1/n) onde n é um número par é apenas o número positivo y tal que y^n = xSe for procurado o número negativo y tal que y^n = x então o valor procurado é y = -x^(1/n)


[obm-l] RE: [obm-l] Re: [obm-l] Dúvida Raiz

2005-12-31 Por tôpico Guilherme Neves
ah, completando minha resposta.. no campo dos complexos, utilize as formulas de De Moivre que você obterá as raízes com os dois sinais.


=
Instruções para entrar na lista, sair da lista e usar a lista em
http://www.mat.puc-rio.br/~nicolau/olimp/obm-l.html
=


[obm-l] Re: [obm-l] Re:[obm-l] Dúvida

2004-06-25 Por tôpico claudio.buffara

 
> Meu caro Cláudio,
>  
> fiquei me perguntando sobre a seguinte afirmação:
>  
> "Mas A pode ser particionado em pares nao ordenados da forma:
> {x,x^(-1)}"
>  
> O que garante que cada x pertencente a A tem seu inverso em A?
>
 
A eh o conjunto dos elementos de G que sao diferentes dos respectivos inversos.
Assim: 
x pertence a A <==> 
x <> x^(-1) <==> 
x^(-1) <> (x^(-1))^(-1) <==> 
x^(-1) pertence a A.   
 
***
 
No mais, aqui vai uma pequena correcao: a definicao precisa do conjunto B abaixo eh:
B = {x em G | x = x^(-1)  e  x <> e}
 
[]s,
Claudio.
 
"claudio.buffara" <[EMAIL PROTECTED]> wrote:

>
> Oi, Eder:
>  
> O Paulo Santa Rita usou uma bazuca pra matar uma barata.
>  
> Uma solucao mais simples seria a seguinte:
>  
> Particione G nos tres subconjuntos a seguir:
> {e},
> A = {x em G | x <> x^(-1)},
> B = {x em G | x = x^(-1)}.
>  
> Como G tem 2n elementos, A uniao B terah 2n - 1 elementos.
>  
> Mas A pode ser particionado em pares nao ordenados da forma:
> {x,x^(-1)}, jah que cada um dos elementos de A eh distinto do seu inverso.
> Isso significa que A tem um numero par de elementos, digamos 2m.
>  
> Logo, B terah 2n - 1 - 2m elementos, um numero impar e, portanto, >= 1.
>  
> Ou seja, deve existir algum x em G tal que x = x^(-1) <==> x^2 = e.
>  
> []s,
> Claudio.
>  
>



De:
[EMAIL PROTECTED]
>



Para:
[EMAIL PROTECTED]
>



Cópia:

>



Data:
Thu, 24 Jun 2004 07:02:59 -0300 (ART)
>



Assunto:
[obm-l] Dúvida
>



 
 
> > Gostaria que alguém me ajudasse com o problema abaixo:
> >  
> > Seja (G, . ) um grupo contento exatamente 2n elementos, n >=1. Prove que existe x <> e t.q. x^2 = x.x = e.
> >  
> > Obs.: (i) x <> e denota x diferente da unidade de (G, . );
> >     (ii) . é uma operação qualquer que torna G um grupo.
> >  
> > Grato, Éder.


Yahoo! Mail - Participe da pesquisa global sobre o Yahoo! Mail. Clique aqui! 


Yahoo! Mail - Participe da pesquisa global sobre o Yahoo! Mail. Clique aqui! 

[obm-l] Re:[obm-l] RE: [obm-l] Dúvida

2004-06-24 Por tôpico claudio.buffara

 




De:
[EMAIL PROTECTED]




Para:
[EMAIL PROTECTED]




Cópia:





Data:
Thu, 24 Jun 2004 14:38:52 -0300




Assunto:
[obm-l] RE: [obm-l] Dúvida




 
 
> Você também está usando o fato do grupo ser abeliano, não?
> 
> "Caso 2: pelo menos dois dos x_i sao distintos.
> Nesse caso, a classe vai conter exatamente p produtos:"
> 
> em especial está usando este fato:
> (x_1 * ... x_{p-1}) * x_p = x_p * (x_1 * ... * x_{p-1})
> 
Não. Não estou.
 
Lembre-se de que eu estou apenas considerando produtos de p elementos (não necessariamente distintos) de G cujo produto é a identidade de G.
 
No seu exemplo acima, o que acontece eh que x_p = (x_1*...*x_(p-1))^(-1) de modo que x_p e (x_1*...*x_(p-1)) comutam, uma vez que um elemento e seu inverso sempre comutam, em qualquer grupo, abeliano ou não.
 
[]s,
Claudio.
 

[obm-l] RE: [obm-l] Re:[obm-l] RE: [obm-l] Dúvida

2004-06-24 Por tôpico Paulo Santa Rita
Oi Claudio,
Se eu nao citei abeliano, foi esquecimento. O Teorema de Cauchy e assim:
"Seja G um grupo FINITO e ABELIANO. Se p e um primo que divide a ordem de G
entao existe um elemento g de G de ordem p".
Esta sua demonstracao ai embaixo e a do Kummer.
Um Abraco
Paulo Santa Rita
5,1356,240604
From: "claudio.buffara" <[EMAIL PROTECTED]>
Reply-To: [EMAIL PROTECTED]
To: "obm-l" <[EMAIL PROTECTED]>
Subject: [obm-l] Re:[obm-l]  RE: [obm-l] Dúvida
Date: Thu, 24 Jun 2004 12:43:59 -0300
Oi, Paulo:
Acho que esta sua demonstracao do teorema de Cauchy soh eh valida se G for 
abeliano, pois no fim, quando voce fala na projecao canonica p: G -> G/H, 
voce estah implicitamente supondo que G/H eh um grupo e, portanto, que H eh 
um subgrupo normal de G. Mas isso soh eh verdade para todo H se G for 
abeliano.

Por outro lado, existe uma demonstracao desse teorema que eh um dos meus 
exemplos favoritos de beleza matematica:

Seja G um grupo e p um primo que divide |G|.
Considere todos os produtos da forma x_1*x_2*...*x_p que sao iguais a "e", 
onde os x_i sao elementos nao necessariamente distintos de G.

Eh facil ver que existem |G|^(p-1) tais produtos pois, escolhendo-se 
livremente os valores de x_1, x_2, ..., x_(p-1), o valor de x_p fica 
unicamente determinado (igual ao inverso de x_1*x_2*...*x_(p-1))

Agora vamos dividir estes |G|^(p-1) produtos em classes de equivalencia de 
forma que dois produtos pertencem a uma mesma classe se e somente se um 
deles for uma permutacao circular do outro. Teremos dois casos a 
considerar:

Caso 1: todos os x_i sao iguais.
Nesse caso, a classe vai conter apenas um produto, pois x_1*x_2*...*x_p = 
a*a*...*a e existe apenas uma permutacao dos x_i.

Caso 2: pelo menos dois dos x_i sao distintos.
Nesse caso, a classe vai conter exatamente p produtos:
x_1*x_2*...*x_(p-1)*x_p;
x_2*x_3*... x_p*x_1;
x_3*x_4*...*x_1*x_2;
...
x_p*x_1*...x_(p-1)*x_(p-1).
Sejam N1 e N2 os numeros de classes de equivalencia de cada tipo.
Entao, teremos que:
numero de produtos =  1*N1 + p*N2 = |G|^(p-1).
Por hipotese, p | |G|^(p-1) e obviamente p | p*N2.
Logo, p | N1.
Alem disso, o produto e*e*...*e obviamente eh do tipo 1, de modo que N1 > 
0.

Ou seja, o numero N1 de produtos da forma a*a*...*a = a^p = e eh um 
multiplo positivo de p.
Em outras palavras, existem pelo menos p-1 elementos em G de ordem p.
Naturalmente, se a eh um tal elemento, entao , o subgrupo ciclico gerado 
por a, terah ordem p.

[]s,
Claudio.
De:[EMAIL PROTECTED]
Para:[EMAIL PROTECTED]
Cópia:
Data:Thu, 24 Jun 2004 14:20:38 +
Assunto:[obm-l] RE: [obm-l] Dúvida

> Ola Eder,
>
> Ok !
>
> Vamos fazer o seguinte. Vou provar um resultado classico que voce podera
> usar na solucao.
>
> TEOREMA DE CAUCHY : Se G e um grupo finito e "p" e um numero primo que
> divide
> a ordem de G entao existe um elemento "g" de G de ordem "p".
>
> PROVA : Vamos usar inducao sobre a ordem de G. Mais especificamente 
vamos
> mostrar que
> ( HIPOTESE DE INDUCAO ) se todos os grupos com ordem menor que G 
satisfazem
> o TEOREMA DE CAUCHY entao G satisfaz o TEOREMA DE CAUCHY.
>
> 1) Se a ordem de G for um numero primo, |G| = p, entao a prova e trivial 
e
> nem precisamos usar a hipotese de inducao, pois "p" sera o unico numero
> primo que pode dividir a ordem de G e se "g"
> for um elemento de G entao, pelo teorema de Lagrange, divide |G|, isto
> e, a ordem de
> "g" e "p". Assim, nao so um, mas todos os elementos de G ( com excecao 
da
> identidade ) tem
> ordem "p"
>
> 2) Se ordem de G nao for um numero primo, seja "p" um numero primo que
> divide a ordem de G.
> Tomando um elemento "g" pertencente a G, "g" diferente de "e", considere 
o
> subgrupo de G : H=. Existem duas possibilidades para H :
>
> PRIMEIRA : H e igual a G. Neste caso, G e ciclico com G=. Seja N=|G| e
> considere o elemento g^(N/p). Claramente que g^(N/p) pertence a G e 
ordem de
> g^(N/p) e "p". Assim,
> G tem um elemento de ordem "p" e acabou.
>
> SEGUNDA : H e diferente de G. Neste caso |H| < |G|.
>
> Se "p" divide |H|, pela HIPOTESE DE INDUCAO, existe "h" pertencente a H 
tal
> que ordem de "h" e "p". Como H e subconjunto de G segue que "h" e tambem
> elmento de G e, portanto, G tem um elemento de ordem "p" e acabou.
>
> Se "p" nao divide |H| ( mas "p" divide |G|, por hipotese ), pelo teorema 
de
> Lagrange |G|=|H|(G:H) teremos que "p" divide (G:H), isto e, "p" divide o
> indice de H em G. Como (G:H) =| G/H | e
> G/H| < |G|, pela HIPOTESE DE INDUCAO, existe um h_ ( h barra ) em G/H de
> ordem "p".
>
> Considere a projecao canonica :
>
>

[obm-l] Re:[obm-l] RE: [obm-l] Dúvida

2004-06-24 Por tôpico claudio.buffara

Oi, Paulo:
 
Acho que esta sua demonstracao do teorema de Cauchy soh eh valida se G for abeliano, pois no fim, quando voce fala na projecao canonica p: G -> G/H, voce estah implicitamente supondo que G/H eh um grupo e, portanto, que H eh um subgrupo normal de G. Mas isso soh eh verdade para todo H se G for abeliano.
 
Por outro lado, existe uma demonstracao desse teorema que eh um dos meus exemplos favoritos de beleza matematica:
 
Seja G um grupo e p um primo que divide |G|.
 
Considere todos os produtos da forma x_1*x_2*...*x_p que sao iguais a "e", onde os x_i sao elementos nao necessariamente distintos de G.
 
Eh facil ver que existem |G|^(p-1) tais produtos pois, escolhendo-se livremente os valores de x_1, x_2, ..., x_(p-1), o valor de x_p fica unicamente determinado (igual ao inverso de x_1*x_2*...*x_(p-1))
 
Agora vamos dividir estes |G|^(p-1) produtos em classes de equivalencia de forma que dois produtos pertencem a uma mesma classe se e somente se um deles for uma permutacao circular do outro. Teremos dois casos a considerar:
 
Caso 1: todos os x_i sao iguais.
Nesse caso, a classe vai conter apenas um produto, pois x_1*x_2*...*x_p = a*a*...*a e existe apenas uma permutacao dos x_i.
 
Caso 2: pelo menos dois dos x_i sao distintos.
Nesse caso, a classe vai conter exatamente p produtos:
x_1*x_2*...*x_(p-1)*x_p; 
x_2*x_3*... x_p*x_1;
x_3*x_4*...*x_1*x_2;
...
x_p*x_1*...x_(p-1)*x_(p-1).
 
Sejam N1 e N2 os numeros de classes de equivalencia de cada tipo.
Entao, teremos que:
numero de produtos =  1*N1 + p*N2 = |G|^(p-1).
 
Por hipotese, p | |G|^(p-1) e obviamente p | p*N2. 
Logo, p | N1.
 
Alem disso, o produto e*e*...*e obviamente eh do tipo 1, de modo que N1 > 0.
 
Ou seja, o numero N1 de produtos da forma a*a*...*a = a^p = e eh um multiplo positivo de p.
Em outras palavras, existem pelo menos p-1 elementos em G de ordem p.
Naturalmente, se a eh um tal elemento, entao , o subgrupo ciclico gerado por a, terah ordem p.
 
[]s,
Claudio.
 




De:
[EMAIL PROTECTED]




Para:
[EMAIL PROTECTED]




Cópia:





Data:
Thu, 24 Jun 2004 14:20:38 +




Assunto:
[obm-l] RE: [obm-l] Dúvida




 
 
> Ola Eder,
> 
> Ok !
> 
> Vamos fazer o seguinte. Vou provar um resultado classico que voce podera 
> usar na solucao.
> 
> TEOREMA DE CAUCHY : Se G e um grupo finito e "p" e um numero primo que 
> divide
> a ordem de G entao existe um elemento "g" de G de ordem "p".
> 
> PROVA : Vamos usar inducao sobre a ordem de G. Mais especificamente vamos 
> mostrar que
> ( HIPOTESE DE INDUCAO ) se todos os grupos com ordem menor que G satisfazem 
> o TEOREMA DE CAUCHY entao G satisfaz o TEOREMA DE CAUCHY.
> 
> 1) Se a ordem de G for um numero primo, |G| = p, entao a prova e trivial e 
> nem precisamos usar a hipotese de inducao, pois "p" sera o unico numero 
> primo que pode dividir a ordem de G e se "g"
> for um elemento de G entao, pelo teorema de Lagrange, divide |G|, isto 
> e, a ordem de
> "g" e "p". Assim, nao so um, mas todos os elementos de G ( com excecao da 
> identidade ) tem
> ordem "p"
> 
> 2) Se ordem de G nao for um numero primo, seja "p" um numero primo que 
> divide a ordem de G.
> Tomando um elemento "g" pertencente a G, "g" diferente de "e", considere o 
> subgrupo de G : H=. Existem duas possibilidades para H :
> 
> PRIMEIRA : H e igual a G. Neste caso, G e ciclico com G=. Seja N=|G| e 
> considere o elemento g^(N/p). Claramente que g^(N/p) pertence a G e ordem de 
> g^(N/p) e "p". Assim,
> G tem um elemento de ordem "p" e acabou.
> 
> SEGUNDA : H e diferente de G. Neste caso |H| < |G|.
> 
> Se "p" divide |H|, pela HIPOTESE DE INDUCAO, existe "h" pertencente a H tal 
> que ordem de "h" e "p". Como H e subconjunto de G segue que "h" e tambem 
> elmento de G e, portanto, G tem um elemento de ordem "p" e acabou.
> 
> Se "p" nao divide |H| ( mas "p" divide |G|, por hipotese ), pelo teorema de 
> Lagrange |G|=|H|(G:H) teremos que "p" divide (G:H), isto e, "p" divide o 
> indice de H em G. Como (G:H) =| G/H | e
> G/H| < |G|, pela HIPOTESE DE INDUCAO, existe um h_ ( h barra ) em G/H de 
> ordem "p".
> 
> Considere a projecao canonica :
> 
> p : G -> G/H
> 
> Sabemos que trata-se de um homomorfismo e que em todo homomorfismo a ORDEM 
> DA IMAGEM DE UM ELEMENTO DIVIDE A ORDEM DO ELEMENTO, isto e, |h_| divide |h| 
> para algum "h" em G. Como |h_| = p => |h| = kp, para algum k inteiro. 
> Considere o elemento h^k. Claramente que h^k pertence a G e | h^k | = p. 
> Assim, G tem um elemento de ordem "p".
> 
> Vemo que a hipotese de inducao vale ( por vacuidade ) para as ordem 1 e 
> tambem para a
> ordem 2. Segue - pelo que vimos acima - que vale para todas as ordens.
> 
> Um Abraco
> Paulo Santa Rita
> 5,1117,240604
> 
> >From: Lista OBM <[EMAIL PROTECTED]>
> >Reply-To: [EMAIL PROTECTED]
> >To: [EMAIL PROTECTED]
> >Subject: Re: [obm-l] RE: [obm-l] Dúvida
> >Date: Thu, 24 Jun 2004 10:08:22 -0300 (ART)
> >
> >Meu caro Paulo, entendi sua solução, o prblema que esse exercício 
> >encontra-se na seção de um >livro onde ainda

[obm-l] RE: [obm-l] Re:[obm-l] RE: [obm-l] Re:[obm-l] dúvida chara!

2004-05-23 Por tôpico Rogério Moraes de Carvalho
Olá Osvaldo,

Não há a necessidade de formalidades, mesmo porque eu não sou Dr..
Eu gostaria de ressaltar o seguinte comentário que eu coloquei no início dos
meus comentários, caso não tenha ficado claro: "A análise que eu apresento a
seguir corresponde a uma crítica de CARÁTER CONSTRUTIVO com relação à
resolução apresentada pelo Osvaldo. O objetivo desta análise não é depreciar
a resolução do Osvaldo, mas sim de mostrar que é necessário sermos rigorosos
nas resoluções de problemas de Matemática para não chegarmos a resultados 
incorretos. Muitas vezes podemos encontrar uma resposta correta para uma
questão resolvendo-a de maneira errada."

É importante ficar claro que mesmo para o conjunto dos números
inteiros a sua solução está incompleta. Observe que você não verificou se os
valores de "a" e "b" encontrados produzem valores inteiros de "x" e "y".
Neste caso, você não utilizaria a condição a >= b > 0 e encontraria todos os
valores inteiros de "a" e "b" que reproduzem o produto. Veja o exemplo da
questão modificada para constatar que a sua resolução apresentará resultados
inválidos mesmo no conjunto dos números inteiros.

Atenciosamente,

Rogério Moraes de Carvalho
-Original Message-
From: [EMAIL PROTECTED] [mailto:[EMAIL PROTECTED] On
Behalf Of Osvaldo
Sent: domingo, 23 de maio de 2004 17:54
To: obm-l
Subject: [obm-l] Re:[obm-l] RE: [obm-l] Re:[obm-l] dúvida chara!

Desculpe-me se fui parcial Dr., porém equivoquei-me ao 
ler o enunciado da questão. Eu apenas fiz os calculos 
para os números inteiros e não naturais, ou seja, 
inclui algumas possibilidades a mais. 
Obrigado pela observação!





> Olá colegas da lista,
> 
>   Apesar da resolução apresentada pelo Osvaldo 
ter seguido um possível
> raciocínio correto para resolver esta questão, a 
análise dele está
> incompleta porque omite alguns passos muito 
importantes, o que pode nos
> levar a encontrar soluções inválidas. Neste problema 
especificamente, a
> resposta encontrada está correta, porém, se 
modificarmos o valor da
> diferença de quadrados de 27 para outro valor, então 
a resolução dele pode
> nos levar a resultados errados.
> 
>   A análise que eu apresento a seguir 
corresponde a uma crítica de
> caráter construtivo com relação à resolução 
apresentada pelo Osvaldo. O
> objetivo desta análise não é depreciar a resolução 
do Osvaldo, mas sim de
> mostrar que é necessário sermos rigorosos nas 
resoluções de problemas de
> Matemática para não chegarmos a resultados 
incorretos. Muitas vezes podemos
> encontrar uma resposta correta para uma questão 
resolvendo-a de maneira
> errada.
> 
>   Na resolução apresentada abaixo, considere 
que "=>" significa
> "implica" e ">=" significa "maior ou igual a".
> 
> 
> QUESTÃO ORIGINAL:
> 
> "A diferença entre os quadrados de dois números 
naturais é 27. UM dos
> possíveis valores do quadrado da soma desses dois 
números:
> a)529
> b)625
> c)729
> d)841"
> 
> 
> RESOLUÇÃO POSSÍVEL:
> 
> Sejam x e y os dois números naturais, então devemos 
ter:
> x^2 - y^2 = 27 <=> (x + y)(x - y) = 27
> 
> Adotando a = x + y e b = x - y, teremos:
> a.b = 27 (i) (Observe que o produto de a e b é 
positivo)
> Resolvendo o sistema de equações nas variáveis x e 
y, podemos encontrar x e
> y em função de a e b:
> a + b = (x + y) + (x - y) <=> a + b = 2x <=> x = (a 
+ b)/2 (ii)
> a - b = (x + y) - (x - y) <=> a - b = 2y <=> y = (a -
 b)/2 (iii)
> 
> Como x e y são naturais, então x >= 0 e y >= 0. 
Portanto:
> x + y >= 0 + 0 => a >= 0. De acordo com a igualdade 
(i), a não pode ser 0,
> logo a > 0 (iv)
> Como a.b > 0 (i) e a > 0 (iv), então b > 0 (v)
> y >= 0 => -y <= 0 => y >= 0 e 0 >= -y => y >= -y => 
x + y >= x - y =>
> a >= b (vi)
> Por (v) e (vi), concluímos que: a >= b > 0 (vii)
> 
> Sendo assim, devemos encontrar a e b inteiros tais 
que sejam satisfeitas as
> seguintes condições:
> a.b = 27 (ii)
> a >= b > 0 (vii)
> x = (a + b)/2 (ii) seja um número natural.
> y = (a - b)/2 (iii) seja um número natural.
> 
> Analisando os divisores de 27, podemos concluir que 
existem apenas dois
> pares de valores de a e b que satisfazem as 
condições (ii) e (vii):
> (a = 27 e b = 1) ou (a = 9 e b = 3)
> 
> Para a = 27 e b = 1:
> x = (27 + 1)/2 = 14 é um número natural.
> y = (27 - 1)/2 = 13 é um número natural.
> Portanto, x = 14 e y = 13 é uma solução possível.
> 
> Para a = 9 e b = 3:
> x = (9 + 3)/2 = 6 é um número natural.
> y = (9 - 3)/2 = 3 é um número natural.
> Portanto, x = 6 e y = 3 é uma solução possível.
> 
> Possíveis valores para (x

[obm-l] Re:[obm-l] RE: [obm-l] Re:[obm-l] dúvida chara!

2004-05-23 Por tôpico Osvaldo
Desculpe-me se fui parcial Dr., porém equivoquei-me ao 
ler o enunciado da questão. Eu apenas fiz os calculos 
para os números inteiros e não naturais, ou seja, 
inclui algumas possibilidades a mais. 
Obrigado pela observação!





> Olá colegas da lista,
> 
>   Apesar da resolução apresentada pelo Osvaldo 
ter seguido um possível
> raciocínio correto para resolver esta questão, a 
análise dele está
> incompleta porque omite alguns passos muito 
importantes, o que pode nos
> levar a encontrar soluções inválidas. Neste problema 
especificamente, a
> resposta encontrada está correta, porém, se 
modificarmos o valor da
> diferença de quadrados de 27 para outro valor, então 
a resolução dele pode
> nos levar a resultados errados.
> 
>   A análise que eu apresento a seguir 
corresponde a uma crítica de
> caráter construtivo com relação à resolução 
apresentada pelo Osvaldo. O
> objetivo desta análise não é depreciar a resolução 
do Osvaldo, mas sim de
> mostrar que é necessário sermos rigorosos nas 
resoluções de problemas de
> Matemática para não chegarmos a resultados 
incorretos. Muitas vezes podemos
> encontrar uma resposta correta para uma questão 
resolvendo-a de maneira
> errada.
> 
>   Na resolução apresentada abaixo, considere 
que "=>" significa
> "implica" e ">=" significa "maior ou igual a".
> 
> 
> QUESTÃO ORIGINAL:
> 
> "A diferença entre os quadrados de dois números 
naturais é 27. UM dos
> possíveis valores do quadrado da soma desses dois 
números:
> a)529
> b)625
> c)729
> d)841"
> 
> 
> RESOLUÇÃO POSSÍVEL:
> 
> Sejam x e y os dois números naturais, então devemos 
ter:
> x^2 - y^2 = 27 <=> (x + y)(x - y) = 27
> 
> Adotando a = x + y e b = x - y, teremos:
> a.b = 27 (i) (Observe que o produto de a e b é 
positivo)
> Resolvendo o sistema de equações nas variáveis x e 
y, podemos encontrar x e
> y em função de a e b:
> a + b = (x + y) + (x - y) <=> a + b = 2x <=> x = (a 
+ b)/2 (ii)
> a - b = (x + y) - (x - y) <=> a - b = 2y <=> y = (a -
 b)/2 (iii)
> 
> Como x e y são naturais, então x >= 0 e y >= 0. 
Portanto:
> x + y >= 0 + 0 => a >= 0. De acordo com a igualdade 
(i), a não pode ser 0,
> logo a > 0 (iv)
> Como a.b > 0 (i) e a > 0 (iv), então b > 0 (v)
> y >= 0 => -y <= 0 => y >= 0 e 0 >= -y => y >= -y => 
x + y >= x - y =>
> a >= b (vi)
> Por (v) e (vi), concluímos que: a >= b > 0 (vii)
> 
> Sendo assim, devemos encontrar a e b inteiros tais 
que sejam satisfeitas as
> seguintes condições:
> a.b = 27 (ii)
> a >= b > 0 (vii)
> x = (a + b)/2 (ii) seja um número natural.
> y = (a - b)/2 (iii) seja um número natural.
> 
> Analisando os divisores de 27, podemos concluir que 
existem apenas dois
> pares de valores de a e b que satisfazem as 
condições (ii) e (vii):
> (a = 27 e b = 1) ou (a = 9 e b = 3)
> 
> Para a = 27 e b = 1:
> x = (27 + 1)/2 = 14 é um número natural.
> y = (27 - 1)/2 = 13 é um número natural.
> Portanto, x = 14 e y = 13 é uma solução possível.
> 
> Para a = 9 e b = 3:
> x = (9 + 3)/2 = 6 é um número natural.
> y = (9 - 3)/2 = 3 é um número natural.
> Portanto, x = 6 e y = 3 é uma solução possível.
> 
> Possíveis valores para (x + y)^2:
> (x + y)^2 = (14 + 13)^2 = 27^2 = 729
> (x + y)^2 = (6 + 3)^2 = 9^2 = 81
> 
> Resposta: Alternativa c
> 
> 
> Observação: Pode parecer que os passos apresentados 
para deduzir as
> condições são desnecessários, mas são eles que 
garantem a validade das
> soluções encontradas.
> 
> 
> EXPLICAÇÃO DO MOTIVO DA RESOLUÇÃO APRESENTADA PELO 
OSVALDO SER INCOMPLETA:
> 
> Na resolução são apresentados 4 valores possíveis 
para a e b (a,b):
> {(1,27),(3,9),(9,3),(27,1)}. Porém, (1,27) e (3,9) 
não satisfazem a condição
> (vii): a >= b > 0. Portanto, somente os pares (9,3) 
e (27,1) correspondem a
> possíveis valores para a e b, restando apenas 
verificar se eles produzem
> valores naturais para x e y. Logo, na lista de 
valores apresentados para
> (x+y)^2 = a^2, {1, 9, 81, 729}, não poderia aparecer 
os valores 1 = 1^2 e
> nem 9 = 3^2. Além disto, não há garantia de que 81 = 
9^2 e 729 = 27^2
> correspondem a valores de a e b válidos, pois os 
valores de x e y não são
> calculados para verificar se eles são naturais, como 
foi descrito no
> enunciado do problema. Portanto, os valores de a e b 
encontrados poderiam
> não ser válidos. Neste problema específico, os 
valores de a e b encontrados
> são válidos, logo a resposta encontrada está 
correta. A seguir, eu apresento
> uma variação deste problema que mostra de maneira 
concreta que a resolução
> apresentada pelo Osvaldo pode apresentar resultados 
errados. Para se ter uma
> idéia apenas 1 resultado, dos 6 encontrados, é 
correto!
> 
> 
> 
> QUESTÃO MODIFICADA:
> 
> "A diferença entre os quadrados de dois números 
naturais é 68. UM dos
> possíveis valores do quadrado da soma desses dois 
números:
> a)16
> b)289
> c)1156
> d)4624"
> 
> 
> RESOLUÇÃO DO OSVALDO ALTERADA PARA A VERSÃO 
MODIFICADA DA QUESTÃO:
> 
> sejam x e y tais numeros, dai temos que
> x^2-y^2=68
> 
> (x+y)(x-y)=68
> 
> 
> a=x+y

[obm-l] RE: [obm-l] Re:[obm-l] dúvida chara!

2004-05-23 Por tôpico Rogério Moraes de Carvalho
Olá colegas da lista,

Apesar da resolução apresentada pelo Osvaldo ter seguido um possível
raciocínio correto para resolver esta questão, a análise dele está
incompleta porque omite alguns passos muito importantes, o que pode nos
levar a encontrar soluções inválidas. Neste problema especificamente, a
resposta encontrada está correta, porém, se modificarmos o valor da
diferença de quadrados de 27 para outro valor, então a resolução dele pode
nos levar a resultados errados.

A análise que eu apresento a seguir corresponde a uma crítica de
caráter construtivo com relação à resolução apresentada pelo Osvaldo. O
objetivo desta análise não é depreciar a resolução do Osvaldo, mas sim de
mostrar que é necessário sermos rigorosos nas resoluções de problemas de
Matemática para não chegarmos a resultados incorretos. Muitas vezes podemos
encontrar uma resposta correta para uma questão resolvendo-a de maneira
errada.

Na resolução apresentada abaixo, considere que "=>" significa
"implica" e ">=" significa "maior ou igual a".


QUESTÃO ORIGINAL:

"A diferença entre os quadrados de dois números naturais é 27. UM dos
possíveis valores do quadrado da soma desses dois números:
a)529
b)625
c)729
d)841"


RESOLUÇÃO POSSÍVEL:

Sejam x e y os dois números naturais, então devemos ter:
x^2 - y^2 = 27 <=> (x + y)(x - y) = 27

Adotando a = x + y e b = x - y, teremos:
a.b = 27 (i) (Observe que o produto de a e b é positivo)
Resolvendo o sistema de equações nas variáveis x e y, podemos encontrar x e
y em função de a e b:
a + b = (x + y) + (x - y) <=> a + b = 2x <=> x = (a + b)/2 (ii)
a - b = (x + y) - (x - y) <=> a - b = 2y <=> y = (a - b)/2 (iii)

Como x e y são naturais, então x >= 0 e y >= 0. Portanto:
x + y >= 0 + 0 => a >= 0. De acordo com a igualdade (i), a não pode ser 0,
logo a > 0 (iv)
Como a.b > 0 (i) e a > 0 (iv), então b > 0 (v)
y >= 0 => -y <= 0 => y >= 0 e 0 >= -y => y >= -y => x + y >= x - y =>
a >= b (vi)
Por (v) e (vi), concluímos que: a >= b > 0 (vii)

Sendo assim, devemos encontrar a e b inteiros tais que sejam satisfeitas as
seguintes condições:
a.b = 27 (ii)
a >= b > 0 (vii)
x = (a + b)/2 (ii) seja um número natural.
y = (a - b)/2 (iii) seja um número natural.

Analisando os divisores de 27, podemos concluir que existem apenas dois
pares de valores de a e b que satisfazem as condições (ii) e (vii):
(a = 27 e b = 1) ou (a = 9 e b = 3)

Para a = 27 e b = 1:
x = (27 + 1)/2 = 14 é um número natural.
y = (27 - 1)/2 = 13 é um número natural.
Portanto, x = 14 e y = 13 é uma solução possível.

Para a = 9 e b = 3:
x = (9 + 3)/2 = 6 é um número natural.
y = (9 - 3)/2 = 3 é um número natural.
Portanto, x = 6 e y = 3 é uma solução possível.

Possíveis valores para (x + y)^2:
(x + y)^2 = (14 + 13)^2 = 27^2 = 729
(x + y)^2 = (6 + 3)^2 = 9^2 = 81

Resposta: Alternativa c


Observação: Pode parecer que os passos apresentados para deduzir as
condições são desnecessários, mas são eles que garantem a validade das
soluções encontradas.


EXPLICAÇÃO DO MOTIVO DA RESOLUÇÃO APRESENTADA PELO OSVALDO SER INCOMPLETA:

Na resolução são apresentados 4 valores possíveis para a e b (a,b):
{(1,27),(3,9),(9,3),(27,1)}. Porém, (1,27) e (3,9) não satisfazem a condição
(vii): a >= b > 0. Portanto, somente os pares (9,3) e (27,1) correspondem a
possíveis valores para a e b, restando apenas verificar se eles produzem
valores naturais para x e y. Logo, na lista de valores apresentados para
(x+y)^2 = a^2, {1, 9, 81, 729}, não poderia aparecer os valores 1 = 1^2 e
nem 9 = 3^2. Além disto, não há garantia de que 81 = 9^2 e 729 = 27^2
correspondem a valores de a e b válidos, pois os valores de x e y não são
calculados para verificar se eles são naturais, como foi descrito no
enunciado do problema. Portanto, os valores de a e b encontrados poderiam
não ser válidos. Neste problema específico, os valores de a e b encontrados
são válidos, logo a resposta encontrada está correta. A seguir, eu apresento
uma variação deste problema que mostra de maneira concreta que a resolução
apresentada pelo Osvaldo pode apresentar resultados errados. Para se ter uma
idéia apenas 1 resultado, dos 6 encontrados, é correto!



QUESTÃO MODIFICADA:

"A diferença entre os quadrados de dois números naturais é 68. UM dos
possíveis valores do quadrado da soma desses dois números:
a)16
b)289
c)1156
d)4624"


RESOLUÇÃO DO OSVALDO ALTERADA PARA A VERSÃO MODIFICADA DA QUESTÃO:

sejam x e y tais numeros, dai temos que
x^2-y^2=68

(x+y)(x-y)=68


a=x+y
b=x-y

Possiveis valores para a e b (x,y):

{(1,68),(2,34),(4,17),(17,4),(34,2),(68,1)}

Assim (x+y)^2=a^2

Temos então que todos os valores de (x+y)^2 pertencem a
{1, 4, 16, 289, 1156, 4624)

Logo quatro dos valores possiveis são 16, 289, 1156 e 4624
resposta a, b, c, d



RESOLUÇÃO CORRETA POSSÍVEL PARA A QUESTÃO MODIFICADA:

Sejam x e y os dois números naturais, então devemos ter:
x^2 - y^2 = 68 <=> (x + y)(x - y) = 68

Adotando a = x + y e b = x - y, teremos:
a.b = 68 (i) (Observe que o produto de a e b é positi

[obm-l] Re: [obm-l] Re: [obm-l] dúvida PARA FAELCCMM

2004-05-08 Por tôpico Fernando Romualdo Braga



Se considerarmos apenas os cortes ADICIONAIS a 
partir do primeiro, a alternativa correta é a C- 6 e 25º, o total de cortes para 
se obter os 14 pedaços são 7.
 
- Original Message - 
From: TSD 
To: [EMAIL PROTECTED] 
Sent: Saturday, May 08, 2004 8:43 PM
Subject: [obm-l] Re: [obm-l] dúvida PARA FAELCCMM

a questão pergunta quantos cortes 
ADICIONAIS

  - Original Message - 
  From: 
  [EMAIL PROTECTED] 
  
  To: [EMAIL PROTECTED] 
  Sent: Saturday, May 08, 2004 7:01 
PM
  Subject: Re: [obm-l] dúvida
  Para 1 corte > 2 pedaços 2 
  cortes > 4 pedaços 3 cortes > 6 pedaços n cortes 
  > 2n pedaços Logo para 14 pedaços serao necessarios 7 cortes. 
  O angulo da fatia = 360/ 14 ~ 26 Alternativa B 
  Em uma mensagem de 8/5/2004 18:49:43 Hora padrão leste da 
  Am. Sul, [EMAIL PROTECTED] escreveu: 
  
  Para 1 corte > 2 pedaço 2 corte > 
4 pedaços 3 corte > 6 pedaços n corte > 2n pedaços 
Logo para 14 pedaços serao necessarios 7 cortes. O angulo da 
fatia = 360/ 14 ~ 26 Alternativa B Em uma mensagem 
de 8/5/2004 18:38:48 Hora padrão leste da Am. Sul, [EMAIL PROTECTED] 
escreveu: 
17 - Uma pizza de formato circular será cortada em 
  fatias pormeio de cortes quesão segmentos de reta que passam pelo"centro" 
  da pizza e medem um diâmetro. A figura a seguir mostra que, com um corte, 
  a pizza fica dividida em duas fatias iguais.( é uma circunferencia com um 
  traço diametro) Para se obter quatorze fatias iguais, a partir 
  do primeiro corte,serão necessários cortes adicionais na 
  pizza. Assinale o item que indica o número de 
  cortes adicionais que terão de ser feitos e o ângulo aproximado que cada 
  fatia, imaginada como um setor circular, terá: (A) 8 e 
  30o;(B) 7 e 26o ;(C) 6 e 25o;(D) 7 e 30o; (E) 8 e 26o. 


[obm-l] Re:[obm-l] Re: [obm-l] Re:[obm-l] dúvida

2004-04-25 Por tôpico rickufrj
-- Início da mensagem original ---

  De: [EMAIL PROTECTED]
Para: [EMAIL PROTECTED]
  Cc: 
Data: Sun, 25 Apr 2004 10:10:46 -0300
 Assunto: [obm-l] Re: [obm-l] Re:[obm-l] dúvida

> No caso, não entendi o porque do i*(raiz de 1998), 
visto que ao elevarmos ao
> quadrado,  i^2= -1 e a expressão seria -1998
> 
> Creio que apenas (raiz de 1998) seja mais correto.
> 
> Porém a questão não deve ser apenas isto, a e b 
devem pertencer a algum
> conjunto específico como os Inteiros...
> 
> Abraços,
> Rossi
> 
É verdade Rossi , acho que ontem a noite eu estava 
meio 'grog' ... rs
Mas , tenta fazer essa questão por calculo .Tipo , é 
obvio que a diferença mínima é 0 , mas quando vc faz 
isso formalmente , aparece i na raiz . É bem 
estranho , na hora que eu fiz eu não consegui ver o pq 
disso.
Abraço
Luiz H. 

 
__
Acabe com aquelas janelinhas que pulam na sua tela.
AntiPop-up UOL - É grátis!
http://antipopup.uol.com.br/



=
Instruções para entrar na lista, sair da lista e usar a lista em
http://www.mat.puc-rio.br/~nicolau/olimp/obm-l.html
=


[obm-l] Re: [obm-l] Re:[obm-l] dúvida

2004-04-25 Por tôpico Fellipe Rossi
No caso, não entendi o porque do i*(raiz de 1998), visto que ao elevarmos ao
quadrado,  i^2= -1 e a expressão seria -1998

Creio que apenas (raiz de 1998) seja mais correto.

Porém a questão não deve ser apenas isto, a e b devem pertencer a algum
conjunto específico como os Inteiros...

Abraços,
Rossi

- Original Message -
From: "rickufrj" <[EMAIL PROTECTED]>
To: "obm-l" <[EMAIL PROTECTED]>
Sent: Sunday, April 25, 2004 2:40 AM
Subject: [obm-l] Re:[obm-l] dúvida


> -- Início da mensagem original ---
>
>   De: [EMAIL PROTECTED]
> Para: [EMAIL PROTECTED]
>   Cc:
> Data: Sat, 24 Apr 2004 23:31:13 -0300
>  Assunto: [obm-l] dúvida
>
> > como é que eu resolvo este inequação de maneira
> inteligente!!!
> >
> > | (x+1)/(-x)| >=0
> >
> > a expresão acima está em módulo.
> >
> > outra dúvida é:
> > a x b = 1998 .Sabendo que "a" e "b" são tais números
> que a diferença entre eles seja a menor possível.
> >
> ===
> Na desigualdade ,qualquer valor para x é válido .
> Já na segunda questão , se a e b puderem ser
> complexos , então a = b = i*[sqrt(1998)] .Sendo a
> menor difereça igual a zero.
>
> __
> Acabe com aquelas janelinhas que pulam na sua tela.
> AntiPop-up UOL - É grátis!
> http://antipopup.uol.com.br/
>
>
>
> =
> Instruções para entrar na lista, sair da lista e usar a lista em
> http://www.mat.puc-rio.br/~nicolau/olimp/obm-l.html
> =
>


=
Instruções para entrar na lista, sair da lista e usar a lista em
http://www.mat.puc-rio.br/~nicolau/olimp/obm-l.html
=


[obm-l] Re: [obm-l] Re: [obm-l] Re: [obm-l] Dúvida persistente!!!

2004-04-13 Por tôpico Rafael
Auggy,

Independentemente das contas, a criatividade na construção dos triângulos é
magnífica. Lendo o link, vi que o Cláudio já havia pensado no cálculo da
área por integral e teve uma idéia muito melhor em relação à posição dos
eixos, com origem em B em vez de A.

Enfim, apesar de trabalhoso, é um problema bonito.


Cláudio,

Parabéns por ambas as soluções!


Abraços,

Rafael de A. Sampaio





- Original Message -
From: "Qwert Smith" <[EMAIL PROTECTED]>
To: <[EMAIL PROTECTED]>
Sent: Tuesday, April 13, 2004 5:41 PM
Subject: RE: [obm-l] Re: [obm-l] Re: [obm-l] Dúvida persistente!!!


Sai na geometria mas da umas contas chatas.

na primeira figura:
---
(area em amarelo)  = (area do circulo menor) - 2*(area em verde)

(area em vermelho) = (area do quadrado) -
1/4*{(area do circulo maior) + [(area do quadrado)-(area circulo menor)]} -
(area em verde)
---

na segunda figura:
---
(area em laranja) = (setor circular PBQ) - 2*(triangulo PBO - area em azul)
(area em verde) = (setor circular POQ) - (area em laranja)
--

Como todos os lados do triangulo sao conhecidos (em funcao do lado do
quadrado).  Agulos e areas sao questao de braco.

A descricao da figura 2 vc encontra no link

http://www.mat.puc-rio.br/~nicolau/olimp/obm-l.200310/msg00574.html

em uma menssagem do grande Claudio BUffara que ainda teve paciencia
montruosa de me explicar em off o problema.

Em meu email anterior eu tinha feito confusao e atribuido a mensagem do link
acima a outro fera, o Paulo Santa Rita que mandou uma mensagem sobre "lua
algebrica", que tb vale a pena conferir.
E' muito genio pra keep track.

Valeu Super Buffara!

[]s,
Auggy



=
Instruções para entrar na lista, sair da lista e usar a lista em
http://www.mat.puc-rio.br/~nicolau/olimp/obm-l.html
=


RE: [obm-l] Re: [obm-l] Re: [obm-l] Dúvida persistente!!!

2004-04-13 Por tôpico Qwert Smith
Sai na geometria mas da umas contas chatas.

na primeira figura:
---
(area em amarelo)  = (area do circulo menor) - 2*(area em verde)
(area em vermelho) = (area do quadrado) -
1/4*{(area do circulo maior) + [(area do quadrado)-(area circulo menor)]} -
(area em verde)
---
na segunda figura:
---
(area em laranja) = (setor circular PBQ) - 2*(triangulo PBO - area em azul)
(area em verde) = (setor circular POQ) - (area em laranja)
--
Como todos os lados do triangulo sao conhecidos (em funcao do lado do 
quadrado).  Agulos e areas sao questao de braco.

A descricao da figura 2 vc encontra no link

http://www.mat.puc-rio.br/~nicolau/olimp/obm-l.200310/msg00574.html

em uma menssagem do grande Claudio BUffara que ainda teve paciencia 
montruosa de me explicar em off o problema.

Em meu email anterior eu tinha feito confusao e atribuido a mensagem do link 
acima a outro fera, o Paulo Santa Rita que mandou uma mensagem sobre "lua 
algebrica", que tb vale a pena conferir.
E' muito genio pra keep track.

Valeu Super Buffara!

[]s,
Auggy








From: "Rafael" <[EMAIL PROTECTED]>
Reply-To: [EMAIL PROTECTED]
To: "OBM-L" <[EMAIL PROTECTED]>
Subject: [obm-l] Re: [obm-l] Re: [obm-l] Dúvida persistente!!!
Date: Tue, 13 Apr 2004 03:20:58 -0300
Eu desisto...

Tentei encontrar uma solução simples, como pedia o Eduardo, mas a melhor
forma que vejo agora é calcular, por integral, a área verde e só depois
encontrar a área amarela.
Minha idéia é pôr a circunferência de centro A na origem do sistema de
coordenadas; o lado do quadrado não será mais x, e sim R; a equação da
circunferência citada será x^2 + y^2 = R^2. A circunferência inscrita no
quadrado terá equação: (x-R/2)^2 + (y+R/2)^2 = R^2/4. Os pontos de
intersecção das equações são:
( R*(5 + sqrt(7))/8 ; R*(sqrt(7) - 5)/8 )

e

( R*(5 - sqrt(7))/8 ; -R*(5 + sqrt(7))/8 )

A área S amarela será dada por:

S = Pi * R^2/4 - 2*(Integral[- sqrt(R^2 - x^2)] dx -
- Integral[- R/2 + sqrt(x*R - x^2)] dx)
O intervalo das integrais é [R*(5 - sqrt(7))/8 ; R*(5 + sqrt(7))/8].

Depois de muito trabalho algébrico (deixado para o Mathematica),
voltando de R para x, chegamos à expressãozinha anexada a esta mensagem,
por razões óbvias...
Dá para entender o porquê de a questão ser persistente...

Abraços,

Rafael de A. Sampaio





- Original Message -
From: "Rafael" <[EMAIL PROTECTED]>
To: <[EMAIL PROTECTED]>
Sent: Sunday, April 11, 2004 3:12 AM
Subject: [obm-l] Re: [obm-l] Dúvida persistente!!!
Obrigado pelo elogio à figura, Qwert.

Na verdade, o que tornou a minha solução errada foi não ter somado quatro
vezes a área vermelha, pois cada uma acabou sendo subtraída duas vezes. 
Pelo
que vejo, descobrindo a área vermelha, teremos a área amarela (que foi a 
que
pretendi calcular) e a diferença da área do círculo menor (de raio x) com
esta área amarela é, precisamente, a área verde.

Descobrir essa área vermelha é que não me parece muito fácil...



<< FigColor.gif >>
<< result.gif >>
_
FREE pop-up blocking with the new MSN Toolbar – get it now! 
http://toolbar.msn.com/go/onm00200415ave/direct/01/
<><>

  1   2   >